Study Questions Aao BCSC 2024-2025
Study Questions Aao BCSC 2024-2025
Please note that these questions are not part of your CME reporting process. They are provided
here for your own educational use and for identification of any professional practice gaps. The
required CME posttest is available online (see “Requesting Continuing Medical Education
Credit”). Following the questions are answers with discussions. Although a concerted effort
has been made to avoid ambiguity and redundancy in t hese questions, the authors recognize
that differences of opinion may occur regarding the “best” answer. The discussions are pro-
vided to demonstrate the rationale used to derive the answer. They may also be helpful in con-
firming that your approach to the problem was correct or, if necessary, in fixing the principle
in your memory. The Section 1 faculty thanks the Resident Self-Assessment Committee for
developing these self-assessment questions and the discussions that follow.
2. The National Eye Institute Visual Function Questionnaire 25 (VFQ-25) asks patients to
rate their difficulty with certain tasks, such as reading street signs, on a scale from 1 to 5,
with 1 being “No difficulty at all” and 5 being “Stopped doing this b ecause of your eye-
sight.” In terms of statistics, what type of variable is being collected?
a. ordinal
b. categorical
c. dichotomous
d. continuous
3. What is the mechanism of action of teprotumumab in the treatment of thyroid eye disease?
a. inhibits thyroid-stimulating hormone (TSH) secretion
b. competitively binds the TSH receptor
c. competitively binds thyroid-stimulating immunoglobulin
d. inhibits the insulin-like growth factor I receptor
4. What is the most common cancer in multiple endocrine neoplasia syndromes 2 and 3?
a. gastrinoma
b. medullary thyroid cancer
c. ganglioneuroma
d. pheochromocytoma
329
330 ● Study Questions
6. What is the role of injectable PCSK9 inhibitors such as evolocumab and alirocumab in the
treatment of hyperlipidemia?
a. initial treatment for children and adolescents with low-density-lipoprotein cholesterol
(LDL-C) levels of ≥190 mg/dL
b. adjunctive therapy for high-risk patients who cannot achieve LDL-C goal with statins
alone
c. initial treatment for diabetic patients with LDL-C levels of 70–189 mg/dL
d. adjunctive therapy for diabetic patients with LDL-C levels of ≤70 mg/dL
7. What is the focus of treatment for a patient with congestive heart failure with a reduced
ejection fraction (HFrEF)?
a. Reduce preload by decreasing circulating blood volume with diuretics.
b. Reduce afterload by lowering blood pressure.
c. Treat associated cardiac ischemia with percutaneous coronary intervention (PCI) or
coronary artery bypass grafting (CABG).
d. Treat associated hyperlipidemia with statins.
8. What is the preferred first-line therapy for patients who have survived a cardiac arrest?
a. implantable cardioverter-defibrillator (ICD)
b. PCI with drug-eluting stenting
c. CABG
d. digoxin therapy
9. A 65-year-old man with diabetes and hypertension is brought to the emergency depart-
ment 2 hours after the onset of weakness on the right side. What imaging study is the most
important in guiding urgent management?
a. brain magnetic resonance imaging
b. cerebral angiography
c. noncontrast computed tomography of the brain
d. transcranial Doppler ultrasonography
Study Questions ● 331
10. What antithrombotic therapy is most effective in the early treatment of ischemic stroke?
a. aspirin
b. aspirin/clopidogrel
c. enoxaparin
d. heparin
12. What type of stem cell is the precursor of red blood cells?
a. myeloid
b. mesenchymal
c. lymphoid
d. induced pluripotent
14. What is the most common hypercoagulable state implicated in retinal vein occlusions in
young White patients?
a. activated protein C resistance (factor V Leiden mutation)
b. prothrombin G20210A mutation
c. protein C deficiency
d. protein S deficiency
15. During a routine eye examination, multiple telangiectatic lesions are found on a patient’s
palpebral conjunctiva. Some lesions are also present on the lips. The patient reports that
the lip lesions sometimes bleed spontaneously or with minimal trauma. What disorder is
consistent with these findings?
a. hereditary hemorrhagic telangiectasia (Rendu-Osler-Weber disease)
b. Ehlers-Danlos syndrome
c. osteogenesis imperfecta
d. pseudoxanthoma elasticum
332 ● Study Questions
16. A 65-year-old woman with symmetric bilateral arthritis presents with ulnar deviation and
a positive anti–cyclic citrullinated peptide antibody test result. What ocular manifestation
is commonly associated with this systemic disease?
a. retinal vasculitis
b. optic neuritis
c. scleritis and episcleritis
d. trabeculitis
17. What form of juvenile idiopathic arthritis is most likely to have ocular involvement?
a. oligoarticular
b. polyarticular
c. psoriatic
d. systemic
18. A 40-year-old man presents with mild bilateral conjunctivitis with a mucopurulent dis-
charge. He has dysuria, which started a few weeks ago and was followed by pain in his left
knee and ankle. Laboratory testing is performed, revealing that he is HLA-B27 positive.
What is the most likely diagnosis?
a. reactive arthritis
b. relapsing polychondritis
c. ankylosing spondylitis
d. rheumatoid arthritis
19. What test is most confirmatory for the diagnosis of Sjögren syndrome in a patient without
autoantibodies?
a. Schirmer testing
b. ocular surface staining
c. salivary gland biopsy
d. buccal mucosal biopsy
21. An 89-year-old patient with a history of frequent bruises, repeated trips to the emergency
department, and poor adherence to follow-up instructions presents for an ophthalmic
examination. What initial management is appropriate?
a. Refer the patient for low vision evaluation.
b. Examine the patient for retinal hemorrhages.
Study Questions ● 333
22. What ocular finding may be present in patients with bulimia nervosa?
a. pinpoint conjunctival hemorrhages
b. papilledema
c. traumatic mydriasis
d. corneal ectasia
23. A 74-year-old man has had trouble reading for 18 months. He underwent uncompli-
cated cataract surgery in both eyes with no noticeable improvement. His family reports
some gradual cognitive decline over the past year. His visual acuity is normal at near
and distance. The posterior chamber intraocular lenses are well centered, with clear pos-
terior capsules. Fundus examination results are normal. Humphrey visual fields show a
left homonymous hemianopia. Magnetic resonance imaging findings are unremarkable.
What diagnosis would explain his symptoms?
a. cancer-associated retinopathy
b. melanoma-associated retinopathy
c. posterior cortical atrophy
d. right occipital stroke
24. According to the US Preventive Services Task Force, in which group of individuals should
yearly prostate-specific antigen testing be done to screen for prostate cancer?
a. all 55-year-old men
b. men of Asian descent
c. 60-year-old men with a positive family history
d. 70-year-old men regardless of family history
25. What is the leading cause of viral hepatitis in the United States?
a. hepatitis A
b. hepatitis B
c. hepatitis C
d. hepatitis D
28. A healthy 52-year-old former nuclear submarine engineer asks whether his previous oc-
cupation exposed him to any health risks. What ocular complication associated with ra-
diation exposure is most likely to be observed on slit-lamp examination?
a. punctate epithelial erosions
b. posterior subcapsular cataract
c. cotton-wool spots
d. optic nerve head hemorrhages
29. What is the term for chemotherapy given before surgical resection, with the goal of tumor
shrinkage and, therefore, a less invasive surgical procedure?
a. adjuvant
b. curative
c. neoadjuvant
d. palliative
30. What is the most serious ocular adverse effect of ipilimumab and other immune check-
point inhibitors in the treatment of cancer?
a. neurotrophic keratitis
b. uveitis
c. optic neuritis
d. nonarteritic anterior ischemic optic neuropathy
31. What was a key finding of the Antibiotic Resistance Monitoring in Ocular Microorgan-
isms (ARMOR) study?
a. Methicillin resistance in staphylococcal isolates was rare.
b. Fluoroquinolones remained effective against nearly all isolates.
c. Vancomycin was effective against all staphylococcal isolates.
d. Antibiotic resistance of ocular isolates increased during the 10-year study period.
32. A 73-year-old man presents with mild preseptal cellulitis of the left lower eyelid, for which
he is prescribed oral clindamycin. Two days later, the patient is seen for a follow-up ap-
pointment, and t here is significant improvement in eyelid erythema and edema. Five days
later, the patient calls the ophthalmologist’s office and reports having a fever and non-
bloody diarrhea. What diagnostic test would be most appropriate?
a. blood culture
b. stool ova and parasites (O&P)
Study Questions ● 335
33. What intervention has been shown to decrease the incidence of Haemophilus influenzae
infection, which can cause orbital cellulitis?
a. isolation of infected individuals
b. improved sanitation
c. treatment of otitis media with antibiotics
d. vaccination
34. What stage of Treponema pallidum infection is characterized by fever, malaise, lymph-
adenopathy, and hair loss?
a. primary
b. secondary
c. latent
d. tertiary
35. What testing should be included when screening for ethambutol optic neuropathy?
a. central visual field testing
b. fluorescein angiography
c. macular optical coherence tomography
d. Goldmann kinetic perimetry
36. For which patients should preoperative testing, such as electrocardiography and routine
blood testing, be done prior to ophthalmic surgery?
a. patients who have not seen their primary care physician within 1 year
b. patients for whom the tests would be indicated even if they w ere not planning
surgery
c. patients with American Society of Anesthesiologists Physical Status class II
d. patients with American Society of Anesthesiologists Physical Status class III
37. You are trying to make a diagnosis of myasthenia gravis in clinic and administer intrave-
nous edrophonium chloride to a patient. The patient then becomes diaphoretic, nause-
ated, and bradycardic. What is the next step in management?
a. Administer intramuscular epinephrine 1:10,000.
b. Administer 0.5 mg intravenous (IV) atropine sulfate.
c. Administer 0.4 mg IV naloxone.
d. Administer 25 g IV dextrose.
336 ● Study Questions
38. An 80-year-old man has had binocular diplopia, worse with upgaze, since he underwent
otherwise uncomplicated left-eye cataract surgery with retrobulbar anesthesia 6 months
ago. His pupils and eyelids are normal. The left eye has poor elevation with positive forced
duction testing. What is the most likely cause of his diplopia?
a. ocular neuromyotonia
b. anesthetic myotoxicity
c. myasthenia gravis
d. decompensated phoria
Answers
1. b. Although the rates of uninsured Americans have decreased under the Affordable Care
Act, approximately 10% of people in the United States are uninsured. Vulnerable popula-
tions, such as underrepresented racial and ethnic groups and low-income individuals, are
at highest risk of being uninsured.
2. a. Ordinal variables have 2 or more categories that can be ordered or ranked. For example,
a variable with response data ranging from “strongly agree” to “strongly disagree” would be
considered ordinal; a Likert scale is a well-known example of this type of variable. Many visual
function questionnaires use ordinal data. Categorical variables are those that have discrete
categories or levels. Examples include blood type, eye color, and educational level. Dichoto-
mous variables are categorical variables with 2 levels, such as yes/no or high/low. Continuous
variables are measured numerically and have an infinite number of possible values.
3. d. Teprotumumab, approved by the US Food and Drug Administration in 2020, has been
shown to slow the progression of ophthalmopathy in patients with moderate to severe
thyroid eye disease (TED). It inhibits insulin-like growth f actor I receptor and represents
a new therapeutic strategy for treating the underlying autoimmune pathogenesis of TED.
4. b. The most common cancer in multiple endocrine neoplasia syndrome (MEN) 2 (for-
merly MEN2A) and MEN3 (formerly MEN2B) is medullary thyroid cancer, which occurs
in 90%–100% of patients and is the main cause of morbidity.
5. a. The 2017 American College of Cardiology/American Heart Association guidelines for
the treatment of hypertension list thiazide-type diuretics and calcium channel blockers
(CCBs) as the drug classes for the initial drug choice in Black patients. For patients who
are not Black, the initial drug choice may be selected from 4 drug classes, on the basis of
clinical setting and comorbidities: thiazide-type diuretics, CCBs, angiotensin-converting
enzyme inhibitors, and angiotensin receptor blockers.
6. b. Evolocumab and alirocumab are injectable monoclonal antibodies to proprotein con-
vertase subtilisin kexin 9 (PCSK9). T hese PCSK9 inhibitors are used as adjunctive therapy
when maximal statin therapy fails to reduce low-density-lipoprotein cholesterol (LDL-C)
to the desired level in high-risk patients. For nearly all patients whose LDL-C goals can-
not be met by therapeutic lifestyle changes alone, 3-hydroxy-3-methylglutaryl coenzyme
A (HMG-CoA) reductase inhibitors, or statins, are the initial choice for medical therapy.
Individuals with LDL-C levels of ≤70 mg/dL do not need therapy.
7. b. The most effective way to manage congestive heart failure with a reduced ejection frac-
tion (HFrEF) is to reduce afterload to decrease the burden on the left ventricle. Regardless
of the baseline values, reducing blood pressure (while maintaining adequate tissue perfu-
sion) is the mainstay of treatment. Heart failure with preserved ejection fraction (HFpEF)
can be improved by reducing preload. If medical therapy fails in patients with HFpEF,
percutaneous coronary intervention (PCI) or coronary artery bypass grafting (CABG) is
considered. Statins are beneficial in HFpEF but not in HFrEF.
8. a. Implantable cardioverter-defibrillators (ICDs) are the preferred first-line treatment for
patients who have survived a cardiac arrest or an episode of hemodynamically unstable
ventricular tachycardia. ICDs are associated with a 20%–30% relative reduction in the
risk of death. Most patients with ICDs require concomitant antiarrhythmic therapy with
337
338 ● Answers
amiodarone or a β-blocker. PCI and CABG are revascularization procedures that can be
used for cardiac ischemia. Digoxin is an inotropic agent used in the treatment of HFrEF.
9. c. Multiple studies have shown that administering the fibrinolytic agent recombinant tis-
sue plasminogen activator (rtPA) within 3 hours of acute ischemic stroke results in lower
morbidity. Imaging with noncontrast computed tomography (CT) of the brain is critical
in evaluating for the presence of intracranial hemorrhage, which would be a contraindica-
tion to rtPA use. Magnetic resonance imaging, while more sensitive than noncontrast CT
in detecting an evolving stroke within hours of its onset, is too time intensive given the
limited window during which rtPA can be administered. Although cerebral angiography
can provide detailed information about vascular abnormalities, it is not indicated in the
acute care setting and would not reveal either stroke or hemorrhage. Transcranial Doppler
ultrasonography may reveal stenosis of the intracranial arteries, but it is not helpful for
guiding initial management.
10. a. Aspirin is the only antiplatelet agent that is effective in the early treatment of ischemic
stroke. Two large clinical t rials showed a benefit of treatment with aspirin over placebo in
short-term mortality and recurrent stroke risk when aspirin is initiated within 48 hours
of onset of an ischemic stroke. Early use of combination antiplatelet agents such as aspirin
with clopidogrel may be beneficial, but the evidence is not consistent. Heparin and related
agents such as enoxaparin are associated with higher mortality and worse outcomes in
patients with cardioembolic or noncardioembolic stroke.
11. b. Ocular conditions associated with obstructive sleep apnea include floppy eyelid syn-
drome, dry eye, glaucoma, papilledema, central serous chorioretinopathy, and nonarter-
itic anterior ischemic optic neuropathy.
12. a. Myeloid stem cells are the precursors of red blood cells, granulocytes, monocytes, and
platelets. Lymphoid stem cells become T cells, B cells, natural killer cells, and innate lym-
phoid cells. Mesenchymal stem cells are multipotent stem cells that are present in various tis-
sues, including bone marrow, and have the ability to differentiate into a variety of cells (bone,
cartilage, fat). Adult cells such as skin fibroblasts can be induced to produce pluripotent stem
cells by using transcription factors. Induced pluripotent stem cells hold great promise in the
field of regenerative medicine and can be used to create every type of cell.
13. c. If iron is not incorporated properly into the heme molecule, hemoglobin synthesis
is reduced; this condition is called sideroblastic anemia. Treatment includes pyridoxine
(vitamin B6), iron-chelating agents such as deferoxamine, and hematopoietic stem cell
transplantation (also called bone marrow transplantation).
14. a. Most patients with activated protein C (APC) resistance harbor a single specific point
mutation in the factor V gene, called factor V Leiden, which renders both forms of factor
V (active and inactive) insensitive to APC proteolysis. This mutation occurs with re-
markable frequency (3%–7%) in healthy White populations but appears to be far less
prevalent or even absent in certain Black and Asian populations. Factor V Leiden muta-
tion and subsequent APC resistance are implicated in central and branch retinal vein
occlusions.
15. a. A number of inherited and acquired disorders of blood vessels and their supporting con-
nective tissues result in pathologic bleeding. Many of these disorders also have ocular find-
ings. Hereditary hemorrhagic telangiectasia (Rendu-Osler-Weber disease) is an autosomal
dominant condition characterized by localized dilatation of capillaries and telangiectasias
Answers ● 339
(actually small arteriovenous malformations) of the skin and mucous membranes. The tel-
angiectasias are often located on the lips and tongue and increase in size and number over a
period of decades.
Ehlers-Danlos syndrome is characterized by hyperplastic fragile skin and hyperextensi-
ble joints. In patients with osteogenesis imperfecta, bone fractures and otosclerosis (lead-
ing to deafness) are common. Pseudoxanthoma elasticum is rare and is often complicated
by gastrointestinal hemorrhage.
16. c. This patient has rheumatoid arthritis (RA). Ocular involvement may include dry eye;
scleritis; episcleritis; and corneal inflammation, melting, and infection. Anti–citrullinated
peptide antibody testing, specifically anti–cyclic citrullinated peptide and anti–mutated
citrullinated vimentin, is as sensitive as rheumatoid factor (RF) but has a higher specificity
for RA (>95%).
17. a. In patients with juvenile idiopathic arthritis, ocular involvement is most common in
those with the oligoarticular form and least common in those with the systemic form.
Patients with oligoarticular or polyarticular (especially RF-negative) disease should be pe-
riodically screened for ocular involvement because it is often asymptomatic. Furthermore,
antinuclear antibody positivity in patients with oligoarticular, polyarticular, or psoriatic
arthritis is associated with an increased risk of developing uveitis.
18. a. The classic triad of reactive arthritis consists of arthritis, urethritis, and conjunctivitis.
Human leukocyte antigen–B27 and genetics appear to be involved in susceptibility to de-
veloping reactive arthritis a fter an infection. The arthritic symptoms typically have their
onset from days to weeks after the antecedent infection. Joint involvement is typically
asymmetric and episodic, primarily affecting the knees and ankles.
Up to 40% of patients with e ither ankylosing spondylitis or reactive arthritis develop the
typical manifestations of acute nongranulomatous iridocyclitis, which can be recurrent and
bilateral. Ocular involvement tends not to correlate with the activity of the joint disease.
Culture-negative bilateral conjunctivitis, which is typically self-limited, is also a common
manifestation of reactive arthritis.
19. c. Labial salivary gland biopsy can be helpful for confirming a diagnosis of Sjögren syn-
drome in patients lacking evidence of autoimmunity (absent or low titers of autoantibod-
ies or the absence of an associated autoimmune disorder). An abnormal Schirmer test
result and abnormal ocular surface staining are less specific. Buccal mucosal biopsy is
indicated when oral mucosal lesions are present.
20. d. Updated recommendations state that daily hydroxychloroquine dosing should not ex-
ceed 5.0 mg/kg actual body weight. Actual body weight is thought to be more predictive
than ideal body weight in assessing risk of retinopathy from hydroxychloroquine.
21. c. In cases of suspected elder neglect or abuse, it is helpful to obtain the patient’s his-
tory with the caregiver out of the room. Directed questions such as “Has anyone at home
tried to harm you?” may yield information. Any suspected case of neglect or abuse should
prompt a written report. Documentation of any suspicious injuries is mandatory. Require-
ments for reporting elder abuse vary from state to state, and many areas have abuse hot-
lines for reporting maltreatment. The physician should be aware of local services for adult
protection, community social services, and law enforcement agencies.
22. a. Bilateral pinpoint conjunctival hemorrhages can occur in patients with bulimia. Other
ocular findings in patients with eating disorders include lagophthalmos from orbital fat
340 ● Answers
31. c. All staphylococcal isolates w ere susceptible to vancomycin. There was a high prevalence
of methicillin resistance among staphylococcal isolates and a high probability of concur-
rent resistance to fluoroquinolones. Overall antibiotic resistance of ocular isolates did not
increase during the 10-year study period.
32. c. With this patient’s history of fever and recent antibiotic use, the diagnosis is most likely
pseudomembranous enterocolitis (C diff). The causative organism, Clostridioides (for-
merly Clostridium) difficile, is an anaerobic gram-positive bacillus that is part of the nor-
mal gastrointestinal flora. Fever and diarrhea develop 1–14 days after the start of antibiotic
therapy. The diarrhea occasionally becomes bloody. The most frequently implicated anti-
biotics include clindamycin, ampicillin, chloramphenicol, tetracycline, erythromycin, and
the cephalosporins. The bacterium elaborates cytopathic toxins (A and B), which can be
detected by enzyme-linked immunosorbent assay (ELISA). A polymerase chain reaction
assay is also sometimes used in the diagnosis. Initial treatment includes discontinuing the
causative antibiotic and prescribing oral vancomycin.
33. d. Prior to the 1990s, a common etiology of orbital cellulitis in c hildren younger than
9 years was Haemophilus influenzae infection. However, since H influenzae type b (Hib)
vaccination in infants became widespread, the incidence of H influenzae infection has
decreased dramatically. Orbital cellulitis does not typically develop from otitis media.
34. b. The secondary stage is heralded by a truncal rash that may spread over the entire body.
Fever, malaise, lymphadenopathy, and hair loss may occur. Initial inoculation occurs
through intact mucous membranes or abraded skin and, within 6 weeks, results in a small,
ulcerated, painless papule called a chancre (primary stage). Latent syphilis is characterized
by positive serologic test results in a patient without clinical signs. Tertiary disease is char-
acterized by destructive granulomatous lesions with a typical endarteritis that can affect
the skin, bones, joints, oral and nasal cavities, parenchymal organs, cardiovascular system,
eyes, meninges, and central nervous system.
35. a. Ethambutol is associated with optic neuropathy in a small percentage of patients. In
patients with suspected ethambutol-induced optic neuropathy, formal visual acuity test-
ing, central visual field testing, fundus examination, and optical coherence tomography of
the peripapillary retinal nerve fiber layer, as well as possibly electrophysiologic testing (eg,
visual evoked potential, multifocal electroretinography), should be considered.
36. b. Preoperative testing, including electrocardiography and routine blood testing, should
be performed only when indicated; that is, the tests would be done even if the patient was
not planning surgery. Multiple clinical trials have failed to show a difference in periopera-
tive adverse events in healthy patients undergoing elective eye surgery, whether or not
preoperative testing was obtained. However, a complete history and physical examination
are required for all patients as part of the preoperative assessment.
37. b. The use of intravenous edrophonium chloride in the diagnosis of myasthenia gravis can
have toxic adverse effects. The signs and symptoms result from cholinergic stimulation
and may include nausea, vomiting, diarrhea, sweating, increased bronchial and salivary
secretions, muscle fasciculations and weakness, and bradycardia. Whenever the test is to
be performed, a syringe containing 0.5 mg of atropine sulfate—an anticholinergic drug
that reverses possible adverse effects such as heart arrhythmias and respiratory arrest—
must be immediately available. The drug should be administered if the aforementioned
signs and symptoms occur; this dose may be repeated every 3–10 minutes if necessary.
342 ● Answers
38. b. This patient has restrictive strabismus, as demonstrated by forced duction testing.
Causes to be considered include thyroid eye disease, traumatic inferior rectus entrapment,
and anesthetic myotoxicity. While the first 2 entities are possible causes, the history is
more consistent with direct toxic effects of anesthetic on extraocular muscles, which can
occur from inadvertent injection of local anesthetic into 1 of the rectus muscles (typically
the inferior rectus). Initially, the patient may have no symptoms or a transient hypertropia
of the affected eye due to muscle weakness. Over time, contraction or fibrosis develops,
causing a restrictive strabismus.
Patients with decompensated phoria should have full ductions. Patients with myasthe-
nia gravis would not have positive forced duction tests. Ocular neuromyotonia is a rare
cause of transient diplopia, usually occurring months or years a fter skull base irradiation,
and is not consistent with the clinical picture.
Study Questions
Please note that these questions are not part of your CME reporting process. They are pro-
vided here for your own educational use and for identification of any professional practice
gaps. The required CME posttest is available online (see “Requesting Continuing Medical Ed-
ucation Credit”). Following the questions are answers with discussions. Although a concerted
effort has been made to avoid ambiguity and redundancy in these questions, the authors
recognize that differences of opinion may occur regarding the “best” answer. The discussions
are provided to demonstrate the rationale used to derive the answer. They may also be help-
ful in confirming that your approach to the problem was correct or, if necessary, in fixing the
principle in your memory. The Section 2 faculty thanks the Resident Self-Assessment Com-
mittee for developing these self-assessment questions and the discussions that follow.
1. Loss of what structure due to increasing age can lead to conjunctivochalasis?
a. levator aponeurosis
b. Müller muscle
c. tarsus
d. Tenon capsule
3. In what condition does a pathogenic variant in the fibrillin-1 gene cause lens subluxation
due to zonular weakness?
a. homocystinuria
b. Marfan syndrome
c. pseudoexfoliation syndrome
d. recurrent uveitis
517
518 ● Study Questions
5. What is the normal density of corneal endothelial cells centrally in young adults?
a. 8000 cells/mm2
b. 3000 cells/mm2
c. 1000 cells/mm2
d. 500 cells/mm2
10. What is the cellular function of the gene product that is defective in cases of retinoblas-
toma?
a. contribution to cellular structure
b. production of energy
c. regulation of the cell cycle
d. transportation of cell wall proteins
Study Questions ● 519
11. Patients with xeroderma pigmentosum are at increased risk for which ocular cancers?
a. choroidal melanoma and retinoblastoma
b. squamous cell carcinoma and choroidal melanoma
c. squamous cell carcinoma and ocular surface melanoma
d. squamous cell carcinoma and retinoblastoma
12. An ophthalmologist is consulted to provide a complete eye evaluation for a patient. The
patient, who has very light hair and skin color, has reported experiencing mild chronic
photosensitivity and blurring in both eyes but no other symptoms. Recently, the patient
had a dental procedure that resulted in excessive bleeding. His sister has a similar history
and symptoms. What is the most likely diagnosis?
a. achromatopsia with myopia
b. Hermansky-Pudlak syndrome
c. Niemann-Pick disease
d. oculocutaneous albinism
13. What 2 medications have been approved by the US Food and Drug Administration (FDA)
for treatment of the inflammatory component of dry eye syndrome?
a. prednisolone acetate and doxycycline
b. topical cyclosporine A emulsion and doxycycline
c. topical cyclosporine A emulsion and lifitegrast
d. topical cyclosporine A emulsion and prednisolone acetate
14. What is the maximum amount of fluid an eye can hold prior to tearing?
a. 5–10 μL
b. 15–20 μL
c. 25–30 μL
d. 35–40 μL
15. What type of collagen production is associated uniquely with corneal stromal wound
healing?
a. type I
b. type III
c. type V
d. type VII
16. Soft contact lenses with low oxygen permeability can cause corneal edema via the accu-
mulation of what substance?
a. aldehyde dehydrogenase
b. carbon dioxide
c. lactic acid
d. proteoglycans
520 ● Study Questions
17. The disruption of what corneal layer during laser subepithelial keratomileusis (LASEK) is
thought to be the cause of increased risk for visual complications postoperatively due to
central corneal haze?
a. Bowman layer
b. endothelium
c. epithelium
d. stroma
18. In the healthy human eye, which of the following components of the aqueous humor is
always higher in concentration in the aqueous than it is in plasma?
a. calcium
b. glucose
c. lactate
d. urea
19. In the Goldmann equation, IOP = (F − U) / C + EVP, what variable represents the rate of
aqueous humor drainage via the pressure-insensitive pathway?
a. C
b. EVP
c. F
d. U
20. Alterations in glucose metabolism can result in cataract formation. Under normal physi-
ologic conditions, what is the primary pathway by which glucose is converted into ad-
enosine triphosphate (ATP) in the human lens?
a. aerobic metabolism
b. glycolysis
c. pentose phosphate pathway (also called hexose monophosphate shunt)
d. polyol pathway (also called sorbitol pathway)
23. What role does the vitreous exert on oxygen tension in the vitreous cavity?
a. Oxygen tension is unrelated to the presence of vitreous.
b. Oxygen tension is lower in the presence of vitreous.
c. Oxygen tension is higher in the presence of vitreous.
d. Eyes that have undergone vitrectomy contain decreased amounts of free radicals.
24. What condition is characterized by a pathogenic variant in a gene encoding type II col-
lagen and leads to premature liquefaction of vitreous with peripheral condensation, which
may induce retinal detachment?
a. familial exudative vitreoretinopathy
b. Marfan syndrome
c. retinitis pigmentosa
d. Stickler syndrome
28. What class of retinal cells functions as the resident macrophage and is activated under
stress?
a. amacrine cells
b. macroglia
c. microglia
d. pericytes
522 ● Study Questions
29. In what clinical presentation is genetic testing for a pathogenic variant in the gene RPE65
indicated?
a. bilateral central scotomas with optic disc pallor
b. bilateral central scotomas with vitelliform macular lesions
c. bilateral ring scotomas with bone spicule retinal lesions
d. bilateral superior arcuate scotomas with optic disc cupping
30. The retinal pigment epithelium is the first developmental site of melanogenesis in the body.
In which of the following ocular processes has ocular melanin been shown to participate?
a. pathogenesis of retinitis pigmentosa
b. retinal adhesion
c. retinal development and neuronal migration
d. vitamin A metabolism
31. The adverse effects of reactive oxygen species (ROS) have been proposed as causal factors
in which ocular conditions?
a. cataract
b. conjunctivitis
c. strabismus
d. posterior capsule opacification
32. What characteristic of the retina makes it more vulnerable to damage from lipid peroxida-
tion?
a. high number of mitochondria in the rod inner segments
b. high levels of saturated fatty acids in the rod outer segments
c. low light exposure at night
d. poor oxygen supply through the choroid
33. Assuming a ratio of 20 drops per milliliter (mL), how many milligrams (mg) of atropine
are found in 1 drop of a 1% solution?
a. 1 mg
b. 0.5 mg
c. 0.1 mg
d. 0.05 mg
37. An orbital floor fracture is suspected in a 23-year-old patient examined in the emergency
department following trauma. What imaging modality is best suited to evaluate the orbit
in this case?
a. computed tomography
b. magnetic resonance imaging
c. optical coherence tomography
d. ultrasound biomicroscopy
38. What are the standard probe positions used in ocular B-scan ultrasonography?
a. axial, sagittal, and radial
b. axial, tangential, and radial
c. axial, transverse, and longitudinal
d. coronal, transverse, and sagittal
39. What term describes a deep learning training set labeled with disease state?
a. algorithm
b. ground truth
c. spatial coherence
d. temporal coherence
40. What represents the digital data standard for diagnostic images and measurements?
a. Current Procedural Terminology
b. Digital Imaging and Communications in Medicine
c. Logical Observations, Identifiers, Names, and Codes
d. Picture Archiving and Communications System
Answers
1. d. Conjunctivochalasis is characterized by redundant folds of conjunctiva between the
globe and the eyelid margin. Because the bulbar conjunctiva fuses with the Tenon cap-
sule as it inserts at the limbus, conjunctivochalasis results from loss of Tenon capsule
caused by increasing age. Dehiscence of the levator aponeurosis with increasing age can
lead to ptosis but is not the cause of conjunctivochalasis. Although the palpebral con-
junctiva adheres firmly to the tarsus, and the forniceal conjunctiva of the upper eyelid is
enmeshed in fibrous elements of Müller muscle, loss of these structures is not the cause of
conjunctivochalasis.
2. c. Each eye lies within the bony orbit, the volume of which is slightly less than 30 mL. The
orbital entrance averages approximately 35 mm in height and 45 mm in width at its wid-
est point. The depth of the orbit varies from 40 mm to 45 mm. Race and sex affect each of
these measurements.
3. b. Each zonular fiber suspending the lens is made up of multiple filaments of fibrillin. In
patients with Marfan syndrome, a pathogenic variant in the fibrillin-1 gene leads to weak-
ness of the zonular fibers and lens subluxation. Homocystinuria may cause lens sublux-
ation, but the pathologic variant in this condition is in a gene that regulates cystathionine
β-synthase production. Although pseudoexfoliation syndrome and uveitis also may cause
lens subluxation, occurrence of these conditions has not been associated with the fibril-
lin-1 gene.
4. c. The long and short posterior ciliary arteries, in addition to the perforating anterior cili-
ary arteries, perfuse the choroid. The choroid is drained by the vortex veins. The choroid
is approximately 0.25 mm thick and comprises 3 layers: the choriocapillaris, the middle
layer of small vessels, and the outer layer of large vessels. The choriocapillaris perfuses the
outer retinal layers. The central retinal artery perfuses the nerve fibers that form the optic
nerve. The major arterial circle perfuses the iris and ciliary body.
5. b. The corneal endothelium is composed of a single layer of hexagonal cells derived from
the neural crest. Therefore, the corneal endothelium is of neuroectodermal origin. In young
adult eyes, approximately 500,000 cells are present, at a density of about 3000/mm2 centrally
and up to 8000/mm2 peripherally. Mitosis of the endothelium is limited in humans, and the
overall number of endothelial cells decreases with age. The principal function of the corneal
endothelium is to regulate transmission of nutrients and solutes from the aqueous humor to
the stroma. The active transport of ions by these cells leads to the transfer of water from the
corneal stroma and the maintenance of stromal deturgescence and transparency. Endothe-
lial cell dysfunction and loss—through surgical injury, inflammation, or disease (eg, Fuchs
endothelial corneal dystrophy)—may cause endothelial decompensation, stromal edema,
and vision loss. Because endothelial mitosis is limited in humans, destruction of cells causes
cell density to decrease and residual cells to spread and enlarge.
6. b. Cranial nerve (CN) VI, the abducens nerve, leaves the brainstem at the pontomedul-
lary junction, follows a vertical course along the ventral face of the pons, and continues in
the subarachnoid space along the surface of the clivus. Nasopharyngeal carcinomas that
extend through the skull base can impact CN VI along the surface of the clivus, prior to
its entry into the cavernous sinus via the Dorello canal. Extension into the cavernous sinus
can affect CN III, CN IV, CN V, and CN VI. Similarly, sympathetic fibers traveling along
525
526 ● Answers
the carotid artery would only be affected if there is extension into the cavernous sinus.
Like CN VI, CN VII also exits the brainstem at the pontomedullary junction. However,
CN VII enters the internal auditory meatus and is not present at the skull base.
7. a. CN VII (the facial nerve) is a complex, mixed sensory and motor nerve. Its motor root
contains special visceral efferent fibers that innervate the muscles of facial expression. Its
sensory root conveys the sense of taste from the anterior two-thirds of the tongue and
sensation from the external auditory meatus and the retroauricular skin. It also supplies
preganglionic parasympathetic innervation by way of the sphenopalatine and submandib-
ular ganglia to the lacrimal, submaxillary, and sublingual glands. CN V3 (the mandibular
division of the trigeminal nerve) provides sensation for the preauricular skin. Pregan-
glionic sympathetic fibers (second-order neurons) run in the sympathetic chain, where
they eventually synapse at the superior cervical ganglion. Taste and sensation from the
posterior one-third of the tongue are provided by CN IX (the glossopharyngeal nerve).
8. c. Congenital cranial dysinnervation disorders are thought to result from delayed muscle
innervation during development, which can provide a window for inappropriate innerva-
tion (by a different nerve) to develop. In Marcus Gunn jaw-winking syndrome, the levator
muscle is innervated by CN V (the trigeminal nerve), consistent with the eyelid eleva-
tion that occurs with engagement of the pterygoid muscle, thus giving the appearance of
“winking” when chewing. Similarly, in Duane syndrome, CN III (the oculomotor nerve)
inappropriately innervates the lateral rectus muscle. Alternatively, delayed muscle inner-
vation may lead to premature differentiation of the mesenchyme into connective tissue
(ie, fibrosis), as in congenital fibrosis of the extraocular muscles and in Möbius syndrome.
9. c. Transcription factors are proteins that bind to specific DNA sequences and control the
flow of genetic information at the level of transcription of DNA to messenger RNA. Ho-
meobox genes, like PAX, represent highly conserved transcription factors dedicated to
embryologic development. Many ophthalmic diseases are caused by pathogenic variants
of homeobox genes, including Waardenburg syndrome with dystopia canthorum, which
is caused by a PAX3 variant. PAX2 variants may lead to optic nerve coloboma and renal
hypoplasia. PAX6 variants lead to aniridia. The BAX gene promotes apoptosis in the pa-
tients with DNA damage (thus preventing tumor development).
10. c. The retinoblastoma protein regulates the cell cycle at the G1 checkpoint and functions
as a tumor suppressor. Pathogenic variants in the retinoblastoma gene (RB1) are found not
only in other related tumors, such as osteosarcoma, but also in unrelated tumors such as
breast cancer and lung cancer. The hereditary pattern of familial retinoblastoma is autoso-
mal dominant. However, at the cellular level, it is autosomal recessive; a pathogenic variant
on both chromosomes in a given cell is required in order for tumorigenesis to occur.
11. c. Xeroderma pigmentosum is a severe condition caused by a pathogenic variation that af-
fects DNA repair enzymes. Patients with this disorder have diffuse pigmented anomalies
on their sun-exposed skin and are at high risk for squamous cell carcinoma and mela-
noma of the ocular surface. The risks for choroidal melanoma and retinoblastoma are not
increased in these individuals.
12. b. All the listed diseases may be concentrated in various racial/ethnic groups. Hermansky-
Pudlak syndrome is more common in certain Puerto Rican communities, but has also
been found in India, Japan, the United Kingdom, and Western Europe. Hermansky-
Pudlak syndrome is an autosomal recessively inherited disorder characterized by ocu-
locutaneous albinism and platelet abnormalities that lead to increased bleeding time and
Answers ● 527
abnormal platelet aggregation (easy bruising and higher bleeding tendency, but normal
platelet counts). Several types of achromatopsia (complete color blindness) with myopia
are common on the South Pacific island of Pingelap. Niemann-Pick disease is a rare in-
herited metabolic disorder of fat metabolism with different subtypes (varying severity and
presentations); type A is associated with increased incidence among the Ashkenazi Jewish
population. Oculocutaneous albinism occurs at a higher rate in the Kuna Indian popula-
tion in Panama.
13. c. Topical cyclosporine A emulsion and lifitegrast have both been approved by the US
Food and Drug Administration (FDA) to treat the inflammatory component of dry eye
syndrome. Prednisolone acetate may be used in the treatment of dry eye syndrome but is
not US FDA approved for this use. Although doxycycline has ocular anti-inflammatory
effects, it is not US FDA approved for the treatment of the inflammatory component of
dry eye syndrome.
14. c. The maximum amount of fluid an eye can hold prior to tearing is 25–30 microliters (μL).
The volume of the average eyedrop is 45 μL; therefore, it is not helpful to use more than 1
drop on an eye at a time.
15. b. Type III collagen production is associated uniquely with stromal wound healing. Type I is
the major collagen component of the corneal stroma; it constitutes approximately 70% of the
total stromal dry weight. Immunohistochemical and biochemical studies have shown that
normal adult corneal stroma also contains collagen types V, VI, VII, XII, and XIV.
16. c. Under anaerobic (low-oxygen) conditions, glucose, the main metabolic substrate for the
cornea, is converted to pyruvic acid and then to lactic acid. Lactic acid accumulation in the
stroma increases the osmotic load, which draws water into the cornea and causes corneal
edema. Aldehyde dehydrogenase, carbon dioxide, and proteoglycans do not accumulate in
the cornea in hypoxic conditions. Aldehyde dehydrogenase is a soluble protein found in the
cornea that absorbs ultraviolet-B (UVB) radiation. Carbon dioxide is a product of glucose
metabolism via glycolysis under aerobic conditions. Proteoglycans confer hydrophilic prop-
erties to the corneal stroma and interact with collagen fibrils to provide corneal clarity.
17. a. It is thought that Bowman layer, by virtue of its acellularity and packing distribution,
serves to prevent exposure of stromal corneal keratocytes to growth factors secreted by
corneal epithelial cells. This effect is notable because, during photorefractive keratectomy
(PRK) or laser subepithelial keratomileusis (LASEK), Bowman layer is removed, along
with the anterior corneal stromal tissue. In these procedures, corneal haze is a potentially
significant postoperative visual complication, presumably because the stromal keratocytes
are exposed to regenerating epithelial growth factors and take on fibroblastic behaviors. In
contrast, during laser in situ keratomileusis (LASIK), Bowman layer is transected but still
retained, and thus central corneal haze is extremely rare.
18. c. The aqueous humor is the major nutrient source for the avascular lens and cornea and
provides a route for the removal of waste products. Ocular fluids are separated from blood
by barriers formed by tight junctions between epithelial and endothelial cells. These junc-
tions make up the blood–aqueous barrier. Aqueous enters the posterior chamber from
the ciliary processes by means of active and passive physiologic mechanisms, resulting in
specific concentrations of various ions to maintain a specific composition of the fluid. In
order of highest to lowest concentration, the components of aqueous humor are:
• Na+ (163 mmol/kg H2O)
• Cl− (134 mmol/kg H2O)
528 ● Answers
which may induce retinal detachment. Wagner syndrome is also characterized by an opti-
cally empty vitreous but is due to a pathogenic variant in the VCAN (versican) gene, which
participates in formation of the vitreous gel. However, patients with Wagner syndrome do
not have increased risk of retinal detachment as do those with Stickler syndrome. Familial
exudative vitreoretinopathy is an inherited disease (resulting from defects in Wnt signaling)
that leads to abnormal retinal angiogenesis and incomplete vascularization of the peripheral
retina. Marfan syndrome is an inherited disease caused by a pathogenic variant in the gene
that encodes fibrillin-1. Its ocular manifestations include ectopia lentis, high myopia, scleral
thinning, and retinal detachment. Retinitis pigmentosa refers to a group of inherited diseases
characterized by diffuse, progressive dysfunction of predominantly rod photoreceptors. In
the United States, 10% of retinitis pigmentosa cases are due to a pathogenic variant in the
rhodopsin (RHO) gene.
25. a. After the collagen fibers and other insoluble elements present in the vitreous gel are re-
moved by filtration or centrifugation, many proteins remain in solution. Serum albumin is
the major soluble vitreous protein, followed by transferrin. Other proteins include neutro-
phil elastase inhibitor and tissue plasminogen activator. The concentration of serum pro-
teins in the vitreous gel depends on the integrity of the retinal vasculature and the degree of
intraocular inflammation.
26. a. Horizontal cells are antagonistic (inhibitory) interneurons that provide negative feed-
back to photoreceptors. Their dendrites synapse with cones, receiving glutamate from
the cones and releasing γ-aminobutyric acid (GABA) back onto them, thus providing
negative feedback. When light causes cone hyperpolarization and cessation of gluta-
mate release, then neighboring horizontal cells are also hyperpolarized and stop releas-
ing GABA. When GABA release onto the cone ceases, the cone then depolarizes. This
feedback inhibition allows for visualization of low-contrast details against background
luminance.
Bipolar cells are neurons that transmit neural signals from photoreceptors to the inner
retina. Glial cells, which provide neuronal support, include Müller cells, macroglia (mainly
astrocytes), and microglia. Photoreceptors are neurons and include rods and cones.
27. a. 11-cis-retinal is a vitamin A derivative. Light activation of rhodopsin starts a series of
reactions that lead to hyperpolarization of the photoreceptor’s membrane potential. Once
rhodopsin absorbs a quantum of light, the 11-cis double bond of retinal (vitamin A) is re-
configured (creating all-trans-retinal, also called all-trans-retinaldehyde), and the opsin
molecule undergoes a series of rapid configurational changes to an activated state known as
metarhodopsin II, initiating a signal transduction cascade. Vitamin B does not participate in
the phototransduction cascade. However, members of the vitamin B family do play support-
ive roles in biochemical processes in the eye. For example, vitamin B12 deficiency can lead to
optic atrophy. Vitamins C and E play antioxidant roles in the retina, but do not participate in
the light response of the retina.
28. c. Microglia are a class of retinal cells that are related to tissue macrophages and are acti-
vated when retinal homeostasis is disturbed. Amacrine cells are inhibitory interneurons
that mediate interactions among bipolar and ganglion cells. Macroglia provide physical
support to neuronal and vascular cells, regulate the ionic and chemical composition of
the extracellular milieu, participate in the blood–retina barrier, form the myelin sheath of
the optic nerve, guide neuronal migration during development, and exchange metabolites
with neurons. Pericytes are modified smooth muscle cells that surround the endothelial
cells and play a role in autoregulation of retinal blood vessels.
530 ● Answers
29. c. The RPE65 gene encodes the enzyme RPE65 isomerohydrolase, which converts all-
trans-retinyl ester to 11-cis-retinal. A pathogenic variant in this gene may result in Leber
congenital amaurosis or retinitis pigmentosa, which present clinically as bilateral ring sco-
tomas with bone spicule retinal lesions. The US FDA has approved gene therapy to treat
diseases caused by pathogenic variants in RPE65; thus, genetic testing is indicated for
patients with this clinical presentation. None of the other conditions have been attributed
to a pathogenic variant in RPE65. Answer choice (a) describes a bilateral optic neuropa-
thy, such as dominant optic atrophy. Answer choice (b) describes the clinical presentation
of Best disease, which is caused by heterozygous variants of the bestrophin gene. Answer
choice (d) is most suggestive of glaucoma.
30. c. Melanin absorbs all wavelengths of light. Patients with oculocutaneous albinism have
foveal hypoplasia and more contralateral projections of the retinal ganglion cells, thought
to be due to reduced melanin levels resulting from defects in the tyrosinase gene. Addi-
tional functions of melanin include stabilization of free radicals and detoxification. Mela-
nin is not known to be involved in the pathogenesis of retinitis pigmentosa, retinal adhe-
sion, or vitamin A metabolism.
31. a. The adverse effects of reactive oxygen species (ROS) are thought to be causal factors in
many vision-threatening diseases, including cataract, age-related macular degeneration
(AMD), diabetic retinopathy, and glaucoma. Lipid peroxides are formed when ROS react
with unsaturated fatty acids. Lipid peroxidation causes not only direct damage to the cell
membrane but also secondary damage to cells through its breakdown products. Lipid per-
oxides are unstable, and they break down to form many aldehydes, such as malondialde-
hyde and 4-hydroxyalkenals. These aldehydes can quickly react with proteins, inhibiting
the proteins’ normal functions. Both the lens and the retina are susceptible to such oxidative
damage. Although repair and regeneration mechanisms are active in the lens epithelium and
superficial cortex, no such mechanisms exist in the deep cortex and the nucleus, where any
damage to lens proteins and membrane lipids is irreversible. One result of this damage can
be crosslinking and insolubilization of proteins, leading to loss of transparency. Conjuncti-
vitis, strabismus, and posterior capsule opacification are not associated with ROS damage.
32. a. Several distinctive characteristics make the retina vulnerable to damage from lipid
peroxidation:
• Rod inner segments are rich in mitochondria, which may leak activated oxygen
species.
• Rod outer segments possess high levels of polyunsaturated fatty acids (PUFAs),
making them susceptible to damage by oxygen. PUFAs are sensitive to peroxida-
tion in proportion to their number of double bonds.
• The outer retina contains many chromophores. Light exposure may trigger photo-
oxidative processes mediated by singlet oxygen.
• The abundant oxygen supply through the choroid and retinal vessels elevates the
risk of oxidative damage.
33. b. A 1% solution has 1 g/100 mL, or 1000 mg/100 mL, of active ingredient. Assuming
there are 20 drops/mL, 1 drop contains 0.05 mL of drug. Multiplying 1000 mg/100 mL ×
0.05 mL yields 0.5 mg per drop of atropine available for systemic absorption. Ophthalmol-
ogists need to be particularly aware of the risk of atropine-related systemic side effects; the
relatively high drop concentration and long half-life of the drug might lead to cardiac or
psychiatric complications depending on the frequency of use and the age/size of patient.
Answers ● 531
34. b. A 1:10,000 dilution has 1 g of drug in 10,000 mL (or 1000 mg/10,000 mL). This concen-
tration is equivalent to a 0.01% solution (0.01 g/100 mL, or 10 mg/100 mL). One milliliter
of the 1:10,000 dilution of epinephrine contains 0.1 mg of epinephrine. If the concentra-
tion of the solution increases to 1:1000, 0.1 mL of it contains the same amount of epineph-
rine as in 1 mL of the 1:10,000 solution.
35. a. In patients with uveitis, topical mydriatic and cycloplegic agents are used to prevent
posterior synechiae and to relieve pain due to ciliary spasm. Atropine sulfate has the lon-
gest duration of action (7–14 days), followed by scopolamine (4–7 days), homatropine (3
days), and cyclopentolate (2 days).
36. b. Cenegermin, a recombinant human nerve growth factor (NGF), is the first drug ap-
proved by the US FDA for the management of neurotrophic keratopathy. Both aflibercept
and ranibizumab are inhibitors of vascular endothelial growth factor (VEGF) and are used
in ophthalmology to treat various retinal diseases, including diabetic retinopathy, retinal
vein occlusion, and neovascular AMD. Teprotumumab is an inhibitor of the insulin-like
growth factor-1 receptor (IGF-1R) and is used in the treatment of active thyroid eye dis-
ease (TED).
37. a. Computed tomography (CT) is the modality of choice for assessing bony abnormali-
ties. Magnetic resonance imaging (MRI) is better suited for assessment of soft-tissue ab-
normalities than is CT. Optical coherence tomography and ultrasound biomicroscopy are
useful for assessing ocular structures but not for assessing the orbit.
38. c. The standard B-scan probe positions are axial, transverse, and longitudinal. Coronal,
sagittal, radial, and tangential are not standard B-scan probe positions.
39. b. Labeled data sets are considered the “ground truth” for machine learning algorithms to
identify patterns that predict those labels on new data. Similarities in data points over time
(eg, a single pixel in an image) are referred to as temporal coherence, whereas similarities
among groups of data points (eg, a group of pixels having similar coloring in an image
of a retinal hemorrhage) are referred to as spatial coherence. During machine learning,
algorithms learn to differentiate between relevant and irrelevant spatial and temporal co-
herences in the training images and, ultimately, to predict disease state in new data sets.
40. b. Digital Imaging and Communications in Medicine (DICOM) is the digital data stan-
dard for diagnostic images and measurements (eg, visual fields, biometry). A Picture Ar-
chiving and Communications System (PACS) is required for the storage and display of
patient images and can also help facilitate image analysis. Logical Observations, Identi-
fiers, Names, and Codes (LOINC) represents the digital data standard for lab results and
clinical observations/measurements, whereas Current Procedural Terminology (CPT)
represents the digital data standard for procedures and services.
Study Questions
Please note that these questions are not part of your CME reporting process. They are pro-
vided here for your own educational use and identification of any professional practice gaps.
The required CME posttest is available online (see “Requesting Continuing Medical Education
Credit”). Following the questions are answers with discussions. Although a concerted effort has
been made to avoid ambiguity and redundancy in these questions, the authors recognize that
differences of opinion may occur regarding the “best” answers. The discussions are provided
to demonstrate the rationale used to derive the answers. They may also be helpful in confirm-
ing that your approach to a problem was correct or, if necessary, in fixing a principle in your
memory. The Section 3 faculty thanks the Resident Self-Assessment Committee for developing
these self-assessment questions and the discussions that follow.
1. The simplest imaging system is a pinhole camera. What is a characteristic feature of this
device?
a. high magnification
b. superb depth of field
c. upright image formed at the image plane
d. rapid exposure times
2. As it is unsafe to view a partial eclipse of the sun directly, observers are often advised to use
a simple pinhole camera viewer to form an image of the eclipse. Of course, these pinhole
images are small and dim. A better alternative is to use a +1.00 lens to project an image of
the sun on the ground (see figure). How far should the lens be held from the ground to
obtain a sharp image?
a. 0.5 m
b. 1.0 m
c. 2.0 m
d. It doesn’t matter—any distance works well, just as in a pinhole camera.
361
362 ● Study Questions
3. Suppose a distant object is located to the left of a lens system, which focuses light from this
object at 1.0 m to the right of the lens system. If a +3.00 D lens is added adjacent to the lens
system, where is the new image located?
a. 25 cm to the right of the lenses
b. 50 cm to the right of the lenses
c. 1.5 m to the right of the lenses
d. 4.0 m to the right of the lenses
4. Where does the image of a distant object form for an eye with an uncorrected myopic
refractive error?
a. in front of the retina
b. on the retina
c. behind the retina
d. This depends on whether the patient is accommodating.
5. The power-cross description of a toric lens is +1.00 @ 25°, −2.00 @ 115°. What is the sphero-
cylindrical specification of this lens, expressed in minus cylinder form?
a. +1.00 −2.00 × 115°
() () () ()
7. What is the spherocylindrical specification of the most commonly used Jackson cross
cylinder (JCC)?
a. +0.25 −0.25 × 090
() () () ()
9. While surfing, a person spots a shark nearby in the water. Where is the shark, in reality,
compared with where the person sees its image from the surfboard?
a. closer to the person
b. further from the person
c. precisely where it appears
d. the same distance, but to the opposite side
10. What part of the conoid of Sturm is on the retina if only a spherical lens is used to give the
clearest image?
a. circle of least confusion
b. elliptical meridian
c. horizontal focal line
d. vertical focal line
11. When a beam of white light passes through a prism, it is dispersed into a spectrum of col-
ors. Which color of visible light is deflected the most?
a. red
b. yellow
c. green
d. violet
12. The radius of curvature of a convex passenger-side automobile rear-view mirror is typically
about 1.0 m. Where does such a mirror form an image of an object 10.0 m behind the car?
a. 0.91 m in front of the mirror (ie, on the opposite side of the mirror from the source
object)
b. 0.47 m in front of the mirror (ie, on the opposite side of the mirror from the source
object)
c. 0.47 m behind the mirror (ie, on the same side of the mirror as the source object)
d. 0.91 m behind the mirror (ie, on the same side of the mirror as the source object)
364 ● Study Questions
–1.00 D +1.50 D
Thin lens Thin lens
1.0 m 1.5 m
13. In the figure above, an object is 1.0 m to the left of a −1.00 D thin lens. The −1.00 D lens is,
in turn, 1.5 m to the left of a 1.50 D thin lens. What is the size of the final image compared
with the original object?,
a. one-fourth the size
b. one-half the size
c. same size
d. twice the size
14. A decrease in the size of the Airy disc image on the retina will result from what change, as
light rays are entering the eye?
a. smaller pupil
b. longer wavelength of light
c. decreased focal length of the eye
d. temporally shifted macula
15. What type of astigmatism is found in a patient with a refraction of −0.25 + 0.25 × 090?
a. compound astigmatism
b. simple myopic astigmatism
c. simple hyperopic astigmatism
d. mixed astigmatism
16. The radius of the bowl of the Goldmann perimeter is 333 mm. The diameter of the largest
available target (“V4”) is 9 mm. What is the diameter of the image of this target on the
retina?
a. 0.23 mm
b. 0.46 mm
c. 0.90 mm
d. 3.60 mm
Study Questions ● 365
17. LogMAR notation (base-10 logarithm of the minimum angle of resolution) is frequently
used to record visual acuity in clinical trials. What is the difference in logMAR scores
associated with a doubling of the minimum angle of resolution (such as the difference
between visual acuity of 20/40 and 20/80 or between 6/12 and 6/24)?
a. 0.01 logMAR
b. 0.20 logMAR
c. 0.30 logMAR
d. 0.50 logMAR
18. While refracting a patient, the ophthalmologist uses the duochrome test. The patient
states that the green letters are much clearer than the red letters. What does this signify
about the patient’s visual acuity?
a. The patient is overminused.
b. The patient is overplussed.
c. The patient is presbyopic.
d. The patient is anisometropic.
19. The clinician is given a small examination room and will refract a patient. If the visual
acuity screen is mounted on the wall 3 m from the patient, which step will ensure that the
refraction is correct?
a. Add −0.33 D to the final prescription.
b. Take no additional step. The refraction should be correct as measured.
c. Perform a duochrome test.
d. Refine the final sphere with a trial frame in a longer hallway (6 m).
20. Which bifocal type results in the least amount of image jump?
a. executive bifocal
b. flat-top bifocal
c. round-top bifocal
d. transition-zone bifocal
21. A patient wears glasses with the following prescription: OD −1.00 sphere and OS +2.00
sphere. When he reads, his eyes move 8 mm below the optic center. What is the induced
phoria (in prism diopters [∆])?
a. 0.8∆
b. 1.6∆
c. 2.4∆
d. 3.2∆
366 ● Study Questions
22. A patient sees well with contact lenses of −10.00 sph in both eyes. If glasses are made with
a vertex distance of 13 mm, which prescription is required to correct the refractive error?
a. −8.70 D
b. −10.90 D
c. −11.50 D
d. −12.80 D
23. A 46-year-old patient with myopia has blurred distance vision wearing his 5-year-old pair
of glasses, which have −4.00 D lenses for both eyes. He has no problem reading. The new
distance spectacle power is found to be −5.50 D in both eyes. He expresses an interest in
contact lenses. What is the likely effect on the patient’s vision of changing the distance
correction?
a. He will be able to see well, far and near, with single-vision spectacles, −5.50 D OU.
b. He will be able to see well, far and near, with contact lenses −4.00 D for each eye.
c. He will be able to see well, far and near, with contact lenses −5.50 D for each eye.
d. He will find it difficult to read with single-vision spectacles, −5.50 OU, and even more
difficult to read with contact lenses.
24. A patient has a refractive error of −4.50 D and a K measurement of 45.00 D (7.50 mm).
What is the residual refractive error if a plano rigid contact lens with a base curve of
7.67 mm (44.00 D) is placed on the eye?
a. −3.50 D
b. −4.00 D
c. −5.00 D
d. −5.50 D
25. What is the purpose of a square-edge design for an intraocular lens implant?
a. minimization of positive dysphotopsias
b. reduced rotation of toric lenses
c. diminished incidence of iris chafing
d. decreased posterior migration of lens epithelial cells
27. How does silicone oil affect the power of a biconvex intraocular lens (IOL)?
a. Because the refractive index of oil is greater than that of vitreous, the lens power will be
increased.
b. Because the refractive index of oil is less than that of vitreous, the lens power will be
increased.
c. Because the refractive index of oil is greater than that of vitreous, the lens power will be
reduced.
d. Because the refractive index of oil is less than that of vitreous, the lens power will be
reduced.
28. A patient presents for laser in situ keratomileusis (LASIK) evaluation to eliminate the
need for a distance contact lens correction of −10.00 D. What is the lowest (flattest) treat-
able keratometric reading that will still avoid excessive corneal flattening?
a. 40.00 D
b. 41.00 D
c. 42.00 D
d. 43.00 D
29. A 25-year-old hyperopic patient presents to you for refractive surgery evaluation. You mea-
sure a large angle kappa. Otherwise, they are a good candidate for photorefractive keratec-
tomy (PRK). What is your plan?
a. Center the ablation based on the optical axis.
b. Center the ablation based on the pupillary axis.
c. Center the ablation based on the visual axis.
d. Recommend that they continue to use contact lenses rather than undergo refractive
surgery.
30. What method can be employed to reduce the risk of night vision difficulties such as glare,
starbursts, and haloes in patients who will undergo keratorefractive surgery?
a. decreasing the size of optical zone
b. creating a transition zone larger than the pupil
c. treating preoperatively with miotics
d. increasing the ablation depth
31. What light phenomenon is the underlying basis of optical coherence tomography (OCT)?
a. reflection
b. interference
c. fluorescence
d. polarization
368 ● Study Questions
32. Considering that the eye acts as a simple magnifier, what is the magnification experienced
when one uses a direct ophthalmoscope?
a. 2×
b. 4×
c. 15×
d. 20×
33. What level of visual field loss qualifies a patient for legal blindness concessions in the
United States?
a. hemianopic field loss
b. foveal-sparing scotoma
c. constriction to 20°
d. severe glaucoma field loss with a mean defect of −19.00 D on 30° field testing
34. A patient with macular degeneration reports blurred vision at near. Their visual acuity
is 20/160 in each eye with no refractive error in either eye. What is the most appropriate
prescription for low-vision reading glasses?
a. OD: +4.00 sphere 4∆ BI, OS: +4.00 sphere 4∆ BI
b. OD: +4.00 sphere 8∆ BI, OS: +4.00 sphere 8∆ BI
c. OD: +8.00 sphere 8∆ BI, OS: +8.00 sphere 8∆ BI
d. OD: +8.00 sphere 10∆ BI, OS: +8.00 sphere 10∆ BI
35. What findings reflect the expected effects of a foveal-sparing scotoma from age-related
macular degeneration (AMD) on vision and contrast sensitivity?
a. 20/200 visual acuity, 7 rows on Pelli-Robson chart, normal confrontation visual field
b. 20/25 visual acuity, 2 rows on Pelli-Robson chart, normal confrontation visual field
c. 20/25 visual acuity, 4 rows on Pelli-Robson chart, bilaterally constricted confrontation
visual field
d. 20/25 visual acuity, 7 rows on Pelli-Robson chart, normal confrontation visual field
Answers
1. b. The magnification of a pinhole camera is limited by the distance from the pinhole to the
image plane. The image formed by a pinhole camera is inverted, not upright. The small
aperture limits the light available for image formation, and so lengthy exposure times are
required and sensitivity is poor in dim light. The depth of field is nearly unlimited.
2. b. By definition, a lens with +1.00 D of refractive power forms an image of a distant object
1 m from the lens. The sun is certainly a “distant object”! The formula for the power of a
lens P = 1/f is helpful, where P is the power of the lens in diopters and f is the focal length
of the lens. A +1.00 D lens forms images of a distant object 1.0 m from the lens, on the op-
posite side of the lens from the object. Similarly, a +2.00 D lens could be used and would
be held 0.5 m from the ground.
3. a. The power of the original lens system is given by the vergence equation: U + P = V,
where P is the power of the lens in diopters, U is the object vergence, and V is the image
vergence. It is worth noting that U = 1/u and V = 1/v, where u is the distance from the lens
to the object and v is the distance from the lens to the image. Here, the distant object can
be considered to be located at infinity, so U = 1/οο = 0. Thus, 0 + P = 1/1.00, so P = +1.00 D.
The net power of the new lens system is the sum of the original power plus the power of
the added lens: Pnew = Pold + 3.00 D = 4.00 D. This new lens system forms an image accord-
ing to the vergence equation: 0 + 4.00 D= 1/v, so v = +0.25 m.
4. a. For an eye with uncorrected myopia, the refractive power is stronger than necessary to
focus the light rays on the retina, and the image of a distant object will form in front of the
retina. This is true whether or not the eye is accommodating. For an eye with uncorrected
hyperopia, the image of a distant object will form behind the retina. For a younger patient
with hyperopia or an older patient with a low hyperopic refractive error, accommodation
may be used to bring a near object into focus on the retina.
5. c. The key to converting a power-cross description to a spherocylindrical specification is
to determine the difference between the power in the 2 principal meridians. Here, the dif-
ference is 3.00 diopters (D). For a spherocylindrical lens, the power of cylindrical correc-
tion is oriented 90° away from its power. As we are asked for the answer in minus cylinder
form, the spherical component is the power in the more positive meridian, here +1.00. The
cylindrical component is thus −3.00 D. This power is observed along the 115° meridian
and is created by a minus cylinder lens component with axis 90° away; that is, axis 25°.
When expressed in plus cylinder form, the spherocylindrical specification of this lens is
()
369
370 ● Answers
use of a JCC is to subjectively refine the axis and magnitude of cylinder after placing the
best available estimate of the spherical equivalent of the refraction in front of the eye. In a
phoropter, the most common JCC powers are +0.25 D and −0.25 D, with the axes deter-
()
()
mined by the orientation of the lens, or +0.25 − 0.50 ×__ = −0.25 +0.50 ×__. Red dots
identify the axis of the minus cylinder power, and white dots identify the axis of the plus
cylinder power.
8. a. If the retinoscopy streak is horizontal, the axis of the cylindrical lens is also horizon-
tal (180°). So the spherocylindrical lens combination for this patient, before the working
distance adjustment is subtracted, is +3.00 + 1.00 × 090. The working distance (67 cm, or
0.67 m) must be subtracted from the final refraction. Thus, subtracting 1/0.67 m, or 1.50 D,
yields the correct answer: +1.50 + 1.00 × 090. Note that the cylindrical power acts 90° from
the axis. If the retinoscopy streak is horizontal, the axis of the cylindrical lens is 180°, but
the actual power is at 90°. Accordingly, the powers, after subtracting the working distance,
are +1.50 D at 90° and +2.50 D at 180°.
9. a. If light travels from a denser medium (in this case, water; refractive index 1.33) to a less
dense medium (in this case, air; refractive index 1.00), then light rays bend away from the
surface normal, according to Snell’s law. This redirection of light produces image displace-
ment, thus causing the shark to appear further away than it actually is. Thus, the shark
is closer than it appears to the surfer. However, for a shark (in the water) looking up at a
surfer (in the air), the surfer would actually be further from the shark than the surfer ap-
pears. See Chapter 1, Figure 1-2.
10. a. Unlike light that passes through a spherical lens and is focused to a single point, light
that passes through a spherocylindrical lens is focused to a 3-dimensional image termed
the conoid of Sturm. The images at the ends of the conoid are lines, and the image at the
center of the conoid is a circle. Images between the ends and center of the conoid are el-
liptical in shape. The circle of least confusion is the circular cross section of the conoid of
Sturm that is halfway between the 2 focal lines; it occurs at the spherical equivalent value
of a spherocylindrical lens and provides the best visual acuity.
11. d. A prism disperses white light into a spectrum of wavelengths, creating a rainbow. While
each of the various colors of light is deflected toward the base of the prism, shorter wave-
lengths of light, such as violet, are bent more than longer wavelengths of light, such as
red. This effect—referred to as chromatic aberration—is caused by dispersion, in which
the index of refraction of a medium may vary with the wavelength of the light passing
through it. This phenomenon is the basis for the duochrome test for accommodative con-
trol, in which green light rays are bent more and come into focus more anteriorly than do
red light rays.
12. b. Mirrors have reflecting power but no refracting power. The reflecting power of a mirror
is calculated as P = −1/f = −2/r, where the focal length (f) equals half of the radius of cur-
vature (r). For this mirror, P = −2/r = −2.0 D. Thus the vergence equation U + P = V reads
−1/10.0 + (−2.0) = 1/v, so v = 1/(−2.1) = −0.47 m; that is, 0.47 m away from the mirror
on the side opposite the source object. This will be an erect, virtual image, minified by a
factor of 0.47/10.0 = 0.047. Such a small image is interpreted by the driver as an object far-
ther away than the original source object—as the reminder printed on the mirror states:
“OBJECTS IN MIRROR ARE CLOSER THAN THEY APPEAR.”
13. a. The vergence equation (U + P = V, or 1/u + P = 1/v) can be used to determine the location of
the intermediate and final images, and the formula for transverse magnification (MT = U/V)
Answers ● 371
can be used to calculate the image size. Application of the vergence equation determines
that the intermediate image is located 0.5 m to the left of the first lens (applying U + P = V,
−1 − 1 = −2; applying v = 1/V, 1/−2 = −0.5). Based on MT = U/V (MT = −1/−2 = +0.5x), the
intermediate image is half the height of the original image and upright, given the positive
sign. Now, U + P = V is represented by −0.5 + 1.5 = +1, and the final image is located 1 m to
the right of the second lens. Again, by MT = U/V (MT = −0.5/+1 = −0.5x), the final image is
half the height of the intermediate image and inverted. The magnification of the final image
can be calculated as (0.5)(−0.5) = −0.25, which means that the final image is one-fourth the
size of the original object and inverted. Of note, transverse magnification describes the
image height, while axial (longitudinal) magnification describes the length, or depth, of
the image (measured along the optical axis) and can be calculated as ML = (MT)2.
14. c. Eyes with shorter focal lengths have smaller Airy discs than eyes with longer focal
lengths and therefore diffract light rays less. The Airy disc is the central portion of a pat-
tern of light and dark rings formed when light from a point source passes through a cir-
cular aperture and is diffracted. Diffraction describes the divergence of light as it traverses
through an aperture. The smaller the aperture is relative to the wavelength of light, the
more pronounced the diffraction. The formula for the diameter of the Airy disc is dia-
meter = 2.44λ f/D, where λ is the wavelength of the light, f is the focal length of the eye,
and D is the diameter of the pupil. Diffraction increases as the focal length of the eye
increases, the wavelength of light increases, or the pupil size decreases. Retinal conditions
have no effect on the size of the Airy disc.
15. b. The prescription in the question stem can be written in plus cylinder notation as
−0.25 + 0.25 × 090 or in minus cylinder notation as plano −0.25 × 180. This is an example
of simple myopic astigmatism, with 1 focal line on the retina. A patient with simple hyper-
opic astigmatism has a hyperopic refraction with 1 focal line on the retina; an example is
plano +0.50 × 090 (same as +0.50 −0.50 × 180). Compound myopic astigmatism involves
2 focal lines in front of the retina (sph and cyl negative in both plus and minus cyl nota-
tion), while compound hyperopic astigmatism involves 2 focal lines behind the retina (sph
and cyl positive in plus and minus cyl notation). Mixed astigmatism involves 1 focal line
behind the retina and 1 in front (sph and cyl with opposite signs in both plus and minus
cyl notation).
16. b. The nodal point in the Gullstrand schematic eye is located 17 mm in front of the ret-
ina. Using the geometric principle of similar triangles, the diameter of the retinal image
of an object located 333 mm from the eye, with a diameter of 9 mm, is 9 mm × (17/333) =
0.46 mm. The corresponding retinal image is about 20 times smaller than its target light in
a Goldmann perimeter. This is because the perimeter’s radius (333 mm) is approximately
20 times larger than the distance from the eye’s nodal point to the retina (17 mm). Simi-
larly, the retinal image size of a Snellen letter on an eye chart can be calculated as follows:
retinal image height / Snellen letter height = 17 mm / distance from chart to eye.
17. c. A change in acuity by a factor of 10 (such as a change from 20/20 to 20/200) corresponds
to a difference of 1.0 logMAR, since 1.0 is the common logarithm of 10. The common
logarithm of 2 is 0.301, which we round to 0.30 for clinical purposes. This is “built-in” to
the common ETDRS eye charts, which feature proportional steps of 0.10 logMAR between
lines, so that a 3-line change in acuity corresponds to a change of 0.30 logMAR (ie, a dou-
bling [or halving] of the visual angle). The logMAR is calculated as the logarithm of the
reciprocal of the Snellen fraction (eg, 20/200 −> reciprocal is 200/20 = 10 −> logarithm of
10 = 1).
372 ● Answers
18. a. This patient is overminused. Shorter wavelengths of light, such as green, are bent more
than longer wavelengths, such as red. When the patient sees the green letters more clearly
than the red letters, the shorter (more bent) green wavelength is more in focus and closer
to the retina than the longer red wavelength of light. Therefore, more plus would need to
be added to the refraction to bring the red wavelength closer to the retina. A balanced
refraction is achieved when the green and red letters are equidistant from the retina and
each one appears equally clear. RAM-GAP is a mnemonic device to help learners remem-
ber to “red add minus, green add plus.”
19. d. Refracting in a short room will produce results that are inherently inaccurate because
normal fogging techniques to relax accommodation will not be effective. One method to
relax the accommodation would be to move the patient to a longer hallway to complete
the refraction in a trial frame. Another option would be to install mirrors to optically
extend the room length to 6m.
20. a. Prism is induced when one looks through a lens at any point other than its optical
center. Therefore, when the eye is looking down and encounters the top of a bifocal add,
the image jumps, unless the optical center of that bifocal add is at the top of the segment.
Executive-style segments have their optical centers at the top of the segment; therefore,
they induce no image jump. The optical center of a flat-top segment is typically 3 mm
from the top of the segment, whereas the optical center of a round-top segment is much
lower. Therefore, flat-top segments cause some image jump, but less than that caused by
round-top segments. The “transition zone” is the name for the area of progressive change
in lens power in a progressive addition lens (PAL). While PALs do not induce image jump,
they are (by definition) not bifocals.
21. c. Prentice’s rule states that the prism power of a lens at any point on its surface, in prism
diopters (∆), is equal to the distance away from the optic center (h) in centimeters times the
power of the lens in diopters (D), or ∆ = hD. Minus-power lenses can be drawn as 2 prisms
with the bases directed outward, while plus-power lenses can be drawn as 2 prisms with the
bases directed inward. For the right eye, ∆ = 0.8 × 1 = 0.8∆. Based on the diagram on the left,
below, we know that this is 0.8∆ base down. For the left eye, ∆ = 0.8 × 2 = 1.6∆. Based on the
diagram on the right, below, we know that this is 1.6∆ base up. Given that the prisms are
opposite each other vertically, the prism diopters are added together, and the induced phoria
is calculated to be 2.4∆.
0.8 cm
0.8 cm
22. c. Ametropia is corrected with a lens whose focal point coincides with the eye’s far point.
First, locate the far point of the eye. Because the patient is myopic, we know that the far
point is in front of the eye; by convention, this is represented as a negative number. The
distance from the patient’s eye to the far point is 10 cm, and the vertex distance is 1.3 cm.
Answers ● 373
Thus, the far point of the eye and the focal point of the contact lenses coincide at this
location: −0.100 m − (−0.013 m) = −0.087 m (in front of the eye). Therefore, the required
lenses have a power of 1/(−0.087) = −11.50 D.
23. d. The old glasses do not give clear vision for distance, and they now function almost as
reading glasses. New spectacles, fully correcting the distance vision about 12 mm from
the eye, have the advantage of “near effectivity,” which reduces accommodative demand,
compared to an emmetropic eye or one corrected by a contact lens. People with myopia
who are in the patient’s age range and who are still able to read with their correct distance
glasses are likely to find it more difficult to read with contact lenses. The presbyopic myo-
pic patient who wears glasses can read more easily by pushing the glasses farther away
from the eyes.
24. a. The curvature of the contact lens (44.00 D) is flatter than the cornea (45.00 D) by 1 D.
This means that the tear film creates a negative lens power: −1.00 D. To counteract this
tear film, +1.00 D must be added to the refraction: −4.50 D + 1.00 D = −3.50 D. This can
be remembered with the SAM-FAP rule: steeper add minus, flatter add plus. Since the
contact lens is flatter by 1.00 D, add +1.00 D to the refraction of −4.50 D for a power of
−3.50 D.
25. d. Lenses with a square edge produce increased pressure of the lens edge against the pos-
terior capsule, resulting in decreased posterior migration of lens epithelial cells and a di-
minished rate of posterior capsular opacification (PCO). Many modern lens implants are
created with a rounded anterior edge and square posterior edge to maintain a decreased
risk of PCO while reducing the risk of iris chafing, which can be especially high with sul-
cus placement of a lens with a square anterior edge. A square-edge lens design has been
associated with positive dysphotopsias, with smooth edges found to minimize dysphotop-
sias. A frosted-edge haptic design has been shown to reduce rotation of toric lenses.
26. c. Misalignment of a toric intraocular lens by as little as 10° will result in a substantial
reduction in the efficacy of the astigmatism correction. Misalignment by 30° will have a
neutral effect, negating the benefit of the toric lens. Misalignment by 90° will double the
patient’s resultant astigmatism. Any misalignment of a toric intraocular lens (other than a
misalignment of exactly 90°) will also rotate the patient’s postoperative astigmatic axis.
27. c. Because the refractive index of silicone oil is greater than that of vitreous, the oil filling
the vitreous cavity reduces the optical power of the posterior surface of the intraocular
lens (IOL) in the eye when a biconvex IOL is implanted. This problem must be counter-
acted by an increase in IOL power of 3 to 5 diopters.
28. b. For myopic laser in situ keratomileusis (LASIK), a change of approximately 0.80 D in the
average keratometry value (K) results from each 1.00 D of refractive change. The following
equation is used to predict corneal curvature after keratorefractive surgery: Kpostop = Kpreop +
(0.8 × RE), where Kpreop and Kpostop are the preoperative and postoperative K readings, re-
spectively, and RE is the refractive error to be corrected at the corneal plane. When myopic
corrections are performed, the minimum corneal power tolerable is 33.00 D. Therefore, the
formula becomes 33.00 = Kpreop + (0.8 × −10.00), where Kpreop equals 41.00. Both excessively
flat (<33.00 D) and excessively steep (>50.00 D) corneal powers should be avoided post-
operatively, as these can result in decreased visual quality and increased higher-order aber-
rations. For hyperopic LASIK, approximately 1.00 D of corneal steepening occurs per 1.00 D
of corneal ablation.
374 ● Answers
29. c. Corneal refractive ablations should be centered on the visual axis in a patient with a high
angle kappa, in order to avoid a decentered ablation. Angle kappa is the angle between the
pupillary axis (line perpendicular to the corneal surface, passing through the pupillary
midpoint) and the visual axis (line connecting point of fixation to fovea); see Chapter 3,
Figure 3-3. High angle kappa may result from hyperopia, exotropia, or macular dragging.
When identified preoperatively, the ablation can be shifted more nasally to center on the
visual axis, rather than the pupillary axis. Patients with decentered ablations may develop
coma, which occurs when rays at one edge of the pupil converge before rays at the oppo-
site edge of the pupil do. This higher-order aberration can result in an image resembling a
comet with a tail. Decentered ablations are particularly problematic in a hyperopic correc-
tion. Large angle kappa is not a contraindication to refractive surgery.
30. b. Postoperative night-vision problems are well described and are often associated with
large pupil sizes (>8 mm). These symptoms, which include the appearance of glare, star-
bursts, and haloes, decreased contrast sensitivity, and decreased quality of vision at night,
tend to occur in patients with both large pupils and small treatment zones (≤6 mm). In
appropriate candidates, third-generation lasers utilize larger optical and transition zones
to help reduce the incidence and severity of night-vision problems. The accepted standard
transition zone between ablated and unablated cornea is 0.5–1.0 mm larger than the pupil
to help reduce night-vision problems, but in order to conserve corneal tissue, smaller
optical zones are used in the correction of high myopia. The incidence of night-vision
problems in these patients increases in part because of the mismatch between the size of
the pupil and that of the optical zone. In this patient, increasing the optical zone may not
be feasible without significantly thinning the cornea and increasing the risk of postsurgi-
cal corneal ectasia. Preoperative treatment with miotics does not reduce the risk of night-
vision problems. In some of the patients, postoperative miotics such as brimonidine 0.2%
or pilocarpine 0.5% or 1% may help reduce the postoperative night-vision problems.
31. b. Optical coherence tomography (OCT) is an optical analogue to ultrasound imaging
and uses infrared light instead of sound. The much higher speed of light compared with
sound allows for finer resolution, but direct electronic measurement of the shorter “echo”
times it takes light to travel from different structures at axial distances within the eye is
not feasible. Interferometry makes it possible to overcome this difficulty. More precisely,
OCT uses interference of tunable broadband coherent light to generate optical sections of
the retina and cornea.
32. c. With direct ophthalmoscopy, the examiner uses the optics of the patient’s eye as a
simple magnifier to look at the retina. The optics of the emmetropic eye are approximately
+60.00 D, thus the magnification is approximately 60/4, or 15×, according to the formula
for a simple magnifier (Mag = D/4). Interestingly, this magnification varies. The optic
disc will appear larger in a myopic patient and smaller in a hyperopic patient when the
dial on the direct ophthalmoscope is rotated to correct the patient’s refractive error. For
a myopic patient, a minus lens is dialed in to overcome the extra plus power “error lens”
inside the patient’s eye. The 2 lenses create a Galilean telescope, increasing magnification
and decreasing the field of view. Similarly, a reverse Galilean telescope is formed during
examination of a hyperopic patient, which decreases magnification.
33. c. The definition of legal blindness in the United States requires the better-seeing eye to
exhibit best-corrected visual acuity (BCVA) of 20/200 or worse, or a visual field with 20°
or less around central fixation, measured with a Goldmann III 4e target. A point seen at
10 dB or higher on a static automated threshold field test is considered to be seen with a
Answers ● 375
4. Which electrophysiologic test is abnormal in acute zonal occult outer retinopathy (AZOOR)?
a. full-field electroretinography
b. multifocal electroretinography
c. single-fiber electromyography
d. visual evoked potential
435
436 ● Study Questions
5. What central ner vous system demyelinating disorder commonly presents with an optic
neuritis that is associated with bilateral optic nerve and perineural enhancement on MRI?
a. glial fibrillary acidic protein astrocytopathy
b. multiple sclerosis (MS)
c. myelin oligodendrocyte glycoprotein immunoglobulin G–associated disorder
d. neuromyelitis optica spectrum disorder
6. Which visual field defect is typically associated with Leber hereditary optic neuropathy?
a. arcuate defect
b. cecocentral scotoma
c. altitudinal defect
d. bitemporal hemianopia
7. A 19-year-old woman has new-onset headaches and transient visual obscurations. She
has severe acne and uses a topical medication prescribed by her dermatologist. She weighs
110 pounds and is 5 feet, 2 inches tall. Visual acuity is 20/20 OU with normal color vision.
Static perimetry shows blind-spot enlargement bilaterally, and ductions are full. What
ONH finding is expected to be seen on ophthalmoscopy?
a. mild hyperemia with telangiectatic vasculature
b. bilateral pallor
c. bilateral edema
d. normal architecture
8. What clinical feature best distinguishes a parietal lobe lesion from an occipital lobe lesion?
a. abnormal optokinetic nystagmus response
b. achromatopsia
c. homonymous hemianopia
d. relative afferent pupillary defect (RAPD)
9. Damage to the Meyer loop produces what type of homonymous visual field defect?
a. contralateral inferior
b. contralateral superior
c. ipsilateral inferior
d. ipsilateral superior
10. A 79-year-old woman presents with a 1-month history of difficulty chewing food, weight
loss, and headache. Her dentist recently diagnosed temporomandibular joint inflamma-
tion and prescribed ibuprofen. Over the past week, she has experienced transient visual
Study Questions ● 437
loss (TVL) in the left eye. Findings from the ophthalmologic examination are normal
except for 1 cotton-wool spot in the left eye. What is the best next step in the management
of this patient?
a. Stop the ibuprofen to minimize postoperative bleeding and schedule a temporal artery
biopsy.
b. Obtain a Westergren erythrocyte sedimentation rate and initiate systemic
corticosteroids.
c. Begin aspirin and obtain a carotid Doppler ultrasonogram and echocardiogram.
d. Obtain a facial CT scan.
11. An 80-year-old man with hypertension presents with 3 episodes of binocular TVL over
the last week. He describes a sudden and complete blackout of the entire visual field for
1 to 2 minutes that resolved as quickly as it started. What is the most likely cause of this
patient’s symptoms?
a. migraine
b. arteriovenous malformation
c. seizures
d. ischemia
12. A 36-year-old woman with a history of migraine reports seeing a trail of an image when
looking at a moving object. Which term is used to describe this phenomenon?
a. visual snow
b. Riddoch phenomenon
c. visual aura
d. illusory palinopsia
13. Acquired cases of prosopagnosia typically result from stroke affecting what region(s) of
the brain?
a. bilateral occipitoparietal lobes or left parietal lobe
b. bilateral inferior occipitotemporal lobes or right inferior occipital lobe
c. bilateral posterior occipital lobes or splenium of corpus callosum and unilateral occip-
ital lobe
d. bilateral superior occipitoparietal lobes or right superior parietal lobe
15. A 35-year-old man has vertical, binocular diplopia. He has a left hypertropia of 5 prism
diopters (Δ) in primary gaze that is absent on right gaze but increases to 9Δ on left gaze.
There is also a clockwise gaze-evoked nystagmus. What additional clinical finding would
be expected on examination?
a. variable ptosis with sustained upgaze
b. slow left-eye adducting saccade
c. proptosis of right eye greater than left
d. increased vertical fusional amplitude
16. A 20-year-old man notes pulsatile synchronous tinnitus and horizontal diplopia following
head trauma from a motor vehicle accident. Examination reveals enlarged, corkscrew-
shaped conjunctival blood vessels; a sixth nerve palsy; and an audible cranial bruit. On
neuroimaging, what vessel would be expected to be dilated?
a. vein of Galen
b. superior ophthalmic vein
c. transverse sinus
d. ophthalmic artery
17. A 54-year-old man with hypertension experienced onset of complete left ptosis 2 days
ago. There is anisocoria and partial limitation of ductions in the left eye. What is the most
appropriate initial management strategy?
a. observation
b. CTA
c. edrophonium test
d. referral to a primary care physician
18. In a patient with ptosis but no extraocular motility limitation, what additional finding
should prompt a workup for myasthenia gravis (MG)?
a. pupil involvement
b. loss of corneal sensation
c. eyelid elevation on adduction
d. orbicularis oculi muscle weakness
19. Which of the following would be the most likely associated examination finding in a pa-
tient with convergence-retraction nystagmus?
a. limitation of conjugate upgaze
b. monocular elevation deficit
c. apraxia of eyelid opening
d. comitant esotropia
Study Questions ● 439
20. A patient has a Horner pupil, ipsilateral impairment of facial pain and temperature sensa-
tion, and contralateral impairment of pain and temperature sensation over the trunk and
limbs. A lesion involving which anatomical structure would explain these findings?
a. cavernous sinus
b. internal carotid artery
c. lateral medulla
d. brachial plexus
21. A 72-year-old man is admitted to the neurology department because of an acute stroke.
Both eyes are tonically deviated to the left, but the doll’s head maneuver produces full ocu-
lar motility. What is the most likely location of the stroke to explain the clinical findings?
a. left pons
b. right pons
c. left cortex
d. right cortex
22. In what class of eye movements does dysfunction characterize congenital ocular motor
apraxia?
a. vertical smooth pursuit
b. horizontal smooth pursuit
c. vertical saccades
d. horizontal saccades
24. What type of nystagmus is characterized by reversal of the normal pattern of optokinetic
nystagmus?
a. fusional maldevelopment nystagmus syndrome (latent nystagmus)
b. infantile nystagmus syndrome (congenital nystagmus)
c. spasmus nutans syndrome
d. gaze-evoked nystagmus
440 ● Study Questions
25. A patient reports oscillopsia while chewing. Examination shows reduced voluntary verti-
cal eye movements. What is the most likely cause of these symptoms and findings?
a. midbrain degeneration
b. Tropheryma whipplei infection
c. Chiari type I malformation
d. chronic alcohol intoxication
26. What is the most likely cause of unilateral upper eyelid retraction on attempted ipsilateral
adduction?
a. aberrant regeneration of cranial nerve (CN) III
b. thyroid eye disease
c. dorsal midbrain syndrome
d. Duane syndrome
27. A 28-year-old woman reported having a small left pupil for the past few weeks. She denied
ptosis, diplopia, or pain. In bright light, the pupils measured 6 mm OD and 3 mm OS. In
dim light, the pupils measured 6 mm OD and 4.5 mm OS. What is the most appropriate
next step in the management of this patient?
a. pilocarpine 1% in each eye
b. pilocarpine 0.1% in each eye
c. cranial MRI with fat suppression
d. cocaine 10% in each eye
28. An asymptomatic, healthy 38-year-old man has small, irregular pupils that react mini-
mally to light stimulus. With attempted reading, the pupils become even smaller. What is
the most appropriate next step in the management of this patient?
a. observation and follow-up for 1 month
b. syphilis serology
c. apraclonidine testing
d. orbital MRI with contrast
29. In benign essential blepharospasm, where in the central nervous system is the seventh
nerve hyperactivity thought to originate?
a. midbrain
b. cerebellopontine angle
c. pons
d. basal ganglia
Study Questions ● 441
30. A 53-year-old woman reports worsening ability to keep her eyes open, which has resulted
in her not being able to read or drive. She is noted to have forceful, episodic bilateral eyelid
closure with simultaneous contraction of the corrugator and procerus muscles. What is
the best next step in management of this patient?
a. acetylcholine receptor antibody test
b. psychiatric referral
c. botulinum toxin injection
d. cranial MRI
31. A 35-year-old man with a history of migraine with aura presents with new onset of weak-
ness. MRI demonstrates widespread leukoencephalopathy. His father had early-onset de-
mentia. A mutation in which gene is responsible for his presentation?
a. ABCA4
b. NOTCH3
c. OPA1
d. WFS1
32. A 40-year-old woman had sudden onset of severe headache, diplopia, and decreased vi-
sion in both eyes. Examination shows bilateral ptosis, poorly reactive pupils, and ophthal-
moparesis. What abnormality is most likely to be present on neuroimaging?
a. posterior communicating artery aneurysm
b. internal carotid artery dissection
c. extensive periventricular white matter lesions
d. hemorrhagic sellar/suprasellar mass
33. A 38-year-old man has periodic right-sided headache with right eye redness and tearing.
The pain occasionally wakes him from sleep. What additional right eye or orbital sign is
most likely to be associated with his headache events?
a. 1+ anterior chamber inflammation
b. 2-mm upper eyelid ptosis
c. grade II ONH swelling
d. 5-mm pupil, poor light reactivity
34. What term describes an oscillating eye movement characterized by irregular, brief bursts
of rapid frequency and low amplitude with no slow phase?
a. end-gaze nystagmus
b. voluntary flutter
c. spasm of the near reflex
d. see-saw nystagmus
442 ● Study Questions
35. A 12-year-old boy presents with sudden vision loss in the right eye. Visual acuity was no
light perception OD and 20/20 OS. There was no RAPD; nonorganic vision loss is sus-
pected. Which test is most appropriate to perform to demonstrate normal 20/20 vision
OD in this patient?
a. mirror test
b. fogging test
c. optokinetic nystagmus drum
d. base-out prism test
36. An 18-year-old woman reports tunnel vision in both eyes. Visual acuity is 20/400 OU,
pupils react briskly, and anterior and posterior segments are normal. She is able to ambu-
late without difficulty. What finding on Goldmann perimetry would confirm a functional
component to her vision loss?
a. overlapping and spiraling isopters
b. central scotomata
c. nasal step
d. generalized visual field constriction
37. An 89-year-old woman with a history of osteoporosis and poorly controlled diabetes mel-
litus was recently diagnosed with temporal artery biopsy–proven giant cell arteritis. Ini-
tiation of which treatment agent is most appropriate to reduce her risk of corticosteroid
adverse effects?
a. eculizumab
b. rituximab
c. teprotumumab
d. tocilizumab
38. A 48-year-old woman suddenly has painful vision loss in the right eye. Two weeks later,
similar symptoms develop in the left eye. Visual acuity is counting fingers OD and hand
motion OS. Both pupils are sluggishly reactive to light without an RAPD. The fundus of the
right eye is normal, but there is mild ONH edema in the left eye. MRI shows bilateral optic
nerve enhancement. After treatment with intravenous methylprednisolone, she experiences
only minimal recovery of vision. Which condition is the most likely cause of the vision loss?
a. MS
b. neuromyelitis optica
c. giant cell arteritis
d. AZOOR
Study Questions ● 443
39. Which medication has been most commonly associated with mental status changes and
hemianopia?
a. fingolimod
b. glatiramer acetate
c. natalizumab
d. interferon beta-1a
40. During ocular motility testing, a patient has episodic diplopia lasting 30–60 seconds and
is noted to have a transient right exotropia in primary gaze after maintaining both eyes in
extreme right gaze for several seconds. Which of the following is most commonly associ-
ated with the clinical findings?
a. poorly controlled diabetes mellitus treated with oral hypoglycemic
b. progressive MS treated with glatiramer acetate
c. metastatic breast cancer treated with tamoxifen
d. tuberculum sellae meningioma treated with radiation therapy
Answers
1. a. Exiting the brainstem, the abducens nerve (cranial nerve [CN] VI) runs rostrally within
the subarachnoid space on the surface of the clivus from the area of the cerebellopontine
angle to the superior portion of the posterior fossa. The nerve pierces the dura approxi-
mately 1 cm below the petrous apex and travels beneath the petroclinoid ligament (the
Gruber ligament, which connects the petrous pyramid to the posterior clinoid) to enter
the canal of Dorello. Lesions within the clivus and certain skull base fractures may damage
the abducens nerve in this location. The oculomotor and trochlear nerves travel above the
clivus in the subarachnoid space to pass into the cavernous sinus. The fascicles of the tri-
geminal nerve (CN V) enter the brainstem ventrally in the pons and extra-axially traverse
the subarachnoid space to penetrate the dura just over the petrous pyramid.
2. b. Computed tomography (CT) is the preferred imaging modality to detect acute sub-
arachnoid hemorrhage. Magnetic resonance imaging (MRI) is not preferred for detect-
ing acute hemorrhage, but it can clarify the evolution of intraparenchymal hemorrhage.
Computed tomography angiography (CTA) and magnetic resonance angiography (MRA)
are preferred for detecting aneurysms and carotid stenosis.
3. b. Optical coherence tomography (OCT) provides noninvasive, high-resolution, in situ
visualization of the retina and optic nerve head (ONH). Automatic segmentation of OCT
images allows for quantification of various retinal layers. The peripapillary retinal nerve
fiber layer (pRNFL) thickness measurement provides indirect information about the axo-
nal integrity of the optic nerve, with thinning indicating axonal damage; however, this
measurement is confounded in the setting of ONH swelling, in which there is thickening
of the pRNFL that can coexist with axonal loss. In these cases, evaluation for thinning of
the ganglion cell–inner plexiform layer, which does not swell, can provide an early indica-
tion of neuronal loss. The ellipsoid zone of the outer retina is affected early in autoimmune
retinopathy and hydroxychloroquine toxicity but does not provide useful information in
the setting of optic neuropathy. OCT angiography (OCTA) uses OCT technology to gen-
erate depth-resolved images of the retinal and choroidal vasculature without the need for
contrast dye administration. OCTA is useful for the evaluation of retinal vascular dis-
ease and age-related macular degeneration, but its uses in optic neuropathy are still being
explored.
4. b. Electroretinography (ERG) measures the electrical activity in the retina in response to
various light stimuli under different states of light adaptation. Multifocal ERG (mfERG)
records and maps ERG signals from up to 250 focal retinal locations within the central
30°. It can differentiate optic nerve from macular disease in occult central vision loss,
as the signal generally remains normal in optic nerve disease. Causes of outer retinal
degeneration, such as multiple evanescent white dot syndrome, acute idiopathic blind-
spot enlargement syndrome, and acute zonal occult outer retinopathy (AZOOR), result
in decreased waveforms on mfERG. Full-field ERG (ffERG) detects diffuse retinal disease
in cases of generalized or peripheral vision loss; because ffERG measures only a mass
response of the entire retina, localized retinal disease, even with severe visual acuity loss,
may not produce an abnormal response. Visual evoked potential (VEP) testing measures
electrical signals over the occipital cortex produced in response to a visual stimulus. VEP
is useful for evaluation of the visual pathways in inarticulate patients and in patients
with suspected nonorganic vision loss. Single-fiber electromyography may be abnormal
445
446 ● Answers
in neuromuscular conditions like myasthenia gravis (MG) but would not provide useful
information regarding vision loss.
5. c. Myelin oligodendrocyte glycoprotein immunoglobulin G–associated disorder (MOGAD)
can involve recurrent optic neuritis, myelitis, and encephalitis, with optic neuritis being the
most common of these presentations in adults. Compared with typical multiple sclerosis
(MS)–optic neuritis, MOG-IgG–optic neuritis is more likely to be bilateral, recurrent, and
associated with ONH edema on clinical examination. In addition, perineural enhancement
is often observed on MRI (enhancement extending to the optic nerve sheath and surround-
ing periorbital fat). Although neuromyelitis optica spectrum disorder should be considered
in cases of bilateral or recurrent optic neuritis, it is not commonly associated with perineural
involvement. Optic neuritis associated with MS is usually unilateral at presentation and does
not typically show perineural enhancement. Glial fibrillary acidic protein astrocytopathy
presents as a meningoencephalitis sometimes with ONH edema; however, isolated optic
neuritis has not been described in association with this condition.
6. b. Central or cecocentral visual field defects are typical of Leber hereditary optic neuropa-
thy (LHON). Primary open-angle glaucoma is usually characterized by slowly progressive
arcuate and peripheral visual field loss. Arcuate defects may be seen with chronic pap-
illedema. In nonarteritic anterior ischemic optic neuropathy, the most common pattern
of visual field loss is an altitudinal defect, but any pattern may be observed. Bitemporal
hemianopia suggests a chiasmal process, such as compression from a pituitary adenoma
or craniopharyngioma.
7. c. Bilateral ONH edema is observed in papilledema from increased intracranial pressure.
Use of anti-acne medication (including tetracyclines like minocycline, as well as reti-
noic acid) is associated with idiopathic intracranial hypertension, as in this case. Normal
ONH appearance is incorrect because the patient’s history and visual field findings are
consistent with papilledema. ONH pallor is incorrect because there is no indication of
optic neuropathy, suggested by visual field testing showing only blind-spot enlargement,
by normal color vision, and by normal visual acuity. Mild hyperemia with telangiectatic
vasculature is characteristic of pseudoedema from LHON in the acute stage, or possibly
consistent with diabetic papillitis. Neither diagnosis is indicated in this prompt, which
describes a young woman with symptoms of increased intracranial pressure.
8. a. Parietal and occipital lobe lesions can produce similar visual field defects (homonymous
hemianopia) with normal pupil responses (no relative afferent pupillary defect [RAPD]).
Cerebral achromatopsia is a feature of occipital lobe lesions. Pursuit and reflexive saccade
movements are initiated in the parietal lobe; consequently, abnormal optokinetic nystagmus
(OKN) responses can occur with a parietal lobe lesion but not with an occipital lobe lesion.
9. b. From the lateral geniculate nucleus, inferior visual fibers first course anteriorly and then
laterally and posteriorly to the Meyer loop of the temporal lobe. Superior fibers course
posteriorly in the parietal lobe. Lesions affecting the Meyer loop thus produce superior
incongruous homonymous defects contralateral to the lesion, also known as pie in the sky
defects. Lesions affecting the optic radiations in the parietal lobe produce predominantly
inferior homonymous defects affecting the contralateral visual field. No optic radiation
lesion would produce an ipsilateral visual field defect.
10. b. The patient’s clinical picture is highly suggestive of giant cell arteritis (GCA). GCA is
an ophthalmologic emergency because of the potential for bilateral, irreversible vision
loss. This patient is already visually symptomatic and has evidence of possible posterior
Answers ● 447
segment ischemia (cotton-wool spot). The most important next step is the immediate
initiation of appropriately dosed systemic corticosteroids, either orally or intravenously,
along with initial serologic testing; in cases with a high suspicion for GCA, the results of
serologic testing should not delay corticosteroid therapy. Serology usually includes the
triad of Westergren erythrocyte sedimentation rate, C-reactive protein, and complete
blood count. The gold standard for the diagnosis of GCA is a temporal artery biopsy
(TAB), which can be safely performed in most patients even while they are anticoagulated.
TAB should be performed as soon as possible, ideally within 10–14 days, as corticosteroid
treatment can alter histologic findings; in some cases, however, the biopsy result remains
positive for months after the start of therapy. Although carotid stenosis may present with
amaurosis fugax, and carotid ultrasonography may in fact be necessary should the workup
for GCA prove negative, the leading diagnosis in this case remains GCA. Jaw claudication
can be misdiagnosed by clinicians as temporomandibular joint disease, but jaw or tongue
claudication is the symptom most specific for GCA. This symptom should always be sought
by the clinician in any patient older than 50 years who reports transient visual loss (TVL).
11. d. Episodes of complete binocular TVL may represent a transient ischemic attack (TIA)
that involves the occipital lobes in the distribution of the basilar artery or posterior cere-
bral arteries. These episodes are particularly common in older patients with vascular
risk factors or cardiac anomalies. Patients with suspected occipital TIA must be referred
immediately to an emergency department and evaluated by a neurologist.
Migraine is the most common cause of TVL that is generally hemianopic, involving
dynamic positive visual phenomena such as zigzag flashes, and lasts between 20 and 30 min-
utes; however, the duration can be outside this range. Arteriovenous malformation or other
mass may be suspected in a patient with hemianopic visual field loss that is repeatedly in the
same location, especially with associated headache that worsens in frequency or severity. Sei-
zures may cause transient hemianopic unformed hallucinations such as grid patterns more
frequently than negative visual symptoms.
12. d. Palinopsia is visual perseveration after removal of the original stimulus (multiple after-
images) and can be divided into 2 categories: illusory and hallucinatory. Illusory palinop-
sia is triggered by contrast and/or motion; the afterimage or visual trails appear in the
same location in the visual field as the original stimulus. Migraine, hallucinogenic drugs
such as LSD, and some medications can cause illusory palinopsia. In contrast, hallucina-
tory palinopsia is not affected by environmental conditions, and the perseverated image
can occur in a different location in the visual field than the original stimulus. When the
afterimages are associated with homonymous hemianopia and appear in the blind hemi-
field, a posterior cortical lesion is usually present.
Visual snow also is often seen in patients with migraine and is described as snowy, pix-
elated vision, similar to television static. It is not uncommon for a patient to have both visual
snow and illusory palinopsia. Visual aura is also common in migraineurs and is classically
described as scintillating scotoma with a fortification spectrum that lasts 5–60 minutes and
is usually followed by headache. Riddoch phenomenon is the preservation of the perception
of motion in a blind hemifield.
13. b. A more specific form of agnosia, prosopagnosia is characterized by the inability to
recognize familiar faces. Acquired cases result predominantly from stroke. The condi-
tion usually occurs with bilateral inferior occipitotemporal lobe damage but may also
occur with right inferior occipital lobe damage. The ventral occipitotemporal pathway
helps process the physical attributes of an object. Examination typically shows superior
448 ● Answers
homonymous visual field defects. The dorsal occipitoparietal pathway is responsible for
visual–spatial analysis and for guiding movements toward items of interest. Alexia with-
out agraphia (patient cannot read but can write) results from damage to the splenium of
the corpus callosum and the left occipital lobe.
14. c. Balint syndrome results from bilateral occipitoparietal lesions and clinically consists of
the triad of simultanagnosia, optic ataxia, and acquired ocular motor apraxia. Lesions of
the medial longitudinal fasciculus result in internuclear ophthalmoplegia. Bilateral ven-
tral pathway dysfunction that affects occipitotemporal projections can cause object agno-
sia. Lesions of the optic tract cause complete or incomplete homonymous hemianopia.
15. b. This patient has an incomitant left hypertropia (worse in left gaze) and gaze-evoked
nystagmus. The hypertropia does not follow a CN pattern, and the presence of nystagmus
suggests a supranuclear disorder such as skew deviation. Skew deviation may occur with
internuclear ophthalmoplegia (INO) in a young person who has demyelinating disease,
and in an older person may result from ischemia. Given the left hypertropia, the lesion
causing the skew deviation is most likely on the left side of the brainstem; the hallmark of
a left INO would therefore be slowed adduction saccadic velocity in the left eye.
Both thyroid eye disease (which can cause proptosis) and ocular MG (which can cause
fluctuating ptosis) may produce the same ocular deviation (left hypertropia worse in
left gaze), but they would not produce nystagmus. Increased vertical fusional amplitude
occurs most commonly with long-standing abducens nerve palsy or other vertical stra-
bismus and would not be expected in this patient, whose strabismus does not fit that
pattern.
16. b. The patient has classic signs and symptoms of a direct carotid-cavernous sinus fistula.
A high-flow, direct fistula most commonly occurs after severe head trauma and produces
a cranial bruit. Arterialization of conjunctival vessels is a classic sign of carotid-cavernous
sinus fistulas. These fistulas often produce elevated intraocular pressure and proptosis.
Diplopia can occur from congestion of the extraocular muscles or involvement of any of
the CNs in the cavernous sinus, most commonly CN VI. In rare cases, a posteriorly drain-
ing fistula can cause an isolated CN palsy with a quiet-appearing eye. Abnormal connec-
tions between the cavernous sinus and the carotid artery or its branches introduce high
arterial pressure into the normally low-pressure venous circulation of the cavernous sinus.
Such a high-pressure connection may reverse blood flow within the superior ophthalmic
vein, leading to an enlarged and dilated superior ophthalmic vein, which is typically seen
on CT or MRI. The other vessels listed (vein of Galen, transverse sinus, and ophthalmic
artery) would not be expected to be enlarged from a carotid-cavernous sinus fistula.
17. b. The description is of a patient with pupil-involving third nerve palsy, the usual causes of
which are intracranial aneurysm, microvascular ischemia, and diabetes. Because an aneu-
rysm may be fatal, it must be ruled out. Observation is not appropriate. The edrophonium
test could be considered for a patient suspected of having MG, which can produce ptosis
and abnormal extraocular motility but does not cause anisocoria. Evaluation by a primary
care physician to control hypertension is good medical practice, but a compressive lesion
needs to be ruled out first in a patient with a pupil-involving third nerve palsy.
18. d. Ptosis is a common problem whose etiology varies from benign involutional changes
to serious neurologic disease. A thorough examination is critical for determining the
workup. Unilateral or bilateral ptosis is the most common sign of MG. In addition to
fatigability and improvement with ice testing, myasthenic ptosis is often accompanied
Answers ● 449
by orbicularis oculi muscle weakness on examination. This is tested with the patient
squeezing both eyes shut and the examiner manually applying pressure around the eyes to
attempt to reopen them. MG never involves the pupil or sensory nerves, and it does not
cause aberrant regeneration/synkinesis of other extraocular muscle nerve fibers.
19. a. Convergence-retraction nystagmus is a feature of the dorsal midbrain syndrome, which
also includes conjugate limitation of vertical gaze (usually upgaze), mid-dilated pupils
with light–near dissociation, and retraction of the eyelids in primary position (Collier
sign). Limitation of conjugate upgaze is the most common feature. Supranuclear fibers
that control vertical gaze decussate through the pretectum as they pass to the rostral inter-
stitial nucleus of medial longitudinal fasciculus, the midbrain structure that functions as
the saccadic generator for vertical eye movements. Comitant esotropia, monocular eleva-
tion deficit, and apraxia of eyelid opening are not the correct answers because they are not
part of the dorsal midbrain syndrome.
20. c. Lateral medullary syndrome (Wallenberg syndrome) produces an ipsilateral Horner
syndrome, ipsilateral impairment of pain and temperature sensation over the face (involve-
ment of the descending tract of CN V), contralateral impairment of pain and temperature
sensation over the trunk and limbs (involvement of the lateral spinothalamic tract), ipsi-
lateral cerebellar ataxia (damage to spinocerebellar tracts), nystagmus, and the ocular tilt
reaction. In addition, patients with lateral medullary syndrome may have dysarthria, dys-
phagia, vertigo, persistent hiccups, lateropulsion, and ocular lateropulsion. A cavernous
sinus lesion, an internal carotid artery dissection, or a congenital brachial plexus injury
might cause Horner syndrome, but they would not cause the crossed impaired pain and
temperature sensation of the face and rest of the body.
21. c. Gaze preference is an inability to produce volitional gaze contralateral to the side of a
cerebral (supranuclear) lesion to the frontal eye field. It is accompanied by a tendency for
tonic deviation of the eyes toward the side of the lesion. The doll’s head maneuver gener-
ates a full range of horizontal eye movements because the vestibular-ocular pathways are
intact. Stroke is the most common etiology.
Gaze palsy is a symmetric limitation of the movements of both eyes in the same direc-
tion. Brainstem lesions producing a horizontal gaze palsy disrupt eye movements toward
the side of the lesion. CN VI nuclear lesions damage the final common site for supranu-
clear innervation of horizontal eye movements; thus, the doll’s head maneuver is ineffec-
tive in driving the paretic eyes toward the side of the lesion.
22. d. As described by Cogan, congenital ocular motor apraxia is idiopathic and is character-
ized by increased latency and intermittent failure of horizontal saccadic initiation. Vertical
eye movements are normal.
23. c. See-saw nystagmus is a form of disjunctive nystagmus in which 1 eye elevates and intorts
while the other eye depresses and extorts, movement reminiscent of that of a see-saw. The
eye movements are typically pendular, slow, and low frequency, as well as similar in ampli-
tude between the eyes. It is most commonly observed in patients with large tumors of the
parasellar region, which can compress the optic chiasm and cause bitemporal hemiano-
pia. The other scotomata and visual field defects would not be expected for a parasellar
lesion causing see-saw nystagmus.
24. b. There are 2 characteristic signs of infantile nystagmus syndrome: (1) reversal of the
normal pattern of OKN, characterized by slow-phase eye movements in the direction oppo-
site that of the rotating OKN drum, and (2) pattern in which the velocity of slow-phase
450 ● Answers
pupil is generally benign and does not raise concern for a compressive mass, aneurysm,
or dissection.
28. b. The patient is demonstrating light–near dissociation with brisk near pupillary response in
the absence of a good light response. The small, irregular pupils in this context raise the pos-
sibility of Argyll Robertson pupils, which occur in patients with tertiary syphilis involving
the central nervous system. Findings can be similar in widespread autonomic neuropathies,
including those due to diabetes and chronic alcoholism. In this case, testing for syphilis
should be performed. If the test result is positive, referral to a specialist in infectious diseases
for further evaluation and treatment should be made.
Observation is inappropriate because there is concern for syphilis in this case. Apra-
clonidine testing is used for diagnosis of a Horner pupil and is not indicated. Orbital
lesions are extremely unlikely to cause bilateral pupil findings of this sort.
29. d. Functional neuroimaging suggests that benign essential blepharospasm occurs because
of dysfunction of the basal ganglia. The site of dysfunction in hemifacial spasm is the
facial root exit zone in the cerebellopontine angle. Facial myokymia typically signifies
intramedullary disease of the pons involving the CN VII nucleus or fascicle. The midbrain
is not involved in eyelid closure.
30. c. Benign essential blepharospasm consists of episodic contraction of the orbicularis oculi
muscle. Onset usually occurs between ages 40 and 60 years. Initially, the spasms are mild and
infrequent, but they may progress to the point that the patient’s daily activities are severely
disrupted. Currently, the treatment of choice for benign essential blepharospasm is injection
of botulinum toxin into the orbicularis oculi muscle. Neuroradiologic studies are generally
unrevealing and rarely indicated. Benign essential blepharospasm can be exacerbated by
stress, but referral to psychiatry is typically not warranted. An acetylcholine receptor anti-
body test is used to diagnose MG, which does not cause forceful contracture of the facial
muscles.
31. b. Cerebral autosomal dominant arteriopathy with subcortical infarcts and leukoencepha-
lopathy (CADASIL) is an autosomal dominant angiopathy associated with a mutation in
NOTCH3 on chromosome 19. Presentation commonly occurs around age 30 years. Head-
aches that mimic migraine are common in patients with CADASIL. Recurrent lacunar
strokes, cognitive decline, and widespread leukoencephalopathy also characterize this con-
dition. Mutations in ABCA4 are associated with Stargardt disease; in OPA1, with dominant
optic atrophy; and in WFS1, with Wolfram (diabetes insipidus, diabetes mellitus, optic atro-
phy, deafness [DIDMOAD]) syndrome.
32. d. The constellation of bilateral vision loss, ophthalmoparesis, and ptosis indicates involve-
ment of multiple CNs as well as the optic nerves or chiasm in the pathologic process.
Tumors or skull base disorders can have all of these findings, either unilaterally or bilater-
ally. The sudden onset with headache should create suspicion for pituitary apoplexy, in
which rapid tumor expansion with hemorrhage causes compression of the optic chiasm
and/or optic nerves as well as CNs in the cavernous sinuses.
With a posterior communicating artery aneurysm, there would be unilateral findings
without vision loss, because the optic nerve is not compressed. Internal carotid artery dissec-
tion in the cavernous sinus can result in CN paresis without vision loss that is almost always
unilateral. Demyelinating disease presents in a less acute and rarely painful manner.
33. b. Unilateral headache with eye redness and tearing characterizes the trigeminal auto-
nomic cephalalgias, which include cluster headache, short-lasting unilateral neuralgiform
452 ● Answers
headache attacks with conjunctival injection and tearing (SUNCT) syndrome, and hemi-
crania continua. Autonomic dysfunction and Horner syndrome–like findings may accom-
pany the headache pain, and 1-mm to 2-mm eyelid ptosis may be seen. The pupil may be
miotic, not dilated (as may occur with migraine headache), and intraocular inflammation
and/or ONH swelling is not caused by these headaches.
34. b. Voluntary flutter, sometimes misdiagnosed as nystagmus, is characterized by irregular,
brief, fatigable bursts of rapid-frequency and low-amplitude eye movements with no slow
phase. Voluntary flutter is usually seen in patients with nonorganic motility disturbances.
End-gaze nystagmus has a fast and slow phase. Spasm of the near reflex consists of intermit-
tent episodes of excess convergence, increased accommodation, and pupillary constriction.
See-saw nystagmus is a form of disjunctive nystagmus in which one eye elevates and intorts
while the other eye depresses and extorts, movement reminiscent of that of a see-saw.
35. b. While all the listed techniques may help uncover functional vision loss in a patient
claiming monocular no light perception vision, only the fogging test can demonstrate
20/20 vision OD with suspected nonorganic vision loss. To perform the fogging test, the
clinician uses a trial frame with plus and minus cylinder lenses (6.00 D), the axes of which
are parallel; these are placed in front of the patient’s “good” eye. The patient is then asked
to read the Snellen chart while 1 of the cylinder lenses is rotated. The cylinder rotation
will severely blur vision as the 2 axes are rotated out of alignment. If the patient continues
to read, he or she is doing so with the “bad” eye. The fogging test is a form of confusion
testing, which requires the patient to be unaware of which eye is being tested.
The mirror test can demonstrate at least light perception vision but would not help
prove that the patient has 20/20 vision. The generation of eye movement with rotation
of the OKN drum indicates 20/400 vision but cannot determine whether the vision is
any better than 20/400. A base-out prism test showing eye movement when the prism
is placed over the “bad” eye indicates some vision in that eye, but it cannot prove 20/20
vision.
36. a. The disparity between visual acuity and examination result raises suspicion for nonor-
ganic vision loss. In Goldmann manual kinetic perimetry, the visual field is tested continu-
ously in a clockwise or counterclockwise direction, starting with the (smallest) I4e stimulus.
A common functional response shows a spiraling isopter getting closer and closer to fixation
as testing continues. As larger stimuli (III4e and V4e) are employed, there is often further
constriction, resulting in overlapping isopters.
A step across the vertical or nasal horizontal midline may indicate that neurologic or
ophthalmic disease is contributing to the visual field abnormality. Central scotomata may
occur in many disease processes, such as toxic or nutritional optic neuropathy or LHON,
and are not suggestive of functional disease. While generalized visual field constriction
can occur with nonphysiologic vision loss, it also can occur in disease processes such as
retinitis pigmentosa. Generalized constriction does not confirm functional vision loss.
37. d. The number of relapses during corticosteroid tapering may be reduced with use of tocili-
zumab, an interleukin-6 receptor alpha inhibitor. Corticosteroid adverse effects can be
problematic in any case, but this woman is at particularly high risk because of her under-
lying conditions (diabetes, osteoporosis); thus, tocilizumab treatment should be considered
to allow for tapering to lower corticosteroid doses. Eculizumab, a recombinant humanized
monoclonal antibody against the complement protein C5, is used in the treatment of neuro-
myelitis optica spectrum disorder. Rituximab, a monoclonal antibody against CD20, can be
Answers ● 453
used for the treatment of MG, neuromyelitis optica spectrum disorder, and other neoplastic
and autoimmune diseases. Teprotumumab, a humanized monoclonal antibody that inhibits
the insulin-like growth factor 1 receptor on orbital fibroblasts, is used to treat thyroid eye
disease.
38. b. This vision loss is most consistent with bilateral optic neuritis. The bilateral nature of
the vision loss, the severity of visual impairment, and the minimal recovery after treat-
ment raise a concern for neuromyelitis optica. It is uncommon for MS to present with
bilateral optic neuritis with minimal recovery. GCA can cause bilateral severe vision loss
associated with headache, but it does not affect young patients. Acute zonal occult outer
retinopathy is a retinal disorder resulting in vision loss associated with photopsias but no
optic nerve enhancement on MRI.
39. c. Altered mental status in the presence of a hemianopic visual field defect raises concern
for progressive multifocal leukoencephalopathy, a condition associated with natalizumab
use in patients who are John Cunningham (JC)–virus positive. The other medications
have either not been associated or been associated with encephalopathy only in rare cases.
Fingolimod can cause macular edema.
40. d. Ocular neuromyotonia, in which episodic diplopia lasting 30–60 seconds develops after
sustained activation of a cranial ocular motor nerve, is usually associated with a history of
radiation therapy for a skull base neoplasm.
Study Questions
Please note that these questions are not part of your CME reporting process. They are provided
here for your own educational use and for identification of any professional practice gaps. The
required CME posttest is available online (see “Requesting Continuing Medical Education
Credit”). Following the questions are answers with discussions. Although a concerted effort
has been made to avoid ambiguity and redundancy in these questions, the authors recognize
that differences of opinion may occur regarding the “best” answer. The discussions are pro-
vided to demonstrate the rationale used to derive the answer. They may also be helpful in con-
firming that your approach to the problem was correct or, if necessary, in fixing the principle
in your memory. The Section 6 faculty thanks the Resident Self-Assessment Committee for
developing these self-assessment questions and the discussions that follow.
1. In testing visual acuity in preliterate children, what optotype has the best calibration and
reliability?
a. HOTV
b. Tumbling E
c. Allen figures
d. Lighthouse chart
2. The lateral muscular branch of the ophthalmic artery supplies which extraocular muscle
(EOM)?
a. superior rectus
b. medial rectus
c. inferior rectus
d. inferior oblique
3. In right gaze, the innervation to the right lateral rectus will increase. What determines the
amount of innervation to the right medial rectus in this situation?
a. arc of contact
b. Sherrington’s law
c. vergence amplitudes
d. recruitment
4. What are the primary synergistic (yoke) muscles that are used for gazing up and to the
right?
a. left inferior oblique and right superior oblique
b. left superior rectus and right inferior oblique
c. left inferior oblique and right superior rectus
d. left superior oblique and right superior rectus
451
452 ● Study Questions
5. In a complete cranial nerve (CN) III palsy, what action of the superior oblique muscle
exacerbates the outward position of the eye?
a. primary
b. secondary
c. tertiary
d. quaternary
6. A parent brings a child to the clinic for evaluation of strabismus. What finding on exami-
nation would suggest that the strabismus is long-standing?
a. incomitant deviation
b. diplopia
c. anomalous retinal correspondence (ARC)
d. normal head position
8. To test for a horizontal deviation using the Maddox rod, in what direction should the
cylinders be placed in front of the right eye?
a. 15° meridian
b. 45° meridian
c. 90° meridian
d. 180° meridian
10. A 12-year-old boy is diagnosed with well-controlled, intermittent exotropia. What is ste-
reopsis testing most likely to reveal?
a. monofixation syndrome
b. no stereopsis because of diplopia
c. excellent stereopsis
d. no stereopsis because of temporal hemiretinal suppression
Study Questions ● 453
11. A 6-year-old child presents with a 35 prism diopters (Δ) intermittent exotropia at dis-
tance and a 20Δ intermittent exotropia at near. After the patch test, measurements for this
patient are 35Δ exotropia at distance and 30Δ exotropia at near. What is the most likely
diagnosis?
a. pseudodivergence excess intermittent exotropia
b. divergence excess intermittent exotropia
c. intermittent exotropia with a high accommodative convergence/accommodation (AC/A)
ratio
d. basic intermittent exotropia
12. What is the most appropriate surgery to perform for a V-pattern esotropia without over-
elevation in adduction?
a. bilateral medial rectus recession with inferior transposition (inferoplacement)
b. bilateral medial rectus recession with superior transposition (superoplacement)
c. bilateral lateral rectus resection with inferior transposition (inferoplacement)
d. unilateral medial rectus recession and lateral rectus resection with inferior transposi-
tion (inferoplacement)
13. What pattern strabismus would be most successfully addressed with bilateral weakening
of the inferior oblique muscles?
a. A pattern
b. V pattern
c. X pattern
d. Y pattern
14. A patient undergoes surgery to repair an orbital floor fracture and free an entrapped in-
ferior rectus muscle. The surgery is technically successful. Postoperatively, a hypotropia is
present in that eye and persists for 6 months following the surgery. What surgery is most
likely to provide benefit at this time?
a. superior rectus resection
b. inferior rectus recession
c. re-exploration of the fracture
d. transposition procedure
16. What intracranial structural abnormality could most likely account for new-onset down-
beat nystagmus in a 4-year-old girl?
a. pinealoma
b. chiasmal tumor
c. craniopharyngioma
d. Arnold-Chiari malformation
17. A 6-year-old child undergoes bilateral 5-mm medial rectus recession for 35Δ esotropia.
One week after surgery, there is a measurement of 30Δ exotropia. What is the most ap-
propriate test to perform next?
a. test of EOM movements
b. measurement of stereopsis
c. measurement of vertical deviations
d. slit-lamp examination
19. The results of a cycloplegic retinoscopy in an other wise healthy, asymptomatic 3-year-old
child are likely to reveal what approximate refractive error?
a. 0.00–1.00 diopters (D) myopia
b. 0.00–1.00 D hyperopia
c. 2.00–3.00 D hyperopia
d. 2.00–3.00 D myopia
20. A 4-month-old boy does not fix or follow but has an other wise normal eye examination.
The child is reexamined when he is 6 months old and is noted to have normal responses
to visual stimuli. What is the most likely diagnosis?
a. cerebral visual impairment (CVI)
b. delayed visual maturation (DVM)
c. intrauterine infection
d. uncorrected high refractive error
21. For a 5-month-old child with poor visual behavior but no nystagmus, what diagnosis is
likely?
a. retinal dystrophy
b. CVI
c. optic nerve hypoplasia
d. congenital cataract
Study Questions ● 455
22. For a child with right severe congenital ptosis and no ipsilateral levator function, what is
the most appropriate method of ptosis repair?
a. blepharoplasty
b. levator resection
c. frontalis suspension
d. brow lift
24. Crouzon syndrome differs from Apert syndrome in that the latter is associated with what
finding?
a. hypertelorism
b. proptosis
c. significant syndactyly
d. inferior scleral show
25. What is the most common type of strabismus associated with craniosynostosis?
a. A-pattern esotropia
b. V-pattern esotropia
c. V-pattern exotropia
d. A-pattern exotropia
26. A 4-day-old infant presents with chemosis, significant discharge, and corneal ulceration.
What is the most likely diagnosis?
a. chemical conjunctivitis
b. herpes simplex virus conjunctivitis
c. Neisseria gonorrhoeae conjunctivitis
d. chlamydial conjunctivitis
28. A child with new-onset ptosis should be examined for what other potentially associated
finding?
a. cataract
b. anisocoria
c. dacryostenosis
d. corneal haze
29. In patients with aniridia, what is the most common cause of progressive corneal scarring?
a. endothelial decompensation
b. exposure keratopathy
c. limbal stem cell deficiency
d. herpes simplex keratitis
30. A 6-week-old boy presents with unilateral epiphora and ocular irritation. The cornea is
enlarged but clear. The intraocular pressure (IOP) is 38 mm Hg. What is the preferred
initial treatment?
a. Ahmed valve placement
b. cycloablation via diode laser
c. trabeculectomy
d. goniotomy
31. What is the preferred laboratory testing for a child with a unilateral congenital cataract?
a. TORCH (toxoplasmosis, other agents, rubella, cytomegalovirus, herpes simplex) titers
b. serum for calcium and phosphorus
c. no testing (typically none is needed)
d. urine for amino acids
32. Children with optic nerve hypoplasia should undergo magnetic resonance imaging (MRI)
to evaluate for what finding?
a. bifid septum pellucidum
b. empty sella
c. ectopic posterior pituitary bright spot
d. chiasmal glioma
33. What growth pattern of retinoblastoma is most likely to be mistaken as childhood uveitis?
a. diffuse infiltrating
b. retinocytoma
c. unilateral multifocal
d. unilateral unifocal
Study Questions ● 457
34. A 6-month-old child has poor vision with nystagmus and eye-poking. The retinal appear-
ance is normal. What is the best test to evaluate this patient?
a. MRI
b. fluorescein angiography
c. A-scan ultrasonography
d. electroretinography
35. An infant is diagnosed with type 1, prethreshold retinopathy of prematurity (ROP). What
is the best step in management, according to the Early Treatment for Retinopathy of Pre-
maturity (ETROP) study?
a. observation
b. cryotherapy to ablate the peripheral retina
c. panretinal laser photocoagulation to ablate the peripheral retina
d. intravitreal injection of steroid
36. What test is most likely to determine the etiology of anterior uveitis in a child?
a. antinuclear antibodies
b. rheumatoid factor
c. human leukocyte antigen B27 (HLA-B27)
d. Lyme serology
39. The greatest risk of rebleeding in traumatic hyphema occurs how long after initial injury?
a. 3–7 days
b. 8–12 days
c. 13–17 days
d. 18–25 days
458 ● Study Questions
40. What is the most common ocular manifestation of abusive head trauma (AHT)?
a. corneal abrasion
b. hyphema
c. subconjunctival hemorrhage
d. retinal hemorrhage
41. What condition is associated with an abnormally high number of crossed fibers in the
optic chiasm?
a. ocular albinism
b. optic nerve hypoplasia
c. achromatopsia
d. morning glory disc anomaly
42. A 4-year-old boy presents with salt-and-pepper fundus and cataracts in both eyes. What
congenital infection did he most likely have?
a. rubella
b. toxoplasmosis
c. toxocariasis
d. syphilis
43. A child has uncontrolled glaucoma due to encephalofacial angiomatosis (also known as
Sturge-Weber syndrome). If glaucoma surgery is needed, what is a particular intraopera-
tive risk?
a. choroidal hemorrhage
b. hyphema
c. exposure of tube under conjunctiva
d. leakage from trabeculectomy flap
459
460 ● Answers
9. b. A comitant deviation is one in which the size of the deviation does not vary by more
than a few prism diopters (Δ) in different positions of gaze or with either eye used for
fixating. Of the choices provided, the best answer is infantile esotropia. A restrictive orbi-
topathy (such as nasal wall fracture with entrapment or thyroid orbitopathy) or a paretic
process (such as CN VI palsy) would lead to an incomitant deviation, in which the mea-
surement varies significantly in different positions of gaze.
10. c. Intermittent exotropia is an outward deviation that becomes manifest during times of
visual inattention, fatigue, or stress. In the early stages of the disorder, the deviation is
usually larger for distance viewing than for near. Thus, sensory testing at near is typically
excellent, including stereopsis and fusion on Worth 4-dot testing. When the eye deviates,
suppression of the image typically occurs, rather than the recognition of diplopia.
11. a. When the exodeviation at distance is larger than the deviation at near fixation by 10Δ
or more but the distance and near measurements become similar after 30–60 minutes
of binocular occlusion using a patch (the patch test), pseudodivergence excess intermit-
tent exotropia is the likely diagnosis. In true divergence excess intermittent exotropia, the
distance–near discrepancy would persist after patch testing. A patient with distance–near
discrepancy may have a high accommodative convergence/accommodation (AC/A) ratio,
which would be defined by persistence of the distance–near discrepancy after the patch
test but resolution of this discrepancy with a +3.00 add. In a patient with basic intermit-
tent exotropia, the measurement of the deviation at distance would be within 10Δ of the
deviation measurement at near without patch testing or a +3.00 add.
12. a. In a V-pattern esotropia, the medial rectus muscles can be recessed and moved toward
the apex of the deviation, which would be inferiorly in this case. Lateral muscles should be
moved away from the apex of the deviation. A useful mnemonic is MALE: medial rectus
muscle to the apex, lateral rectus muscle to the empty space. When horizontal muscle
recession-resection is performed, displacement should be in opposite directions, but this
could cause symptomatic torsion.
13. b. Inferior oblique overaction commonly causes a V pattern. A patterns are most com-
monly caused by superior oblique overaction. X and Y patterns result from pseudo-
overaction of the inferior oblique muscles; weakening them would not collapse the
pattern.
14. b. Despite the repair of the orbital floor and release of the entrapped muscle, restriction
can persist, so re-exploration of the fracture will not be of benefit. In these cases, recession
of the restricted muscle is the next step. An inferior rectus recession is indicated in this
case. A superior rectus resection may be done if the inferior rectus recession results in
undercorrection. A transposition could be considered for a muscle palsy, but an inferior
rectus muscle palsy would result in a hypertropia, not a hypotropia.
15. a. Type 1, the most common form of Duane syndrome, is classically characterized by poor
abduction and esotropia. A patient may adopt a head turn to establish binocular fusion,
and the esotropia may be seen only in primary gaze or with a head turn to the opposite
direction. Type 2 is classically characterized by limited adduction and exotropia. Type 3 is
characterized by poor abduction and adduction; either esotropia or exotropia may result.
16. d. Acquired downbeat nystagmus can be secondary to Arnold-Chiari malformation.
Pinealoma is associated with convergence-retraction nystagmus. Chiasmal tumors have
been associated with spasmus nutans, a triad of nystagmus, head bobbing, and torticollis.
Craniopharyngioma is associated with see-saw nystagmus.
Answers ● 461
17. a. The extreme overcorrection suggests that there is a slipped medial rectus muscle. This
would be evident by limited adduction in either eye. In this patient, absence of stereopsis
would be expected but not helpful diagnostically. Vertical measurements would not be
helpful, nor would findings from a slit-lamp examination.
18. d. Dysraphism represents a failure to fuse. Agenesis represents developmental failure, as is
found in anophthalmia, whereas hypoplasia is due to developmental arrest. Developmen-
tal excess such as that observed with distichiasis is the cause of hyperplasia.
19. c. A 3-year-old child is expected to have some hyperopia, approximately between 2.00 and
3.00 diopters (D). Infants are hyperopic and can become slightly more hyperopic until ap-
proximately 6–8 years of age. Then the refraction shifts toward plano until approximately
age 16, during the process of emmetropization. If myopia presents before age 10 years,
later high myopia (6.00 D or more) has a greater likelihood of developing. Low amounts
of astigmatism, typically with the rule, are common in infants and can resolve. Large am-
etropias or anisometropias can be amblyogenic.
20. b. The term delayed visual maturation (DVM) describes a condition wherein normal fixa-
tion and tracking do not develop within the first 3–4 months of life, but visual behavior
subsequently normalizes. Cerebral visual impairment (CVI) is caused by pathology pos-
terior to the lateral geniculate nucleus. CVI can be congenital or acquired. Causes include
structural central nervous system abnormalities, intrauterine infection, periventricular
leukomalacia, hypoxia, hydrocephalus, abusive head trauma, and encephalitis. Although
some improvement in vision can occur, a complete normalization of vision is uncom-
mon. Uncorrected refractive error and intrauterine infection would not be associated with
spontaneous normalization of vision.
21. b. When vision loss is due to pathology posterior to the lateral geniculate nucleus, nystag-
mus is generally not present.
22. c. As long as there is adequate levator function, levator resection is an appropriate option
for congenital ptosis treatment; in this case, levator resection would fail. Neither blepha-
roplasty nor brow lift would be successful—congenital ptosis is not typically associated
with dermatochalasis or brow ptosis. In this case, a frontalis suspension is the appropri-
ate procedure. Harvesting autologous fascia lata may be difficult in very young children.
Many surgeons would utilize either suture (prolene, nylon) or a silicone rod to temporar-
ily elevate the eyelid and proceed with a fascial sling once the child is older. Some surgeons
would opt for banked fascia lata.
23. a. Blepharophimosis–ptosis–epicanthus inversus syndrome may occur as a sporadic or
autosomal-dominant disorder. It consists of blepharophimosis, congenital ptosis, telecan-
thus, and epicanthus inversus. Surgery may be necessary early in life.
24. c. Crouzon and Apert syndromes appear similar clinically. However, Apert syndrome is
associated with an extreme amount of syndactyly. Both syndromes are associated with
hypertelorism, proptosis, and inferior scleral show.
25. c. Patients with craniosynostoses can have various types of horizontal strabismus. V-pattern
exotropia is the most common type.
26. c. Ophthalmia neonatorum caused by Neisseria gonorrhoeae typically presents in the first
3–4 days of life. In severe cases, it is associated with marked chemosis, significant dis-
charge, and a risk of corneal perforation. Chemical conjunctivitis is a mild, self-limited
inflammation occurring in the first 24 hours of life due to instillation of silver nitrate.
462 ● Answers
Herpes simplex virus conjunctivitis usually presents later, often in the second week of
life. Chlamydial conjunctivitis usually occurs around 1 week of age and is associated with
minimal to moderate discharge and possible pseudomembrane formation.
27. a. Stevens-Johnson syndrome is a hypersensitivity reaction that affects the skin and mu-
cous membranes. Early ocular involvement may range from conjunctivitis to corneal per-
foration. Symblepharon, corneal vascularization, and eyelid anomalies such as entropion
are later sequelae.
28. b. Horner syndrome caused by a lesion along the oculosympathetic pathway must be ruled
out in a patient with new-onset ptosis. Affected patients have anisocoria that is greater in
dim light and ptosis secondary to paralysis of the Müller muscle. In a child, new-onset
Horner syndrome can be idiopathic or secondary to trauma, surgery, or neuroblastoma
along the sympathetic chain in the chest. Cataract, dacryostenosis, and corneal haze are
not associated with acute ptosis.
29. c. Classic aniridia is autosomal dominant, associated with a pathogenic variant in the PAX6
gene. Large deletions of the gene are typically sporadic and can confer a risk for Wilms
tumor of the kidney. Patients with congenital aniridia can develop progressive corneal scar-
ring within the first decade of life, necessitating corneal transplantation to restore vision.
Adult patients with aniridia can also develop later-onset issues with nonhealing epithelial
defects and corneal neovascularization. The major underlying mechanism is related to lim-
bal stem cell deficiency, although abnormal cell differentiation may play a role as well.
30. d. Congenital glaucoma can present with epiphora and ocular irritation. The initial treat-
ment is surgical, with goniotomy or trabeculotomy preferred for the initial treatment. Pri-
mary trabeculectomy has a high failure rate and is contraindicated as primary treatment.
Both Ahmed valve placement and cycloablation are reserved for secondary treatments.
31. c. Unilateral cataracts are not usually associated with occult systemic or metabolic disease;
therefore, laboratory tests are not warranted. Note that bilateral cataracts can have signifi-
cant asymmetry. In cases of bilateral cataracts, the history and physical examination guide
whether and which laboratory investigations are indicated. A complete ocular evaluation
is needed to rule out a secondary ocular cause.
32. c. Optic nerve hypoplasia is characterized by a decreased number of optic nerve axons.
It is associated with absence of the septum pellucidum, not a bifid septum pellucidum;
with agenesis of the corpus callosum; and with pituitary gland abnormalities. Magnetic
resonance imaging (MRI) reveals an ectopic posterior pituitary bright spot at the upper
infundibulum. This finding is associated with pituitary hormone deficiencies. Optic nerve
hypoplasia is not associated with an empty sella on MRI. Chiasmal gliomas are associated
with optic nerve elevation, not optic nerve hypoplasia.
33. a. Diffuse infiltrating retinoblastoma can mimic childhood uveitis. It typically presents in
older children (5–6 years old). Retinocytoma is a spontaneously regressed retinoblastoma.
34. d. Early-onset inherited retinal disorders (IRDs) are genotypically and phenotypically het-
erogeneous. Because the retinal appearance is sometimes normal in early-onset IRDs, signs
and symptoms are particularly important to recognize. For early-onset IRDs, nystagmus
is one of the most common presenting signs, typically occurring between 8 and 12 weeks
of age. In addition to poor vision, symptoms include nyctalopia, photodysphoria, and eye-
poking (oculodigital sign). Signs include paradoxical pupil and high refractive error. An
electroretinogram is typically used to make the diagnosis, showing extinguished waveforms.
Answers ● 463
A-scan ultrasonography, fluorescein angiography, and MRI would not be helpful in making
the diagnosis.
35. c. Prethreshold retinopathy of prematurity (ROP) is classified into 2 types. Type 1 in-
cludes zone I, any stage ROP with plus disease; zone I, stage 3 ROP, without plus disease;
and zone II, stage 2 or 3 ROP, with plus disease. The Early Treatment for Retinopathy
of Prematurity (ETROP) trial found that earlier treatment at prethreshold resulted in
better structural and visual outcomes when compared with conventional treatment at
threshold. On the basis of ETROP, laser treatment is recommended for any eye with
type 1 ROP.
36. a. The most common identifiable etiology of anterior uveitis in a child is juvenile idio-
pathic arthritis, which occurs in 15%–47% of cases. Antinuclear antibody positivity is
typically present in these children; rheumatoid factor is typically negative. Although ju-
venile spondyloarthropathies are associated with anterior uveitis in children, they are less
common; thus, human leukocyte antigen B27 (HLA-B27) testing is less likely to be ben-
eficial. Infectious etiologies of anterior uveitis in children are less common, and results of
serologic testing are less likely to reveal the etiology in these patients.
37. a. Tubulointerstitial nephritis and uveitis (TINU) syndrome is kidney disease associ-
ated with chronic or recurrent anterior uveitis in adolescents; the median age at onset is
15 years. The renal disease is characterized by low-grade fever, fatigue, pallor, and weight
loss. The uveitis is usually bilateral and may occur before, simultaneously with, or after
the renal disease. TINU nephritis is not commonly associated with cataract, serous retinal
detachment, or papilledema.
38. a. Although trauma is the most common cause of hyphema in children, spontaneous hy-
phemas can occur in juvenile xanthogranuloma of the iris, retinoblastoma, leukemia, and
blood dyscrasias. Iris nevus, iris capillary hemangioma, and iris cyst are not associated
with spontaneous hyphema formation.
39. a. One of the risks of hyphema is rebleeding, with worsening symptoms. This is especially
critical in children, who often do not adhere to activity restrictions. The greatest risk for
rebleeding occurs within 3 to 7 days. After 7 days, the risk progressively decreases.
40. d. The most common ocular manifestation of abusive head trauma (AHT), present in
approximately 80% of cases, is retinal hemorrhage. These hemorrhages can be seen in all
layers of the retina and may be unilateral or bilateral. They are found most commonly in
the posterior pole but can also be present peripherally. The anterior segment and ocular
adnexa are often normal. Hyphema, corneal abrasion, and subconjunctival hemorrhages
are less commonly seen in AHT.
41. a. Ocular albinism is associated with iris transillumination, foveal aplasia or hypopla-
sia, and decreased retinal pigmentation. Both human and animal studies have shown an
abnormally high number of crossed fibers in the optic chiasm, which precludes stere-
opsis and is associated with strabismus. Optic nerve hypoplasia and morning glory disc
anomaly are not associated with abnormalities in the crossed fibers of the optic chiasm.
Achromatopsia is not typically associated with optic nerve abnormalities.
42. a. Congenital rubella syndrome is caused by rubella virus that is transmitted transpla-
centally to a fetus. Eye findings include a variable pigmentary retinopathy, ranging from
a salt-and-pepper appearance to pseudoretinitis pigmentosa. In this case, the patient has
macular pigmentary changes as well as peripheral pigmentary changes. Other eye findings
464 ● Answers
2. What sensory nerve enters the orbit through the superior orbital fissure?
a. optic nerve
b. maxillary division of cranial nerve V
c. facial nerve
d. ophthalmic division of cranial nerve V
3. What is the most common ocular finding in an infant with a unilateral small orbit and no
visible eye?
a. microphthalmia
b. anophthalmia
c. eyelid coloboma
d. blepharophimosis
345
346 ● Study Questions
5. A 58-year-old man with poorly controlled diabetes mellitus presents to the emergency
department with severe orbital pain, decreased vision, and proptosis. A computed tomog-
raphy (CT) scan shows fulminant sinusitis with orbital invasion. A biopsy is performed,
and Grocott-Gomori methenamine silver stain (GMS) shows septate branching hyphae of
uniform width. What is the most likely diagnosis?
a. mucormycosis
b. echinococcosis
c. aspergillosis
d. cysticercosis
7. A 45-year-old woman presents with axial proptosis and vision loss. On examination, her
affected eye also has optic atrophy and an optociliary shunt vessel. Her MRI scan reveals
enhancement of the optic nerve sheath around a darker, normal optic nerve. What is the
most likely diagnosis?
a. thyroid eye disease (TED)
b. optic nerve sheath meningioma
c. optic nerve glioma
d. sphenoid wing meningioma
9. Why are orbital roof fractures more likely to occur in young children than in adults?
a. There is better pneumatization of frontal sinuses in young children.
b. There is overprojection of the midface in young children.
c. There is a larger cranial vault-to-midface ratio in young children.
d. The frontal lobe is underdeveloped in young children.
Study Questions ● 347
10. A patient arrives in the trauma bay after a motor vehicle crash in which he hit his face on
the dashboard. He has ecchymosis and significant swelling to his right eye. The eyelids are
“rock hard” to the touch. His visual acuity is 20/200 OD and 20/20 OS. He has a right af-
ferent pupillary defect and limited extraocular movements of the right eye. IOP is 45 OD
and 22 OS. What is the most appropriate next step?
a. Perform an immediate lateral canthotomy and inferior cantholysis.
b. Obtain CT of the orbits.
c. Start maximum topical ocular antihypertensive eyedrops and recheck the pressure in
30 minutes.
d. Perform orbital exploration in the operating room.
11. What is the best approach to an intraconal orbital tumor located between the optic nerve
and the lateral rectus?
a. retrocaruncular orbitotomy
b. lateral orbitotomy
c. medial orbitotomy
d. vertical eyelid-splitting orbitotomy
12. Six months after enucleation, a patient returns to the office reporting mucoid discharge
from the anophthalmic side. There is no pain. She states that the prosthesis fits well. What
is the most likely cause of the mucus discharge?
a. giant papillary conjunctivitis
b. implant extrusion
c. implant infection
d. lower eyelid entropion
13. When one is operating in the region of the temporal fossa, in what tissue layer is the tem-
poral (frontal) branch of cranial nerve VII found?
a. deep temporal fascia
b. temporoparietal fascia
c. temporal fat pad
d. subcutaneous tissue
14. What is the term for an extra row of eyelashes that emerge from the meibomian glands?
a. trichiasis
b. madarosis
c. poliosis
d. distichiasis
348 ● Study Questions
16. A patient with a history of “eyelid cancer” in the past now presents with ipsilateral forehead
pain, diplopia, and ptosis. What condition is most likely responsible for these symptoms?
a. Bowen disease
b. squamous cell carcinoma
c. basal cell carcinoma
d. Muir-Torre Syndrome
17. What is the most common form of basal cell carcinoma (BCC)?
a. morpheaform
b. nodular
c. multicentric
d. basosquamous
18. A 33-year-old man arrives in the emergency department with blunt trauma to the right
eyelid. He has a 1.5-cm horizontal laceration of the upper eyelid with prolapse of fat at the
site of injury. What is the most important structure to evaluate prior to repair of the eyelid
laceration?
a. tarsal conjunctiva
b. extraocular muscles
c. globe
d. levator aponeurosis
20. During patient evaluation, the ophthalmologist attempts to digitally elevate the margin
of the lower eyelid to the superior corneal limbus with the eye in primary position. The
inability to perform this maneuver suggests what diagnosis?
a. spastic ectropion
b. involutional entropion
c. involutional ectropion
d. cicatricial ectropion
Study Questions ● 349
21. What procedure that is used to correct punctal ectropion involves a horizontal fusiform
excision of conjunctiva and eyelid retractors 4 mm inferior to the puncta?
a. lateral tarsal strip
b. punctoplasty
c. medial spindle
d. rotational suture repair
22. Lower eyelid retractor repair (reattachment) is most appropriate for correction of what
condition?
a. entropion without significant horizontal laxity
b. lower eyelid retraction
c. cicatricial entropion
d. mechanical ectropion
23. A patient has eyelid malposition. The ophthalmologist recommends mucosal grafting
with additional surgical repair. What eyelid malposition is most likely to lead to this rec-
ommendation?
a. cicatricial ectropion
b. cicatricial entropion
c. spastic entropion
d. involutional ectropion
24. In a patient with asymmetric ptosis, how is ptosis unmasked in the contralateral eye?
a. careful measurement of the margin–reflex distance (MRD)
b. instillation of phenylephrine 2.5%
c. lifting of the more ptotic eyelid
d. ice-pack test
25. Middle lamellar deficiency most commonly results from surgical or traumatic shortening
of what structure?
a. orbicularis muscle
b. orbital septum
c. conjunctival and eyelid retractors
d. tarsal plate
26. When an endoscopic brow lift is performed, the incision in each temporal region can
result in what complication?
a. damage to cranial nerve (CN) VII
b. damage to CN V
c. eyelid ptosis
d. Horner syndrome
350 ● Study Questions
27. Darkly pigmented skin increases the risk of what complication of laser skin resurfacing?
a. lagophthalmos
b. postoperative inflammatory hyperpigmentation
c. lower eyelid retraction
d. herpes simplex virus outbreak
28. What bone forms the anterior portion of the nasolacrimal fossa?
a. ethmoid bone
b. lacrimal bone
c. maxillary bone
d. palatine bone
30. A 2-year-old child is brought into the clinic because the mother notes tears constantly
running down the child’s face. There is no reflux on palpation of the lacrimal sac and there
is no increased tear film height. There is a small tuft of eyelashes a few millimeters lateral
to and above the eyelash line. What is the proper management?
a. probing of the puncta and canaliculi
b. probing and intubation of the lacrimal system
c. excision of aberrant ductules
d. observation
Answers
1. b. The lateral canthal tendon attaches at the lateral orbital tubercle, also known as the
Whitnall tubercle. The Whitnall ligament is a separate fascial band that does not insert on
the tubercle of the same name. The lateral orbital tubercle is posterior to the orbicularis
oculi muscle and lateral orbital rim.
2. d. The ophthalmic division of cranial nerve V enters the orbit through the superior orbital
fissure. The optic nerve enters through the optic canal, and the maxillary division of cra-
nial nerve V enters through the foramen rotundum. The facial nerve is a motor nerve and
does not enter the orbit.
3. a. Microphthalmia is defined by the presence of a small eye with an axial length that is at
least 2 standard deviations below the mean axial length for age. Microphthalmia is much
more common than anophthalmia, and the vast majority of microphthalmic eyes have
no potential for vision. Although no visible eye may be present, most of these patients
actually have a microphthalmic globe. The defect can be isolated or occur with a constel-
lation of abnormalities as part of a syndrome. An eyelid coloboma is a characterized by
a congenital defect of the eyelid margin, and blepharophimosis is characterized by eyelid
malpositions in the presence of a normal orbit.
4. b. Microphthalmia with orbital cyst results from failure of the choroidal fissure to close in
the embryo. Craniofacial clefts occur as a result of a developmental arrest or mechanical
disruption of development. Etiologic theories include a failure of neural crest cell migra-
tion and a failure of fusion or movement of facial processes. Anophthalmia occurs when
the primary optic vesicle fails to grow out from the cerebral vesicle at the 2-mm stage of
embryonic development. Consecutive anophthalmia is thought to result from a secondary
degeneration of the optic vesicle.
5. c. The clinical scenario described is a classic presentation of acute invasive aspergillo-
sis. On Grocott-Gomori methenamine silver (GMS) stain, Aspergillus is characterized by
branching, septate hyphae. Mucormycosis also would be on the differential diagnosis of
acute sino-orbital disease in a patient with poorly controlled diabetes. However, GMS
stain demonstrates broad, nonseptate hyphae in mucormycosis. Echinococcosis is caused
by a dog tapeworm; it can sometimes cause progressive inflammation and a severe im-
mune response if there is rupture of an orbital cyst. It is not associated with sinus disease.
Cysticercosis is caused by the Taenia solium parasite and may rarely cause orbital infec-
tion. The enzyme-linked immunosorbent assay (ELISA) for serum antibodies may aid in
diagnosis.
6. d. Cavernous venous malformations (CVMs; previously known as cavernous hemangio-
mas) are common orbital lesions of young to middle-aged adults, with a female predilec-
tion. On magnetic resonance imaging (MRI), a characteristic heterogeneous enhancement
pattern is often seen during initial gadolinium infusion that becomes homogeneous as the
study progresses; this feature greatly narrows the differential diagnosis. In atypical cases,
specific MRI and magnetic resonance angiography (MRA) sequences may be very helpful
in differentiating these lesions from active venolymphatic or arteriovenous malformations,
usually abrogating the need for and inherent risks of cerebral arteriography. Computed
tomography has less-specific patterns: longstanding lesions may exhibit areas of calcifica-
tion. Although cavernous venous malformations usually grow slowly, the presence of a
351
352 ● Answers
compressive optic neuropathy typically requires intervention. Although some patients with
early optic neuropathy may opt for observation to at least temporarily avoid the risks of
treatment, surgical intervention should be offered. While stereotactic radiotherapy, includ-
ing gamma knife radiosurgery, may be an option for lesions that are difficult to access in
the orbital apex, anterior or mid-orbital lesions typically do well with excisional surgery.
Observation with serial examinations and imaging is certainly a reasonable option if the
CVM is found incidentally and the patient is asymptomatic.
7. b. Optic nerve meningiomas arise in the arachnoid of the optic nerve sheath and en-
large to cause compression of the optic nerve. They occur most commonly in women in
the third and fourth decades of life. Pertinent clinical features include gradual, painless,
unilateral loss of vision. There can be axial proptosis and the nerve can appear normal,
swollen, or atrophic. Optociliary shunt vessels may be present. Optic nerve sheath menin-
giomas may be associated with neurofibromatosis type 2, especially if bilateral. Enhance-
ment of the lesion surrounding the optic nerve results in the “tram-track” sign on axial
imaging or a doughnut appearance on coronal sections. Optic nerve gliomas usually show
fusiform enlargement of the optic nerve. Thyroid eye disease (TED) does not usually pre-
sent as described. Sphenoid wing meningiomas arise from the meninges and originate in-
tracranially and extend into the orbit. Imaging may show hyperostosis and calcifications,
along with dural enhancement on MRI.
8. c. Orbital pulsations can occur in various situations and can often be divided, depending
on the presence or absence of a bruit. With bruits, pulsations can be seen in carotid cavern-
ous fistulas, orbital arteriovenous fistulas, and dural arteriovenous fistulas. Without bruits,
they can be found in meningoencephaloceles, neurofibromatosis 1 (NF1), and in cases
involving orbital roof defects, such as removal of the orbital roof. Sphenoid wing dysplasia
or aplasia in NF1 is associated with this orbital finding. Patients with NF1 with pulsatile
proptosis have sphenoid wing dysplasia or aplasia. Tuberous sclerosis, von Hippel-Lindau
disease, and neurofibromatosis type 2 (NF2) are other neuro-oculocutaneous disorders
(phakomatoses) but are not associated with pulsatile exophthalmos.
9. c. Orbital roof fractures occur much more commonly in young children than adults for 2
main reasons. First, pneumatized frontal sinuses, the structures that absorb much of the
force of a frontal blow in adults, are not yet developed in children. This absence of pneu-
matized frontal sinuses allows force to be transferred directly to the orbital roof, resulting
in fracture. Second, an infant or toddler has a much higher cranial vault-to-face ratio than
an adult (8:1 vs 2:1, respectively) along with a flatter face, lacking the midfacial and nasal
projection of adults. This results in a larger and more prominent forehead that is more
susceptible to injury. The frontal lobe is not underdeveloped in young children.
10. a. To preserve visual function, immediate canthotomy and cantholysis should be per-
formed to decompress the orbit. Orbital compartment syndrome is caused by hemorrhage
in the orbit, leading to optic nerve compression and compromised arterial perfusion. This
patient is experiencing increased intraocular pressure as well as decreased visual acuity
and color vision, which suggests optic nerve involvement. The most appropriate next step
is to intervene with a canthotomy and cantholysis. The other options would only delay
emergency treatment.
11. b. An intraconal tumor located between the lateral rectus muscle and the optic nerve is
best approached with a lateral orbitotomy, with or without a bone flap. Vertical eyelid-
Answers ● 353
splitting orbitotomy is best for superior lesions, while retrocaruncular and medial orbi-
totomy are best for lesions in the medial orbit.
12. a. Chronic mucus discharge from an anophthalmic socket is usually due to giant papil-
lary conjunctivitis. This is due to mechanical friction between the palpebral conjunctival
surface and the prosthesis. Everting the upper eyelid will demonstrate the papillary reac-
tion. Treatment consists of topical corticosteroids, mast cell stabilizers, and, occasionally,
prosthetic modification. Implant extrusion can present in a similar fashion, but it is less
common than papillary conjunctivitis. Implant infection is usually painful, and examina-
tion reveals purulent discharge. The weight of the prosthesis can stretch the lower eyelid
over time, but this typically manifests as lower eyelid laxity with or without ectropion
rather than entropion.
13. b. In the temporal area, the temporal (frontal) branch of cranial nerve VII crosses the zy-
gomatic arch and courses superomedially in the deep layers of the temporoparietal fascia
(also called the superficial temporalis fascia), often near branches of the temporal artery.
The facial nerve branches inferior to the zygomatic arch are deep to the superficial mus-
culoaponeurotic system (SMAS). The temporal branch of the facial nerve is found within
the superficial portion of the temporoparietal fascia (an extension of the SMAS).
14. d. Distichiasis, which can be acquired or congenital, is defined as accessory eyelashes
growing posterior to the normal row of eyelashes (from the meibomian glands or tarsus).
Trichiasis is defined as an acquired misdirection of eyelashes that curve toward the ocular
surface. Madarosis is the absence or loss of the eyelashes. Poliosis refers to a decrease or
absence of melanin in the eyelashes.
15. b. Blepharophimosis (also called blepharophimosis-ptosis-epicanthus inversus syndrome
[BPES] or blepharophimosis syndrome) is usually inherited in an autosomal dominant
fashion. Myogenic congenital ptosis, which is a condition in the differential diagnosis, is
transmitted in an autosomal recessive manner. Additionally, Turner syndrome is associ-
ated with ptosis and complete absence of 1 X chromosome or structural changes in the
chromosome. Chronic progressive external ophthalmoplegia is typically associated with
mitochondrial transmission.
16. b. This patient presents with a classic “cavernous sinus syndrome”—facial sensory changes
(in this case limited to the first division of the trigeminal nerve), diplopia, and ptosis. A
history of cutaneous squamous cell carcinoma (SCCA) of the upper eyelid should im-
mediately raise the strong probability that perineural spread of the tumor has occurred
along the skull base. Cutaneous SCCA carcinoma has a propensity for spread along sen-
sory nerves (and on occasion along the facial nerve). It is notorious for recurrence in
this cryptic fashion, even with “clear margins” on initial extirpation. The sensory changes
may precede other cranial neuropathy by weeks to months, and the symptoms are often
misdiagnosed as a “pain syndrome” or trigeminal neuralgia. The affected sensory nerve is
usually enlarged and enhanced on MRI, but these findings may be subtle and missed by the
radiologist without a good clinical history. In many cases, no previous history of cutaneous
SCCA is present, but the patient either has widespread actinic changes over the face (with
an undiagnosed SCCA) or has undergone multiple sessions of cryotherapy or cautery of
suspicious facial lesions without biopsy. Bowen disease is another name for squamous cell
carcinoma in situ of the skin. In situ carcinomas are not metastatic and would not cause
these symptoms. Basal cell carcinoma does not spread by perineural invasion. Muir-Torre
354 ● Answers
anterior lacrimal crest. The maxillary bone fuses with the lacrimal bone within the fossa,
and the lacrimal bone goes on to form the posterior lacrimal crest. A small portion of the
palatine bone juts up in the posterior orbit to form a portion of the orbital floor.
29. c. The lacrimal pump is driven by the action of the pretarsal orbicular oculi muscle, which
is innervated by the facial nerve, or CN VII. Blinking pushes the tears from the lateral
eyelid margin to the nasal eyelid margin. When the eyelids open, negative pressure pulls
the tears into the sac; when the eyelids close, the action of the orbicularis muscle creates
positive pressure that forces those tears through the nasolacrimal duct. CN III is the ocu-
lomotor nerve that supplies the medial, superior, and inferior rectus and oblique muscles,
as well as the levator muscle. CN V is the trigeminal nerve; it supplies sensory innervation
to the face as well as motor innervation to some muscles of mastication. The lacrimal
nerve carries parasympathetic innervation to the lacrimal gland as part of the lacrimal
secretory system.
30. c. When present, aberrant lacrimal ductules are found several millimeters lateral and su-
perior to the eyelash line. These ductules can lead to tearing on the skin, which mim-
ics epiphora. Accessory lacrimal ductules are usually accompanied by a tuft of eyelashes.
Simple excision is the treatment. The absence of discharge or increased tear film suggest
that this symptom is not a result of an obstruction of the lacrimal system, a condition
that could improve with either probing or probing and intubation of the lacrimal system.
Observation would not resolve the tearing.
Study Questions
Please note that these questions are not part of your CME reporting process. They are provided
here for your own educational use and identification of any professional practice gaps. The
required CME posttest is available online (see “Requesting Continuing Medical Education
Credit”). Following the questions are answers with discussions. Although a concerted effort
has been made to avoid ambiguity and redundancy in these questions, the authors recognize
that differences of opinion may occur regarding the “best” answer. The discussions are pro-
vided to demonstrate the rationale used to derive the answer. They may also be helpful in con-
firming that your approach to the problem was correct or, if necessary, in fixing the principle
in your memory. The Section 8 faculty thanks the Resident Self-Assessment Committee for
drafting these self-assessment questions and the discussions that follow.
2. What is the best noninvasive way to visualize the cysts pathognomonic of Acanthamoeba
keratitis?
a. anterior segment optical coherence tomography (AS-OCT)
b. confocal microscopy
c. sclerotic scatter
d. specular microscopy
3. Subtle abnormalities in posterior corneal contour are best evaluated by what diagnostic
tool?
a. confocal microscopy
b. Placido-disk imaging
c. Scheimpflug tomography
d. slit-lamp microscope
4. A 70-year-old man presents with chronic unilateral crusting and discharge from the right
lateral canthus. Gram stain of the discharge reveals numerous gram-negative cocci in pairs.
What pathogen is the most likely cause of the patient’s symptoms?
a. Chlamydia trachomatis
b. Haemophilus influenzae
c. Moraxella lacunata
d. Pseudomonas aeruginosa
517
518 ● Study Questions
6. Which type of topical glaucoma medication is most likely to cause aqueous tear deficiency?
a. α2-adrenergic agonists
b. β-blockers
c. carbonic anhydrase inhibitors
d. prostaglandin analogues
7. What is the primary cause of exposure keratopathy in patients with Parkinson disease?
a. cicatricial ectropion
b. decreased blink frequency
c. proptosis
d. upper-eyelid retraction
8. A 58-year-old obese man presents with chronic ocular irritation and inflammation. On ex-
amination, the clinician notes significant eyelid laxity; in addition, the upper tarsus everts
with minimal upward force. What systemic workup is indicated?
a. buccal mucosal biopsy
b. magnetic resonance imaging (MRI) of the brain
c. sleep study
d. thyroid function tests
9. After surgical removal of a pterygium, what is the best adjunctive treatment option to min-
imize the risk of recurrence and avoid any late-term complications?
a. amniotic membrane transplant
b. bevacizumab injection
c. conjunctival autograft
d. mitomycin-C (MMC) injection
11. In addition to antibiotics, what is the most appropriate management for a 3-mm central
corneal perforation caused by Neisseria gonorrhoeae?
a. application of cyanoacrylate glue
b. corneal patch graft
c. multi-layered amniotic membrane graft
d. primary closure with 10-0 nylon sutures
12. In cases of untreated congenital syphilis, at what age does the onset of interstitial keratitis
typically occur?
a. 1–3 months
b. 1–2 years
c. 6–12 years
d. 20–40 years
13. How can posterior keratoconus be distinguished from Peters anomaly anatomically?
a. Descemet membrane is present in posterior keratoconus but absent in Peters anomaly.
b. Descemet membrane is present in Peters anomaly but absent in posterior keratoconus.
c. A thin layer of uveal tissue lining the posterior cornea is present in posterior keratoco-
nus but absent in Peters anomaly.
d. A thin layer of uveal tissue lining the posterior cornea is absent in posterior keratoco-
nus but present in Peters anomaly.
16. A 73-year-old woman with hyperparathyroidism reports chronic ocular irritation and
foreign body sensation. There is a white, horizontal plaque in the interpalpebral zone of
the cornea bilaterally. Her corneal condition primarily affects what layer of the cornea?
a. epithelium
b. Bowman layer
c. anterior stroma
d. deep stroma
520 ● Study Questions
17. What is the appropriate surgical therapy for visually significant Salzmann nodular
degeneration?
a. cauterization
b. cryotherapy
c. deep anterior lamellar keratoplasty (DALK)
d. superficial keratectomy
18. A 54-year-old man who has been taking amiodarone for cardiac arrhythmia has devel-
oped deposits in a whorl-like pattern in the inferocentral corneal epithelium of both eyes.
What is the most appropriate management of the corneal deposits?
a. discontinuation of the amiodarone
b. observation
c. reduction of the dose of amiodarone
d. superficial keratectomy with excimer laser
19. A 55-year-old woman with a history of multiple episodes of severe eye pain upon awaken-
ing reports that both eyes have been affected, although not always simultaneously. What
corneal findings are most likely to be present on slit-lamp examination?
a. basement membrane fingerprint lines
b. epithelial filaments
c. punctate epithelial erosions
d. subepithelial infiltrates
20. What condition is characterized by bilateral dysfunctional corneal endothelial cells that
demonstrate epithelial-like behavior?
a. congenital hereditary endothelial dystrophy
b. Fuchs endothelial corneal dystrophy (FECD)
c. iridocorneal endothelial (ICE) syndrome
d. posterior polymorphous corneal dystrophy
21. A 57-year-old woman has confluent guttae in both eyes. She reports intermittent, tran-
sient blurred vision in her left eye on waking. At 8 am, her best-corrected visual acuity is
20/25 OD and 20/30 OS. What is the appropriate course of management?
a. bandage soft contact lens
b. hypertonic sodium chloride ointment in the left eye at night
c. topical corticosteroids 4 times daily
d. penetrating keratoplasty (PK)
Study Questions ● 521
22. What behavioral modification can lower the risk of keratoconus progression?
a. avoidance of eye rubbing
b. increased dietary consumption of omega-3 fatty acids
c. smoking cessation
d. reduction of alcohol use
23. Where does corneal thinning usually occur in eyes with pellucid marginal degeneration?
a. centrally
b. inferior periphery
c. nasal and temporal periphery
d. superior periphery
24. A 42-year-old man is referred for evaluation of red and irritated eyes. Examination reveals
severe punctate erosions on the cornea and conjunctiva. A foamy appearing patch is noted
on the temporal bulbar conjunctiva of the right eye. Tears bead along the conjunctival sur-
face in small, individual droplets. The patient’s medical history is significant for morbid
obesity, for which he underwent gastric bypass surgery 6 months prior. Abnormalities in
what cell type are responsible for the patient’s symptoms and examination findings?
a. goblet cells
b. keratocytes
c. lymphocytes
d. squamous epithelial cells
25. A young child with cognitive impairment and hyperkeratotic lesions of the palms and
soles is treated for recurrent episodes of corneal erosions and pseudodendrites that don’t
stain well with fluorescein or rose bengal dye. What treatment is indicated?
a. high-dose oral ascorbic acid
b. oral and/or topical cysteamine
c. oral penicillamine
d. restriction of tyrosine and phenylalanine intake
26. A 32-year-old man visits a clinic for a routine examination. He has a history of thyroid
cancer and uncontrolled hypertension. The anterior segment examination reveals en-
larged corneal nerves. What additional finding is most likely present in this patient?
a. angioid streaks
b. marfanoid body habitus
c. microspherophakia
d. preauricular skin tags
522 ● Study Questions
27. A 15-year-old adolescent girl with a history of renal transplant reports experiencing a
right-sided headache for 2 days. Examination reveals redness of the right forehead, right
upper eyelid, and nose, with a few clear vesicles. Slit-lamp examination shows conjunc-
tival hyperemia and superficial punctate keratitis without deeper corneal involvement or
iritis. What is the preferred management?
a. systemic antiviral medications
b. topical antiviral medications
c. oral corticosteroids
d. topical corticosteroids
28. A 34-year-old woman was treated for herpes simplex virus (HSV) epithelial keratitis with
topical trifluridine 8 times daily for 2 weeks. She reports that the ocular discomfort im-
proved but that shortly afterward, she developed a new onset of redness, tearing, and dis-
comfort. What is the most likely cause of her newer symptoms?
a. recurrent HSV keratitis
b. superimposed bacterial conjunctivitis
c. toxic epithelial keratopathy
d. trichiasis
30. A 7-year-old boy presents with a 2-day history of unilateral purulent conjunctivitis with
diffuse subconjunctival hemorrhage. Gram stain shows numerous gram-negative bacilli.
What is the most likely causative pathogen?
a. Haemophilus influenzae
b. Moraxella catarrhalis
c. Neisseria gonorrhoeae
d. Streptococcus pneumoniae
31. A 27-year-old patient has 24-hour duration of severe, hyperpurulent conjunctival discharge
without a corneal ulcer. Conjunctival Gram stain reveals numerous neutrophils with gram-
negative intracellular diplococci (GNID). What is the most appropriate treatment?
a. intramuscular ceftriaxone
b. intravenous penicillin G
c. oral azithromycin
d. topical gentamicin ointment
Study Questions ● 523
32. For acute Stevens-Johnson syndrome (SJS) with a conjunctival defect, what intervention
has the greatest long-term benefit?
a. amniotic membrane transplantation
b. placement of a symblepharon ring
c. repeated forniceal debridement with a glass rod
d. temporary tarsorrhaphy
33. A patient presents with bilateral stromal infiltrates without epithelial ulceration. During
the review of systems, the patient reports a recent viral upper respiratory tract infection
and difficulty with balance and tinnitus. Rapid plasma reagin (RPR) test and micro-
hemagglutination assay (fluorescent treponemal antibody absorption; FTA-ABS) results
are negative. What systemic treatment is indicated as part of the typical therapeutic regi-
men for this condition?
a. systemic antibiotic
b. systemic antiviral medication
c. systemic corticosteroid
d. systemic nonsteroidal anti-inflammatory drug
34. What systemic disease is most frequently associated with peripheral ulcerative keratitis?
a. granulomatosis with polyangiitis (Wegener granulomatosis)
b. rheumatoid arthritis
c. systemic lupus erythematosus
d. ulcerative colitis
35. Conjunctival melanoma in what location is associated with the best long-term prognosis?
a. bulbar conjunctiva
b. caruncle
c. fornix
d. palpebral conjunctiva
36. A 32-year-old man is diagnosed with ocular surface squamous neoplasia (OSSN). A sero-
logic test should be performed to look for what condition?
a. herpes simplex virus
b. HIV
c. human herpesvirus 8
d. varicella-zoster virus
524 ● Study Questions
37. In a patient with primary acquired melanosis (PAM), what clock-hour extent of conjunc-
tival involvement is an indication to remove the lesion?
a. more than 1 clock-hour
b. more than 2 clock-hours
c. more than 3 clock-hours
d. more than 6 clock-hours
38. What physical examination finding is the best predictor of late corneal neovascularization
in a patient with a chemical injury?
a. amount of scleral and limbal ischemia
b. degree of corneal edema
c. extent of skin and eyelid burns
d. presence of particulate chemical matter in the fornices
39. Why is tissue from donors younger than 2 years generally not used in corneal transplantation?
a. The corneal endothelial cells are not sufficiently developed.
b. The corneal stroma is too thin.
c. The tissue is steeply curved and flaccid.
d. There are ethical concerns about using tissue from infants and very young children.
40. In US eye banks, how long can corneal tissue remain viable once preserved in the storage
medium?
a. 24–48 hours
b. 3–5 days
c. 11–14 days
d. 25–35 days
525
526 ● Answers
14. b. Conjunctival concretions are epithelial inclusion cysts filled with epithelial and keratin
debris, as well as mucopolysaccharide and mucin. The majority of conjunctival concre-
tions are asymptomatic. If they erode the conjunctival epithelium and cause symptoms
of persistent foreign body irritation, they can be removed with a fine-gauge needle. Oral
azithromycin, warm compresses, and topical steroids are not effective treatments for con-
junctival concretions.
15. d. Spheroidal degeneration is characterized by the appearance of translucent, golden-
brown, spheroidal deposits in the superficial stroma and Bowman layer. Despite its
appearance, the brown deposit is proteinaceous in nature, rather than lipid. In contrast to
spheroidal degeneration, lipid keratopathy most often occurs in association with stromal
neovascularization. It can occur in the mid and deep stroma and is creamy white in color.
Glycosaminoglycan deposition is found in multiple lysosomal storage diseases. Carbohy-
drate deposits are not associated with this corneal opacification.
16. b. Calcific band keratopathy is a degeneration of the superficial cornea that involves
mainly Bowman layer. The degeneration begins as fine, dustlike, basophilic deposits in
at the level of Bowman layer, usually first seen peripherally in the 3- and 9-o’clock posi-
tions. Eventually, the deposits may coalesce to form a dense horizontal band of calcific
plaques across the interpalpebral zone of the cornea. Numerous systemic diseases, includ-
ing hyperparathyroidism, chronic renal failure, sarcoidosis, and gout are risk factors for
band keratopathy.
17. d. Salzmann nodular degeneration refers to grayish subepithelial nodules that occur pri-
marily or secondary to previous inflammatory or mechanical insult to the cornea. These
changes are typically confined to the very superficial anterior stroma and are amenable to
debridement with superficial keratectomy. Deep anterior lamellar keratoplasty (DALK)
is unnecessarily invasive for this superficial condition. Cauterization and cryotherapy are
not effective treatments for Salzmann nodular degeneration.
18. b. Cornea verticillata, or vortex keratopathy, manifests as a whorl-like pattern of golden-
brown or gray deposits in the inferior interpalpebral aspect of the cornea. The anti-
arrhythmic medication amiodarone is the most common cause of corneal verticillata.
Because it is unusual for these deposits to result in reduction of vision or ocular symp-
toms, treatments such as superficial keratectomy with excimer laser are not necessary. It
is acceptable for the patient to continue taking the recommended dose of systemic medi-
cation. The patient should be monitored by the ophthalmologist.
19. a. Recurrent corneal erosions typically occur in eyes that have suffered prior ocular sur-
face trauma, usually an abrading injury with shearing force, such as a fingernail injury.
Approximately 50% of patients with recurrent corneal erosions are believed to have under-
lying epithelial basement membrane dystrophy (EBMD), although only 10% of patients
with EBMD will develop recurrent corneal erosions. Patterns of corneal epithelial find-
ings evident on slit-lamp examination in eyes with EBMD include fingerprint lines, maps,
dots, and a bleb pattern. Filamentary keratitis is more characteristically associated with
dry eye disease and superior limbic keratoconjunctivitis than with recurrent corneal ero-
sions. Punctate epithelial erosions are associated with corneal surface dryness from dry eye
disease, but they are not a hallmark of recurrent corneal erosions. Subepithelial infiltrates
are most typically seen in epidemic keratoconjunctivitis.
20. d. Posterior polymorphous corneal dystrophy (PPCD) is characterized by bilateral (can
be asymmetric) vesicular or scalloped areas of abnormal endothelium that demonstrates
528 ● Answers
32. a. Early amniotic membrane transplantation (within the first 3–7 days) has been shown to
reduce the formation of symblepharon and minimize long-term ocular surface complica-
tions for patients with conjunctival inflammation from Stevens-Johnson syndrome (SJS).
Symblepharon rings and forniceal debridement have been largely supplanted by amniotic
membrane grafting. Repeated forniceal debridement may even worsen conjunctival scar-
ring due to mechanical trauma. Temporary tarsorrhaphy may be indicated for nonhealing
epithelial defects, but it is not directly beneficial to preventing symblepharon formation.
33. c. Bilateral nonulcerative stromal inflammation is suggestive of interstitial keratitis, an
inflammatory, nonulcerative condition that primarily involves the corneal stroma. Herpes
simplex and syphilis are the most common etiologies of interstitial keratitis in the United
States. In this case, the negative rapid plasma reagin (RPR) and fluorescent treponemal
antibody absorption (FTA-ABS) test results and the vestibuloauditory pathology sug-
gested by the review of systems are consistent with Cogan syndrome, a rare autoimmune
disease often precipitated by a viral illness, with features that overlap with polyarteritis
nodosa. Cogan syndrome can be associated with vertigo and can lead to permanent deaf-
ness if not treated promptly with systemic corticosteroids. A systemic antibiotic would not
be appropriate with a negative syphilis serology. A systemic antiviral medication could
be considered if herpes simplex was being considered. A systemic nonsteroidal anti-
inflammatory drug would not be the best choice for a patient with Cogan syndrome.
34. b. Rheumatoid arthritis is the condition most frequently associated with peripheral ulcer-
ative keratitis. Testing for rheumatoid factor (RF) is warranted as part of the systemic
evaluation. Systemic lupus erythematosus, granulomatosis with polyangiitis (Wegener
granulomatosis), and ulcerative colitis are also associated with peripheral ulcerative kera-
titis but are less commonly present.
35. a. Conjunctival melanoma–related mortality at 10 years ranges from 13% to 38%. Involve-
ment of the caruncle, palpebral conjunctiva, or conjunctival fornix are all associated with
a twofold or greater risk for mortality than is bulbar conjunctival involvement alone.
36. b. Risk factors associated with ocular surface squamous neoplasia (OSSN) include older
age, ultraviolet light exposure, prior skin cancer, male sex, smoking, fair complexion,
human papillomavirus (HPV) infection, HIV infection, and systemic immunosuppres-
sion. In a young adult, OSSN should prompt consideration of a serologic test for HIV.
Rapid growth of lesions may occur in a person with AIDS. Neither herpes simplex virus
nor varicella-zoster virus is considered a risk factor for OSSN. Human herpesvirus 8 is the
etiology of Kaposi sarcoma.
37. b. Primary acquired melanosis (PAM) is an acquired, noncystic, flat, patchy or diffuse, tan
to brown pigmentation of the conjunctival epithelium. PAM is usually unilateral or asym-
metric if bilateral and is most often seen in individuals with fair skin. Two clock-hours or
less of conjunctival involvement is associated with a lower risk of malignant transforma-
tion; involvement of more than 2 clock-hours is an indication to remove the lesion for
histologic diagnosis.
38. a. Although all these clinical features are pertinent when documenting the extent of ocu-
lar surface injury resulting from chemical trauma, the degree of limbal ischemia is the
most important predictor of progression to limbal stem cell deficiency (which results in
corneal neovascularization). Treatments to improve prognosis include those intended to
minimize ongoing exposure to the agent (eg, immediate irrigation with water or saline,
Answers ● 531
1. What are the main effector cells of the innate immune system?
a. CD4+ T cells and CD8+ T cells
b. macrophages and neutrophils
c. mast cells and dendritic cells
d. plasma cells and B lymphocytes
2. What type of macrophage produces the full spectrum of inflammatory and cytotoxic
cytokines?
a. resting macrophage
b. primed macrophage
c. activated macrophage
d. stimulated macrophage
363
364 ● Study Questions
5. What agent used in therapy for uveitis may induce anti-idiotypic antibodies?
a. infliximab
b. interferon alfa-2a/2b
c. methotrexate
d. tacrolimus
6. What feature allows the ocular surface to mount an antibody-mediated effector response
to influenza viral antigens after a patient is administered an intranasal influenza vaccine?
a. abundance of antigen-presenting cells in the conjunctiva
b. predominance of immunoglobulin (Ig) A within the tear film
c. presence of conjunctiva-associated lymphoid tissue
d. high density of mast cells within the conjunctiva
8. What immune cells are present in the choroid under normal physiologic conditions?
a. B lymphocytes
b. eosinophils
c. macrophages
d. neutrophils
9. What ocular inflammatory disease has the strongest human leukocyte antigen (HLA)
association?
a. birdshot chorioretinopathy and HLA-A29
b. acute anterior uveitis and HLA-B27
c. Behçet disease and HLA-B51
d. intermediate uveitis and HLA-DR15
11. A patient is treated with topical corticosteroids for anterior uveitis in the right eye. At the
1-month visit, the eye is quiet without medication. Four months later, the patient returns
Study Questions ● 365
and is again found to have anterior uveitis in the right eye. According to the Standard-
ization of Uveitis Nomenclature (SUN) Working Group classification, what is the best
description of this patient’s uveitis?
a. acute
b. chronic
c. limited
d. recurrent
12. What medication used to treat uveitis may exacerbate multiple sclerosis?
a. adalimumab
b. interferon alfa-2a/2b
c. methylprednisolone
d. rituximab
13. A patient is being treated for first-time presentation of undifferentiated granulomatous iri-
docyclitis. She responds very well to hourly topical corticosteroid, but in the past 3 months,
the intraocular inflammation recurred every time the ophthalmologist tried to taper the
corticosteroid frequency to less than 4 times per day. A posterior subcapsular cataract is
beginning to develop, and the intraocular pressure is now 23 mm Hg despite the patient’s use
of 3 types of antiglaucoma eyedrops. What is the most appropriate next step in management
of this case?
a. Continue topical corticosteroids and schedule the patient for glaucoma drainage im-
plant surgery.
b. Continue topical corticosteroids and initiate therapy with methotrexate.
c. Increase topical corticosteroids and initiate therapy with oral acetazolamide.
d. Increase topical corticosteroids and initiate therapy with an oral nonsteroidal anti-
inflammatory drug (NSAID).
14. What type of scleritis is most likely to be associated with life-threatening systemic disease?
a. nodular scleritis
b. posterior scleritis
c. diffuse scleritis
d. necrotizing scleritis
15. What finding is associated with an increased risk of uveitis in patients with juvenile idio-
pathic arthritis (JIA)?
a. antinuclear antibody positivity
b. polyarticular involvement
c. rheumatoid factor positivity
d. systemic involvement
366 ● Study Questions
16. Abnormal urinalysis findings, with increased β2-microglobulin level, in a patient with
uveitis should prompt initiation of what systemic treatment?
a. antibiotics
b. anticoagulants
c. corticosteroids
d. xanthine oxidase inhibitor
17. A 23-year-old man purchases a genetic testing kit and learns that he is positive for HLA-
B27. He becomes worried about developing acute anterior uveitis after reading stories on
the Internet and presents for an eye examination. What is the most likely finding on this
patient’s slit-lamp examination?
a. hypopyon
b. posterior synechiae
c. posterior subcapsular cataract
d. quiet anterior chamber
18. How often should a 6-year-old girl with a 2-year history of antinuclear antibody–positive
oligoarticular JIA be screened for uveitis?
a. every 3 months
b. every 6 months
c. every 9 months
d. every 12 months
19. A patient is referred for evaluation of anterior uveitis in the right eye. On examination,
there is conjunctival injection, 1+ cell, and granulomatous keratic precipitates (KPs). The
patient is on topical therapy (dorzolamide, timolol, and brimonidine) for glaucoma and is
also undergoing treatment for latent tuberculosis with oral rifampin. What medication is
most likely to be responsible for this patient’s presentation?
a. brimonidine
b. dorzolamide
c. rifampin
d. timolol
20. What is the typical onset and disease course for birdshot chorioretinopathy?
a. acute onset with progressive course
b. acute onset with self-limiting course
c. insidious onset with progressive course
d. insidious onset with self-limiting course
21. Peripheral necrotizing retinochoroiditis resembling acute retinal necrosis can be seen in
patients infected with what pathogen?
a. Mycobacterium tuberculosis
b. Nocardia asteroides
Study Questions ● 367
c. Pneumocystis jirovecii
d. Treponema pallidum
22. When stellate KPs are present in intraocular infections caused by herpes simplex virus
type 1 (HSV-1), what description best characterizes their appearance on the corneal
endothelium?
a. They are distributed diffusely, often extending above the corneal equator.
b. They are distributed inferiorly, specifically only in the Arlt triangle.
c. They manifest as ring-shaped clusters.
d. KPs are not found in HSV-1 intraocular infections.
23. What is an appropriate induction treatment option for a patient with varicella-zoster
virus–associated acute retinal necrosis?
a. oral acyclovir 800 mg 5 times/day
b. oral valacyclovir 1 g 3 times/day
c. oral valacyclovir 2 g 3 times/day
d. oral valganciclovir 900 mg twice/day
26. A patient with a history of AIDS (last CD4+ T-lymphocyte count 89 cells/μL) presents
for evaluation of decreased vision and is diagnosed with ocular toxoplasmosis. What test
should be ordered promptly for this patient?
a. serologic tests for syphilis (rapid plasma reagin/fluorescent treponemal antibody ab-
sorption test)
b. serologic tests for Toxoplasma gondii (IgG and IgM)
c. computed tomography of the chest
d. magnetic resonance imaging of the brain
368 ● Study Questions
27. A patient with AIDS presents with bilateral pale-yellow, placoid retinal lesions. What test-
ing and treatment should be ordered?
a. Check CD4+ helper T-lymphocyte count and start antiretroviral therapy.
b. Check CD4+ helper T-lymphocyte count and start systemic corticosteroids.
c. Check syphilis antibodies and rapid plasmin reagin and start intramuscular benza-
thine penicillin G, 2.4 million units.
d. Check syphilis antibodies and rapid plasmin reagin and start intravenous penicillin G,
18–24 million units.
28. An immunosuppressed patient presents with features consistent with unilateral endoph-
thalmitis. A pars plana vitrectomy is performed, and analysis of vitreous fluid specimens
reveals septate hyphae with dichotomous branching on silver stain. What is the most likely
organism?
a. Aspergillus fumigatus
b. Candida albicans
c. Coccidioides immitis
d. Cryptococcus neoformans
29. A patient with candidemia is found to have yellow-white chorioretinal lesions without
vitritis. What is the most appropriate initial treatment option?
a. intravitreal vancomycin
b. oral voriconazole
c. pars plana vitrectomy
d. posterior sub-Tenon injection of triamcinolone
30. Four months after cataract surgery in the right eye, a patient had recurrent 1–2+ anterior
chamber cell and flare each time topical corticosteroids were tapered. Now, a hypopyon,
an iris mass, necrotizing scleritis, and vitreous snowballs have developed. Infection with
what organism most likely caused this condition?
a. Candida species
b. Histoplasma species
c. Pseudomonas species
d. Streptococcus species
31. A 78-year-old man presents for a routine eye examination. His ocular history is notable
only for uncomplicated bilateral cataract surgery 5 years earlier. On examination, intra-
ocular pressure is 6 mm Hg in the right eye and 13 mm Hg in the left eye. The right-eye
examination is notable for 1+ cell and 2+ flare in the anterior chamber. Findings from the
left-eye examination are unremarkable. The fundus examination shows clear vitreous in
both eyes and dilated retinal venules and narrowed arterioles, worse in the right eye. What
is the most appropriate next step in the care of this patient?
a. Start topical corticosteroid drops.
b. Perform a vitreous biopsy to obtain a specimen for flow cytometry.
Study Questions ● 369
32. A 65-year-old woman with vitritis and negative results on the treponemal antibody test
and interferon-gamma release assay responds well initially to systemic corticosteroids;
however, as the corticosteroids are tapered, the vitritis recurs. A vitreous biopsy is per-
formed, and cytologic analysis shows pleomorphic lymphoid cells with a high nuclear-
to-cytoplasm ratio. Approximately what proportion of patients with this condition will
develop intracranial lesions characterized by the same cells?
a. less than 1 in 10
b. 1 in 3
c. 1 in 2
d. more than 2 in 3
33. A child with a history of reddish-yellow skin lesions presents with unilateral recurrent
hyphema and anterior chamber cells. A few iris lesions are noted. What is the best test to
identify the etiology of this patient’s intraocular inflammation and hyphema?
a. anterior chamber paracentesis with directed polymerase chain reaction (PCR) for
cytomegalovirus
b. skin or iris biopsy to evaluate for foamy macrophages
c. anterior chamber paracentesis with directed PCR for rubella virus
d. skin or iris biopsy to evaluate for caseating granulomas
34. A patient who is being treated with mycophenolate mofetil for bilateral pars planitis has
persistent macular edema in the right eye. Examination of the anterior chamber and vitre-
ous shows no evidence of active inflammation. What is the most appropriate next step to
manage the macular edema in this patient?
a. Switch to cyclophosphamide.
b. Administer intravenous methylprednisolone 1 g daily for 3 days.
c. Administer periocular or intravitreal triamcinolone acetonide injection.
d. Perform pars plana vitrectomy with membrane peeling.
35. A 12-year-old patient with a 7-year history of JIA-associated chronic anterior uveitis
(CAU) presents for her regular follow-up appointment. She uses prednisolone drops once
daily and is asymptomatic. What finding can be observed on examination?
a. band keratopathy
b. fibrinous anterior chamber reaction
c. hypopyon
d. vitritis
370 ● Study Questions
36. A child with JIA-associated chronic bilateral nongranulomatous uveitis has developed
band keratopathy. What pathologic finding can be observed in this patient’s cornea?
a. subepithelial urate deposition
b. subepithelial calcium deposition
c. pre-Descemet urate deposition
d. pre-Descemet calcium deposition
Answers
1. b. The innate immune system is a relatively broad-acting rapid reaction force that rec-
ognizes “nonself ” (foreign) substances, proteins, or lipopolysaccharides. The innate re-
sponse can be thought of as a preprogrammed reaction that is immediate, requires no
prior exposure to the foreign substance, and is similar for all encountered triggers. The
result is the generation of biochemical mediators and cytokines that recruit innate effector
cells, especially macrophages and neutrophils, to remove the offending stimulus through
phagocytosis or enzymatic degradation.
2. c. Activated macrophages are classically defined as macrophages that produce the full
spectrum of inflammatory and cytotoxic cytokines; thus, they mediate and amplify acute
inflammation, tumor killing, and major antibacterial activity. Epithelioid cells and giant
cells represent different terminal differentiations of the activated macrophage.
Resting macrophages are the classic scavenging cell, capable of phagocytosis and uptake
of dead cell membranes, chemically modified extracellular protein (ie, acetylated or oxi-
dized lipoproteins), sugar ligands, naked nucleic acids, and bacterial pathogens. Resting
monocytes express at least 3 types of scavenging receptors but synthesize very low lev-
els of proinflammatory cytokines. Scavenging can occur in the absence of inflammation.
Primed macrophages are derived from resting macrophages that become primed by expo-
sure to certain cytokines. Upon priming, these cells become positive for human leukocyte
antigen (HLA) class II molecules and capable of functioning as antigen-presenting cells
(APCs) to T lymphocytes. Primed macrophages thus resemble dendritic cells. They can
exit tissue sites by afferent lymphatic vessels to reenter the lymph node. Stimulated (or re-
parative) macrophages are partially activated and produce some inflammatory cytokines
that contribute to fibrosis and wound healing.
3. c. Monocytes can be primed and induced to become APCs. Basophils, eosinophils, and
neutrophils generally do not have the ability to act as APCs.
4. c. Because they are proteins, antibodies themselves can be antigenic. Their antigenic sites
are called idiotopes, as distinguished from epitopes, which are the antigenic sites on foreign
molecules. An allotope is a site on the constant or nonvarying part of an antibody molecule
that is recognizable by a combining site of other antibodies. Isotopes are 2 or more atoms
whose nuclei have the same number of protons but different numbers of neutrons.
5. a. Infliximab is a mouse/human-chimeric monoclonal antibody directed against tumor
necrosis factor α (TNF-α). One study showed that as many as 75% of patients receiving
more than 3 infusions develop antinuclear antibodies, causing drug-induced lupus syn-
drome and the formation of human anti-idiotypic (anti-chimeric) antibodies, which can
lead to reduced efficacy of infliximab.
Methotrexate, interferon alfa-2a/2b, and tacrolimus are not associated with formation
of anti-idiotypic antibodies. Methotrexate is an antimetabolite drug that is often coad-
ministered with biologics for synergistic effect and to reduce the risk of developing anti-
drug antibodies. Interferon alfa-2a/2b, administered subcutaneously, has been beneficial
in some patients with uveitis. Interferon alfa-2a has antiviral, immunomodulatory, and
antiangiogenic effects. Tacrolimus, a product of Streptomyces tsukubaensis, is a calcineurin
inhibitor that eliminates T-cell receptor signal transduction and downregulates interleu-
kin-2 gene transcription and receptor expression of CD4+ T lymphocytes.
371
372 ● Answers
Uveal manifestations can include granulomatous KPs and iris nodules, which are granulo-
mas on histologic examination. Hypertensive anterior uveitis does not occur, however. Cho-
roidal involvement manifests as multifocal choroidal granulomas most frequently located in
the postequatorial fundus.
25. d. The pathogen associated with the development of Kaposi sarcoma is human herpesvirus
(HHV)-8. Epstein-Barr virus (HHV-4) is associated with the development of primary cen-
tral nervous system lymphomas and vitreoretinal lymphomas in HIV-infected individuals.
HSV-1 (HHV-1) and CMV (HHV-5) are not associated with any specific neoplasias in per-
sons with HIV infection.
26. d. Ocular toxoplasmosis in immunocompromised patients may be associated with cere-
bral or disseminated toxoplasmosis. For patients with AIDS who have active ocular toxo-
plasmosis, neuroimaging (preferably magnetic resonance imaging) should be performed to
evaluate for central nervous system involvement. Although syphilis is reemerging globally,
particularly in association with HIV coinfection, there is no indication for urgent syphilis
testing in this patient. Since the patient has already been diagnosed with ocular toxoplasmo-
sis, serologic testing is not immediately useful. Computed tomography of the chest might
be indicated if there is concern for pulmonary infection or inflammation, but in this case,
expeditious investigation for toxoplasmic encephalitis is necessary.
27. d. Suspicion for syphilis should be high in this patient. Syphilis is a common coinfection in
HIV-infected individuals. A classic manifestation of syphilis in patients with AIDS is uni-
lateral or bilateral pale-yellow, placoid retinal lesions that preferentially involve the macula.
Checking syphilis antibodies (treponemal test) and the rapid plasmin reagin (RPR, nontrepo-
nemal test) will help confirm the diagnosis and assess response to treatment. Ocular cases of
syphilis should be treated in the same manner as neurosyphilis, and 18–24 million units of
IV penicillin G administered daily for 10–14 days is indicated. Syphilis without neurologic or
ocular involvement may be treated intramuscularly as a single dose.
Checking CD4+ helper T-lymphocyte count and starting antiretroviral therapy (ART)
would be appropriate if the patient exhibited features of HIV retinopathy (cotton-wool spots,
microaneurysms, and retinal hemorrhages) only. Systemic corticosteroids may be used in
treating immune recovery uveitis, an inflammatory process that affects patients with a his-
tory of CMV retinitis and AIDS whose immune status improves with ART. Systemic cortico-
steroids may also be used if there is a prominent inflammatory reaction in syphilis, but this
medication should not be started until after treatment with IV penicillin has commenced.
28. a. Histologically, Aspergillus species is characterized by septate hyphae with dichotomous
branching. Candida species are recognized as budding yeast with a characteristic pseudo-
hyphate appearance. Histologically, C immitis appears as spherules with multiple endo-
spores. Cryptococcus neoformans does not appear as hyphae but rather as round capsules
with a halo on India ink; periodic acid–Schiff (PAS) and Gomori methenamine silver
(GMS) stain the organism and leave the capsule unstained.
29. b. In this case, the hematogenous dissemination of Candida organisms involves only the
choroid, so appropriate initial treatment includes systemic antifungal medication and
close monitoring. Oral voriconazole has excellent intraocular penetration. If there is vit-
real involvement, pars plana vitrectomy in conjunction with systemic and intravitreal
antifungals may be necessary. Monotherapy with periocular or intraocular depot cortico-
steroids will worsen the infectious endophthalmitis. Intravitreal injection of the antibiotic
vancomycin will not treat the Candida infection.
376 ● Answers
30. a. Recurrent inflammation in an eye that has previously undergone surgery is concern-
ing for chronic postoperative endophthalmitis. While the presentation of chronic post-
operative fungal endophthalmitis may be similar to that of a bacterial infection, certain
clinical signs make a fungal etiology more likely, including a corneal infiltrate or edema,
a mass in the iris or ciliary body, necrotizing scleritis, vitreous snowballs, or a “string-
of-pearls” appearance in the vitreous. Chronic postoperative bacterial endophthalmitis
is most commonly caused by Cutibacterium acnes (formerly Propionibacterium acnes), a
gram-positive anaerobe. Other gram-positive bacteria, gram-negative bacteria, Mycobac-
terium species, and fungi (such as Candida parapsilosis, Aspergillus flavus, and Torulopsis
candida) can also cause chronic postoperative endophthalmitis.
31. c. The most likely diagnosis is ocular ischemic syndrome, which can masquerade as uve-
itis. Carotid Doppler ultrasonography should be ordered to assess for ipsilateral stenosis
of the carotid artery. Given the lack of symptoms and lack of previous uveitic episodes,
this presentation in an older man is unlikely to be caused by uveitis; therefore, empiric
topical corticosteroid therapy is not indicated. Given this patient’s age, it is appropriate to
consider lymphoma in the differential diagnosis. however, the patient’s vitreous cavity is
clear, and the diagnosis of vitreoretinal lymphoma (formerly called primary intraocular
lymphoma) is unlikely. While extensive intraocular surgery can result in breakdown of
the blood–aqueous barrier, this breakdown does not typically result from uncomplicated
cataract surgery (nor would it explain the hypotony and absence of cells in the left eye).
32. d. The case described clinically is consistent with vitreoretinal lymphoma (VRL), and the
cytopathologic findings confirm the diagnosis. In more than two-thirds of cases of VRL,
patients will have or will go on to develop intracranial disease. In fact, VRL is a subset of
primary central ner vous system lymphoma (PCNSL). Approximately 25% of patients with
PCNSL (who do not yet have intraocular involvement) will go on to develop VRL.
33. b. The case described is consistent with juvenile xanthogranuloma, which may feature
reddish-yellow skin lesions. Intraocular features may include recurrent hyphema. Histo-
logically, skin or iris biopsy specimens would be expected to show large histiocytes with
foamy cytoplasm and Touton giant cells. Caseating granulomas are a feature of tubercu-
lous granulomas, but tuberculosis-associated uveitis does not feature recurrent hyphemas.
CMV and rubella virus have been associated with forms of hypertensive anterior uveitis,
but they are not associated with the skin lesions described in this case.
34. c. Uveitic macular edema is a frequent complication of uveitis and may occur or persist
even when inflammation seems otherwise controlled. The use of periocular or intravitreal
corticosteroids is the most appropriate next step to resolve this patient’s uveitic macu-
lar edema. The PeriOcular vs. INTravitreal Corticosteroids for Uveitic Macular Edema
(POINT) trial demonstrated superior visual outcomes with intravitreal triamcinolone and
intravitreal dexamethasone implant compared with sub-Tenon triamcinolone injection.
Macular optical coherence tomography can help identify factors contributing to the
macular edema, such as vitreomacular traction. Since there is no evidence of prominent
vitreomacular traction in this case, a pars plana vitrectomy with membrane peeling is not
indicated. Although macular edema can result in irreversible vision loss, an alkylating
agent such as cyclophosphamide would be reserved for more aggressive, sight-threatening
ocular inflammation. Similarly, pulses of IV methylprednisolone are more typically used
to rapidly control acute, high-grade inflammation.
Answers ● 377
35. a. Chronic anterior uveitis (CAU) associated with JIA is an indolent, relatively asymptomatic
anterior uveitis in children. After years of low-grade inflammation, these eyes may develop
signs of chronic inflammation, which include band keratopathy, cataracts, and/or synechiae.
Fibrinous anterior chamber reaction and hypopyon are characteristic of acute anterior uve-
itis, which is usually symptomatic. Although patients with JIA-associated CAU may have an-
terior vitreous cells, the vitreous is not the predominant site of inflammation and therefore
vitritis is usually not present.
36. b. Band keratopathy results from the deposition of calcium hydroxyapatite at the level of
the Bowman membrane. Aside from chronic inflammation associated with uveitis, other
etiologies of calcific band keratopathy include systemic disorders that cause hypercalcemia
or elevated serum phosphorus levels, as well as silicone oil (particularly in an aphakic eye),
exposure to mercurial vapors or preservative, and primary hereditary band keratopathy.
Study Questions
Please note that these questions are not part of your CME reporting process. They are pro-
vided here for your own educational use and for identification of any professional practice gaps.
The required CME posttest is available online (see “Requesting Continuing Medical Education
Credit”). Following the questions are answers with discussions. Although a concerted effort has
been made to avoid ambiguity and redundancy in these questions, the authors recognize that
differences of opinion may occur regarding the “best” answer. The discussions are provided to
demonstrate the rationale used to derive the answer. They may also be helpful in confirming
that your approach to the problem was correct or, if necessary, in fixing the principle in your
memory. The Section 10 faculty thanks the Resident Self-Assessment Committee for develop-
ing these self-assessment questions and the discussions that follow.
1. From where is the arterial blood supply of the anterior optic nerve primarily derived?
a. central retinal artery
b. circle of Zinn-Haller
c. pial arteries
d. short posterior ciliary arteries
2. According to the Goldmann equation, what is the ratio of the increase in intraocular pres-
sure (IOP) to the increase in episcleral venous pressure in millimeters of mercury (mm Hg)?
a. 1:0.5
b. 1:1
c. 1:1.5
d. 1:2
3. What type of tonometer is based on the Imbert-Fick principle for measurement of IOP?
a. Perkins tonometer
b. pneumotonometer
c. rebound tonometer
d. Schiøtz tonometer
4. Which parameter of the modified Goldmann equation cannot be directly measured clin-
ically and must be calculated from the Goldmann equation after determination of the
other parameters?
a. aqueous humor production rate
b. episcleral venous pressure
c. outflow facility
d. uveoscleral outflow rate
295
296 ● Study Questions
5. When the biomechanical properties of the cornea and glaucoma are considered, what
instrument can measure corneal hysteresis?
a. corneal pachymeter
b. dynamic contour tonometer
c. iCare tonometer
d. Ocular Response Analyzer
7. A patient presents with narrow angles and findings suggestive of iris and/or ciliary body
cysts. What is the best imaging modality for further investigation?
a. anterior segment optical coherence tomography
b. anterior segment ultrasound biomicroscopy
c. B-scan ultrasonography
d. computed tomography scan of the orbits
8. An ophthalmologist examines a patient’s optic nerve head at the slit lamp using indirect bio-
microscopy. The patient’s cup–disc ratio is asymmetric, with a difference of approximately
0.3 between the 2 eyes. What percentage of individuals without glaucoma would be expected
to have this amount of asymmetry?
a. less than 20%
b. less than 10%
c. less than 5%
d. less than 1%
10. A glaucoma suspect patient demonstrates superior and inferior defects in the left eye on
perimetry with a reliable baseline visual field test (Swedish Interactive Threshold Algo-
rithm Standard 24-2). What is the most appropriate next step?
a. Begin medical therapy with a prostaglandin analogue.
b. Obtain imaging of the optic nerve head.
c. Order neuroimaging.
d. Repeat visual field testing.
12. In the Collaborative Normal-Tension Glaucoma Study (CNTGS), what was the IOP re-
duction target?
a. 10%
b. 20%
c. 30%
d. 40%
13. What percentage of patients with primary open-angle glaucoma (POAG) present with an
IOP below 22 mm Hg?
a. 0%–25%
b. 30%–50%
c. 50%–75%
d. 75%–100%
14. What was the finding of the Baltimore Eye Survey regarding the impact of aging on glaucoma?
a. The impact of perfusion pressure on glaucoma increases with age.
b. The prevalence of glaucoma increases with age.
c. The rate of conversion from ocular hypertension to POAG increases with age.
d. The rate of glaucoma progression increases with age.
15. A patient with a history of 2 piggyback intraocular lenses in the left eye, both placed within
the capsular bag, is referred by a local optometrist for evaluation of elevated IOP in that eye.
Endothelial pigment deposition is noted on slit-lamp examination. What gonioscopic find-
ing might be expected in this scenario?
a. peripheral anterior synechiae
b. posterior embryotoxon
c. Sampaolesi line
d. widened ciliary body band
298 ● Study Questions
16. An 89-year-old woman presents with pain in her right eye. She reports that the vision in her
right eye has been poor for years, owing to complications from a retinal detachment. On
examination, the IOP in the right eye is 40 mm Hg. Her examination is significant for a white
cataract with a wrinkled anterior capsule. On gonioscopy, ciliary body is visible in all quad-
rants. Cell and flare is noted in the anterior chamber, but no keratic precipitates are observed.
What is the most likely diagnosis?
a. lens particle glaucoma
b. phacoantigenic glaucoma
c. phacolytic glaucoma
d. phacomorphic glaucoma
17. Which category of topical medication should be avoided in patients with uveitic glaucoma?
a. α-agonists
b. β-blockers
c. carbonic anhydrase inhibitors
d. miotic agents
19. What is the most common cause of glaucoma associated with primary or metastatic tu-
mors of the ciliary body?
a. angle closure resulting from rotation of the ciliary body
b. deposition of tumor cells and inflammatory cells in the trabecular meshwork
c. direct invasion of the anterior chamber angle
d. neovascularization of the anterior chamber angle
20. A patient reports seeing halos and having blurred vision, with mild pain in the left eye, for
2 days. The IOP is 14 mm Hg in the right eye and 46 mm Hg in the left eye. There is no con-
junctival hyperemia. There are a few inferior keratic precipitates, and the anterior chamber is
deep with occasional circulating cells. Eight months previously, the patient had a similar epi-
sode that resolved within days of using timolol eyedrops. What is the most likely diagnosis?
a. primary angle-closure glaucoma
b. Fuchs uveitis syndrome
c. herpetic keratouveitis
d. glaucomatocyclitic crisis
Study Questions ● 299
21. What is the most common cause of blindness related to glaucoma in the Chinese population?
a. POAG
b. secondary open-angle glaucoma
c. primary angle-closure glaucoma
d. secondary angle-closure glaucoma
22. What were the findings of the Zhongshan Angle-Closure Prevention (ZAP) trial?
a. Laser peripheral iridotomy (LPI) had no impact on the course of progression of any
type of angle closure.
b. LPI significantly reduced the incidence of the development of acute angle closure.
c. LPI significantly reduced the incidence of the development of primary angle closure.
d. LPI significantly reduced the incidence of the development of POAG.
23. What surgical vitreoretinal procedure can result in non–pupillary block angle closure?
a. scleral buckle placement
b. pars plana vitrectomy with intraocular gas placement
c. pars plana vitrectomy with silicone oil placement
d. intravitreal anti–vascular endothelial growth factor injection
24. An 82-year-old patient who takes warfarin and previously had a central retinal vein occlu-
sion in the left eye presents with an IOP of 39 mm Hg and neovascularization of the iris and
angle. The patient has severe eye pain and has had substantially decreased vision for several
months. Examination reveals a relative afferent pupillary defect in the left eye and hand mo-
tion vision. What is the most appropriate treatment option for the elevated IOP?
a. selective laser trabeculoplasty
b. transscleral cyclophotocoagulation
c. anterior chamber tube shunt implantation
d. trabeculectomy with mitomycin C
25. A 50-year-old woman presents with an IOP of 27 mm Hg in the right eye and 12 mm Hg
in the left eye. On examination, an abnormal corneal endothelium is noted in the right
eye; findings in the left eye are normal. Findings from the anterior segment examination
are otherwise unremarkable. On gonioscopy, peripheral anterior synechiae anterior to the
Schwalbe line are noted in 1 clock-hour. What is the most likely diagnosis?
a. iridocorneal endothelial syndrome
b. pseudoexfoliation syndrome
c. uveitis
d. Fuchs endothelial corneal dystrophy
300 ● Study Questions
26. A patient with primary congenital glaucoma (PCG) asks the ophthalmologist’s advice re-
garding family planning. Notably, she has no relatives with PCG. What is the likelihood of
her having a child affected with PCG?
a. 0%–25%
b. 25%–50%
c. 50%–75%
d. 75%–100%
27. A 2-year-old child is brought in for an eye examination because of persistent tearing.
What corneal diameter measurement should trigger concern for glaucoma?
a. greater than 11.5 mm
b. greater than 12.0 mm
c. greater than 12.5 mm
d. greater than 13.0 mm
28. What finding on examination of a patient with congenital glaucoma can continue to
change and indicate progressive glaucoma even though the IOP appears to be controlled?
a. axial length
b. corneal thickness
c. gonioscopic findings
d. myopia
29. What class of topical ocular hypotensive medication should be discontinued in a patient
who also takes an oral monoamine oxidase inhibitor?
a. prostaglandin analogues
b. α-adrenergic agonists
c. β-blockers
d. carbonic anhydrase inhibitors
31. An 84-year-old patient who received multiple-drug therapy for glaucoma returns to the
clinic after 1 year and has an IOP of 31 mm Hg, as measured by applanation tonometry,
during the follow-up examination. The patient lives alone and has a history of chronic
obstructive pulmonary disease, Parkinson disease, and nonadherence. What is the best
surgical treatment option for this patient?
a. laser trabeculoplasty
b. trabeculectomy
c. plate-based tube implantation
d. microinvasive glaucoma surgery
Study Questions ● 301
32. A patient with chronic angle-closure glaucoma develops a shallow anterior chamber 3 days
after undergoing a trabeculectomy. On examination, the patient has no bleb, no leak, a pa-
tent surgical iridectomy, no choroidal effusions, and an IOP of 20 mm Hg. What is the best
initial treatment option?
a. placement of an oversized contact lens
b. increase in frequency of topical steroid
c. cycloplegic therapy
d. injection of viscoelastic into the anterior chamber
Answers
1. d. The arterial supply of the anterior optic nerve is derived entirely from branches of the oph-
thalmic artery via 1–5 posterior ciliary arteries. Typically, between 2 and 4 posterior ciliary
arteries course anteriorly before dividing into approximately 10–20 short posterior ciliary ar-
teries, which occurs before these arteries enter the posterior globe. Often, the posterior ciliary
arteries separate into a medial and a lateral group before branching into the short posterior
ciliary arteries, which penetrate the perineural sclera of the posterior globe to supply the
peripapillary choroid, as well as most of the anterior optic nerve. Some short posterior ciliary
arteries course, without branching, through the sclera directly into the choroid; others divide
within the sclera to provide branches to the choroid and the optic nerve. A discontinuous ar-
terial circle, the circle of Zinn-Haller, is often present within the perineural sclera. The central
retinal artery, also a posterior orbital branch of the ophthalmic artery, penetrates the optic
nerve approximately 10–15 mm posterior to the globe. The central retinal artery has few, if
any, intraneural branches, the exception being an occasional small branch within the retrolam-
inar region, which may anastomose with the pial system. The central retinal artery courses
adjacent to the central retinal vein within the central portion of the optic nerve. The retrolam-
inar region is supplied by branches of the short posterior ciliary arteries and by branches
of the pial arteries coursing adjacent to this region. The pial arteries originate both from the
central retinal artery, before it pierces the retrobulbar optic nerve, and from branches of the
short posterior ciliary arteries more anteriorly.
2. b. The modified Goldmann equation is a mathematical model of the relationship between
intraocular pressure (IOP) and the parameters that contribute to its level in the eye at
steady state:
P0 = (F − U)/C + PV
where P0 is the IOP in millimeters of mercury (mm Hg), F is the rate of aqueous humor
production in microliters per minute (μL/min), U is the rate of aqueous humor drain-
age through the pressure-insensitive uveoscleral pathway in μL/min, C is the facility of
outflow through the pressure-sensitive trabecular meshwork pathway in microliters per
minute per millimeter of mercury (μL/min/mm Hg), and PV is the episcleral venous pres-
sure (EVP) in mm Hg. According to this equation, IOP rises approximately 1 mm Hg for
every 1-mm-Hg increase in EVP.
EVP is often increased in patients with Sturge-Weber syndrome (encephalofacial an-
giomatosis), carotid-cavernous sinus fistulas, and cavernous sinus thrombosis as a result
of obstruction of venous return to the heart or from shunting of blood from the arterial to
the venous system. EVP may be partially responsible for the elevated IOP seen in thyroid
eye disease. EVP is normally relatively stable, ranging from 6 to 9 mm Hg, as measured
with special equipment. Elevated EVP can increase outflow resistance by decreasing the
cross-sectional area of the Schlemm canal and may alter uveoscleral outflow.
3. a. Like the Goldmann tonometer, the Perkins tonometer is an applanation tonometer and,
as such, it is based on the Imbert-Fick principle. The Imbert-Fick principle states that the
pressure inside an ideal dry, infinitely thin-walled sphere equals the force required to flat-
ten its surface divided by the area of the flattening. The Goldmann applanation tonometer
and the Perkins tonometer use the same measurement tip, which balances the surface ten-
sion of the tear film with the rigidity of the cornea to approximate a dry, infinitely flexible,
303
304 ● Answers
thin-walled sphere for eyes with central corneal thickness (CCT) of 520 μm. The pneumo-
tonometer, rebound tonometer, and the Schiøtz tonometer do not rely on the Imbert-Fick
principle.
4. d. Direct measurement of the uveoscleral outflow rate is an invasive process that involves
perfusion of a tracer into the anterior segment of the eye, followed by estimation of the tissue
distribution of the tracer. Thus, in humans, the uveoscleral outflow rate must be calculated
by using the Goldmann equation and the parameters IOP, aqueous humor production rate,
outflow facility, and EVP. The rate of aqueous humor production by the ciliary processes
cannot be measured noninvasively, but it is assumed to be equal to the aqueous humor out-
flow rate in an eye at steady state. Aqueous humor outflow can be measured with fluoro-
photometry; EVP, with venomanometry; and outflow facility, with tonography. All of these
procedures are noninvasive.
5. d. The Ocular Response Analyzer (Reichert Technologies; see Chapter 2, Fig 2-7C), a type
of noncontact tonometer, uses correction algorithms so that its IOP readings more closely
match those obtained with applanation techniques, and the effect of corneal biomechani-
cal properties on pressure measurement is reduced. Studies have found that the corneal
compensated IOP (IOPcc) has a stronger correlation with glaucoma progression than does
IOP measured with Goldmann or rebound tonometry. In addition to measuring IOPcc,
the Ocular Response Analyzer calculates indicators of ocular biomechanical properties,
including corneal hysteresis, which is the difference between IOP measured during the
initial corneal indentation and IOP measured during corneal rebound. Reduced corneal
hysteresis has been associated with an increased risk of developing visual field defects in
glaucoma suspects and with disease progression in patients with confirmed glaucoma.
None of the other instruments listed as answer options measure corneal hysteresis.
A corneal pachymeter is used to measure CCT. IOP measurement may be affected by a
thicker or thinner CCT. The dynamic contour tonometer has a tip that is similar in shape
and size to an applanation tonometer tip except that it has a rounded concave surface and
a pressure transducer embedded in the center. This allows the device to measure the ocular
pulse amplitude in addition to IOP. Evidence suggests that IOP measurements obtained with
dynamic contour tonometry may be more independent of corneal biomechanical proper-
ties and thickness than those obtained with applanation tonometry. The iCare tonometer
is based on a principle of rebound. Rebound tonometry determines IOP by measuring
the speed at which a small probe propelled against the cornea decelerates and rebounds
after impact (see Chapter 2, Fig 2-7D). IOP readings obtained with rebound tonometry are
strongly influenced by CCT.
6. a. Obtaining a careful medication history before prescribing new treatments, whether
topical or systemic, is essential. Patients with a known allergy to a sulfonamide antibiotic
may have an increased risk of a subsequent reaction to multiple classes of medications,
including carbonic anhydrase inhibitors (CAIs), although cross reactivity between sulfon-
amide antibiotics and sulfonamide nonantibiotics such as CAIs may be as low as 10%, as
they are different medication classes.
7. b. Ultrasound biomicroscopy is the best technique to image the ciliary body and ante-
rior choroid, and it can facilitate diagnosis of plateau iris syndrome, ciliary body cysts,
and ciliary tumors. Anterior segment optical coherence tomography does not penetrate
deeply enough to adequately image the ciliary body. B-scan ultrasonography is not ideal
Answers ● 305
for imaging anterior structures. Computed tomography scans do not provide detailed im-
ages of anterior segment structures.
8. d. Examination of the optic nerve head (ONH) should involve careful comparison with the
fellow eye, because cup–disc ratio asymmetry >0.2 between the 2 eyes is unusual in healthy
eyes in the absence of ONH size asymmetry (see Chapter 5, Fig 5-1). The vertical cup–disc
ratio typically ranges from 0.1 to 0.4, although up to 5% of individuals without glaucoma
have cup–disc ratios >0.6 (ie, physiologic cupping). Cup–disc ratio asymmetry values >0.2
occur in fewer than 1% of individuals without glaucoma. This asymmetry may be related to
ONH size asymmetry. Increased size of the physiologic cup may be a familial trait, which
may be clarified by examination of other family members.
9. b. Glaucomatous cups are associated with both generalized and focal ophthalmoscopic signs.
Generalized signs include a large optic cup, asymmetry of the optic cup between the eyes,
and progressive enlargement of the cup. Focal signs include notching of the neuroretinal
rim, vertical elongation of the optic cup, retinal nerve fiber layer hemorrhage, and segmental
nerve fiber layer loss. Less specific signs may also be observed, including exposed lamina
cribrosa, nasal displacement of rim vessels, baring of circumlinear vessels, and peripapillary
atrophy.
10. d. In this scenario, before treatment is started, it would be best to obtain at least 1 more
visual field test to determine whether the scotoma is reproducible. If the defect has a typi-
cal glaucomatous pattern matching the optic nerve appearance, neuroimaging is usually
not required. ONH imaging is important in diagnosis and management of glaucoma, but
repeating the visual field test would be the most appropriate next step to confirm a defect
and obtain a baseline.
Visual field testing is a subjective examination, and different responses may be ob-
tained each time the test is performed or even during the same test. Such fluctuation can
confound the detection of disease progression. To detect true visual field progression, the
clinician needs to evaluate whether the observed change exceeds the expected variability
for a particular point or area.
11. d. A false-positive error occurs when the patient presses the response button even though
no stimulus is presented. A false-negative error occurs when the patient does not respond
to a stimulus that should have been seen based on prior responses at that location. Fixation
losses can be estimated by periodically presenting stimuli within the physiologic blind spot.
Patients who see these stimuli are presumed to be looking away from the fixation target.
Movement of the eyes from the central fixation point can be detected by modern perimeters,
which possess a gaze tracker that monitors pupil location throughout the test.
12. c. The Collaborative Normal-Tension Glaucoma Study (CNTGS) was a multicenter ran-
domized controlled clinical trial comparing observation and treatment (30% reduction
in IOP) for normal-tension glaucoma. The study found that lowering IOP by at least 30%
decreased the 5-year risk of visual field progression from 35% to 12%, after adjusting for
the effect of cataract.
13. b. Elevated IOP is a significant risk factor for the development of glaucoma, but it is not
required for a primary open-angle glaucoma (POAG) diagnosis. Several studies have
shown that between 30% and 50% of individuals in the general population with glauco-
matous optic neuropathy and/or visual field loss present with an initial IOP measurement
below 22 mm Hg. In addition, IOP can vary considerably over a 24-hour period.
306 ● Answers
14. b. The Baltimore Eye Survey found that the prevalence of glaucoma increases consider-
ably with age, particularly among Black individuals. In this group, prevalence exceeded
11% in people 80 years of age and older. Ocular perfusion was not studied in the Baltimore
Eye Survey. The Ocular Hypertension Treatment Study found an increased risk of conver-
sion from ocular hypertension to glaucoma with age (per decade). The Collaborative Ini-
tial Glaucoma Treatment Study found that visual field defects (ie, glaucoma progression)
were 7 times more likely to progress in patients aged 60 years or older than in patients
younger than 40 years.
15. c. Interlenticular opacification and iris chafing are complications that may occur with
implantation of piggyback intraocular lenses within the capsular bag. Iris chafing can
result in pigment dispersion syndrome and associated elevated IOP, presumably from ob-
struction of the trabecular meshwork by pigment granules. On gonioscopy, the trabecular
meshwork commonly appears as homogeneous and densely pigmented, with speckled
pigment at or anterior to the Schwalbe line, forming a Sampaolesi line. Other findings
associated with pigment dispersion syndrome include pigment deposits on the corneal
endothelium (Krukenberg spindle); midperipheral iris transillumination defects; and pig-
ment deposits on the zonular fibers, on the anterior hyaloid, and in the equatorial region
of the lens capsule (Zentmayer line/ring, or Scheie stripe). The findings listed as other
answer choices are not associated with pigment dispersion syndrome.
16. c. The most likely diagnosis is phacolytic glaucoma, caused by leakage of high-molecular-
weight proteins through microscopic openings in the lens capsule of a mature or hyperma-
ture cataract. These proteins obstruct the trabecular meshwork, resulting in elevated IOP.
The lack of keratic precipitates helps distinguish phacolytic glaucoma from phacoantigenic
glaucoma. Phacoantigenic glaucoma occurs because of sensitization of the eye to native lens
protein, causing a granulomatous reaction. Lens particle glaucoma occurs because of reten-
tion of lens material after cataract surgery or ocular trauma. Phacomorphic glaucoma is
induced by a relatively large lens pushing the iris forward, causing angle closure.
17. d. Miotic agents are not recommended in patients with anterior uveitis because they may
exacerbate the inflammation and result in the formation of central posterior synechiae.
Topical CAIs, α-agonists, and β-blockers are all effective medications in the treatment of
uveitic glaucoma.
18. c. Corticosteroid-induced glaucoma is an open-angle glaucoma caused by the use of topi-
cal, periocular, intravitreous, inhaled, or oral corticosteroids. Approximately 33% of the
population without glaucoma demonstrates a mild (6–15-mm-Hg) increase in IOP with
corticosteroid use, and 4%–6% of this population has significant IOP elevation of more
than 15 mm Hg. In contrast, up to 95% of individuals with POAG develop an ocular
hypertensive response to topical corticosteroids. The type and potency of the agent, the
route and frequency of its administration, and the susceptibility of the patient all affect
the timing and extent of the IOP rise. Risk factors for corticosteroid-induced glaucoma
include a history of POAG, a first-degree relative with POAG, very young age (<6 years),
or older age.
19. c. Although angle closure resulting from rotation of the ciliary body, deposition of tumor
cells and inflammatory cells in the trabecular meshwork, direct invasion of the anterior
chamber angle, and neovascularization of the angle can all cause glaucoma associated with
primary or metastatic tumors of the ciliary body, direct invasion of the anterior chamber
angle is the most common cause of glaucoma in this setting. This can be exacerbated by an-
Answers ● 307
terior segment hemorrhage and inflammation, which further obstruct aqueous outflow. Ne-
crotic tumor and tumor-laden macrophages obstruct the trabecular meshwork and result in
a secondary OAG. Tumors causing a secondary glaucoma in adults include uveal melanoma
and melanocytoma (see Chapter 8, Figs 8-10 and 8-11), metastatic carcinoma, lymphoma,
and leukemia.
20. d. This patient has characteristic features of glaucomatocyclitic crisis (also called Posner-
Schlossman syndrome). Patients with this uncommon form of open-angle inflammatory
glaucoma are often mildly or minimally symptomatic despite markedly elevated IOP. On
examination, only a few discrete keratic precipitates may be present. Episodes are usually
unilateral and recurrent, with normalization of IOP in between attacks. Although attacks
are episodic, patients can develop glaucomatous ONH damage and visual field loss. In some
cases, filtering surgery is performed to prevent IOP spikes in eyes with advanced optic nerve
damage or in those experiencing frequent attacks. Compared with glaucomatocyclitic cri-
sis, herpetic uveitis is generally associated with more inflammation and discomfort. Fuchs
uveitis syndrome (formerly, Fuchs heterochromic iridocyclitis) is an insidious and chronic
form of uveitis without episodic attacks of elevated IOP. Angle-closure glaucoma is associ-
ated with pain and a shallow anterior chamber.
21. c. Primary angle-closure glaucoma has been estimated to account for over 90% of blind-
ness due to glaucoma in the Chinese population.
22. c. The Zhongshan Angle-Closure Prevention (ZAP) trial identified 889 primary angle-
closure suspects. One eye of each subject was randomly assigned to laser peripheral iri-
dotomy (LPI), and the contralateral eye served as a control. The study results showed that
very few cases in either group progressed to primary angle-closure glaucoma or an acute
attack of angle closure. There was a significantly lower risk of conversion to primary angle
closure with treatment. Most cases of conversion to primary angle closure were attributed to
formation of peripheral anterior synechiae (PAS) alone.
23. a. Scleral buckles (especially the encircling bands) used for retinal detachment repair can
produce angle narrowing and frank angle closure, often accompanied by choroidal effu-
sion and anterior rotation of the ciliary body that result in flattening of the peripheral iris
with a relatively deep central anterior chamber. With medical therapy (cycloplegics, anti-
inflammatory agents, β-adrenergic antagonists, CAIs, and hyperosmotic agents), the an-
terior chamber usually deepens with opening of the angle over days to weeks. Injection of
expansile gas or silicone oil into the eye after pars plana vitrectomy can cause angle closure
through forward movement of the lens and/or iris. In general, choroidal effusions are not
present in this situation. A surgical iridectomy can be done at the time of the vitrectomy. If a
component of pupillary block is present, an LPI may be subsequently performed and benefi-
cial. If performed, the iridotomy or iridectomy should be located inferiorly. Less commonly,
the gas or oil may have to be removed. Anti–vascular endothelial growth factor injections
can cause elevated IOP, but the mechanism is not due to angle closure.
24. b. Neovascular glaucoma (NVG) is a relatively common outcome of central retinal vein
occlusion. This patient appears to have an advanced stage of NVG. In this case, given the
poor visual prognosis, transscleral cyclophotocoagulation (CPC) may be considered as an
alternative to filtering surgery or tube shunt implantation for control of elevated IOP. Com-
plications during filtering surgery or tube shunt implantation often occur in active NVG, so
these treatment options are not appropriate for this patient. Lowering the IOP with CPC will
likely improve the eye pain. Medical management of the IOP may be instituted at the same
308 ● Answers
in the trabecular meshwork and Schlemm canal, allowing increased aqueous outflow. Rho
kinase inhibitors also decrease EVP.
Prostaglandin analogues (eg, latanoprost) lower IOP primarily by increasing uveo-
scleral outflow; they also increase conventional outflow. Topical CAIs (eg, dorzolamide),
β-blockers (eg, timolol), and α2-adrenergic agonists (eg, brimonidine) decrease aqueous
humor production. Hyperosmotic agents (eg, mannitol) increase blood osmolality, thus cre-
ating an osmotic gradient to draw water from the eye.
31. c. A valved plate-based tube implant is the best option for this patient, as the patient has
not been seen by an ophthalmologist in the past year and has risk factors that jeopardize
the postoperative outcome for a procedure requiring postoperative intervention (as with
trabeculectomy). For these reasons, less invasive procedures would also be good; however,
because of the disease severity and IOP elevation, this patient likely requires subconjunc-
tival filtering surgery.
Key considerations when making decisions for the management plan include glaucoma
severity, life expectancy, and the presence of any major systemic disease that could affect the
outcome. Another key consideration is whether the patient can adhere to medical therapy.
For example, a patient who has poor preoperative medication adherence (because of mem-
ory loss, poor vision, tremor, or arthritis) is likely to also display poor postoperative adher-
ence to medical therapy. The surgeon should recommend to the patient the procedure that is
most likely to successfully reduce IOP with the fewest possible complications. This selection
involves considering the patient’s ability to return for multiple follow-up visits.
It is important to minimize risk of surgery in older adult patients through careful pre-
operative considerations for anesthesia and detailed preoperative evaluation for any po-
tential systemic comorbidities that may affect decisions for the management plan. For this
patient, minimizing and simplifying the use of postoperative medications may be impor-
tant, as well as reducing the frequency of office visits by choosing a procedure that requires
the least intensive postoperative management.
32. c. When the anterior chamber shallows postoperatively in the absence of overfiltration
or choroidal effusions, there should be a high index of suspicion for malignant glaucoma
(also known as aqueous misdirection). This rare condition is seen most often after tra-
beculectomy performed for treatment of angle-closure glaucoma. Patients with malignant
glaucoma often have elevated IOP; however, the pressure can be “normal.”
Cycloplegic therapy should be initiated in an attempt to normalize the anatomy. Re-
formation of the anterior chamber is avoided, as the increased posterior pressure will pre-
vent this treatment from being effective. The placement of an oversized contact lens can
provide a scaffold for re-epithelialization and may help to stop a bleb leak; with a bleb leak,
the IOP invariably will be low in the early postoperative period. Steroids are often increased
in the setting of hypotony/choroidal effusions to decrease inflammation.
Study Questions
Please note that these questions are not part of your CME reporting process. They are provided
here for your own educational use and for identification of any professional practice gaps. The
required CME posttest is available online (see “Requesting Continuing Medical Education
Credit”). Following the questions are answers with discussions. Although a concerted effort
has been made to avoid ambiguity and redundancy in these questions, the authors recognize
that differences of opinion may occur regarding the “best” answer. The discussions are pro-
vided to demonstrate the rationale used to derive the answer. They may also be helpful in con-
firming that your approach to the problem was correct or, if necessary, in fixing the principle
in your memory. The Section 11 faculty thanks the Resident Self-Assessment Committee for
developing these self-assessment questions and the discussions that follow.
259
260 ● Study Questions
5. What carbohydrate metabolism–related change occurs in the lens in the presence of high
levels of glucose?
a. Aldose reductase activity increases.
b. Glucose that is not phosphorylated to glucose-6-phosphate (G6P) enters the Krebs
cycle.
c. The hexose monophosphate shunt is deactivated.
d. Sorbitol is eliminated from the lens via simple diffusion through the capsule into the
aqueous humor.
6. The lens has higher levels of what electrolyte in comparison to the surrounding aqueous
and vitreous humors?
a. chloride ions (Cl−)
b. potassium ions (K+)
c. sodium ions (Na+)
d. calcium (Ca2+)
9. Glaukomflecken are seen on examination. What is the histologic finding and most likely
clinical scenario?
a. endothelial cells associated with Cogan-Reese syndrome
b. gray-white fibrillar material from pseudoexfoliative glaucoma
c. macrophages engorged with phagocytosed proteinaceous, eosinophilic lens material
from phacolytic glaucoma
d. necrotic lens epithelial cells (LECs) and degenerated subepithelial cortex following
angle-closure glaucoma
Study Questions ● 261
10. A 14-year-old female patient with atopic dermatitis has experienced decreased vision in
both eyes. Her best-corrected visual acuity (BCVA) is 20/100 OD and 20/80 OS. What is
the most likely cause of her decreased vision?
a. anterior subcapsular cataract
b. dry eye syndrome
c. cystoid macular edema
d. lens subluxation
11. A 47-year-old female vegan web developer spends 16 hours per day working on the com-
puter, takes high-dose vitamin C supplements, and drinks an average of 3 glasses of wine
each evening. She describes gradual, progressive bilateral blurry vision and has visually
significant nuclear cataract in both eyes. Which of her lifestyle habits predisposed her to
early cataract development?
a. prolonged computer use
b. veganism
c. vitamin C supplementation
d. alcohol consumption
12. After an episode of uveitis, what is the minimum amount of time, ideally, that the eye
should be free from inflammation, without the use of topical corticosteroids, before cata-
ract surgery is performed?
a. 1 week
b. 1 month
c. 2 months
d. 3 months
13. A patient with nystagmus has visually significant cataract and would like to undergo cata-
ract surgery. He is capable of lying flat for surgery and prefers not to undergo general
anesthesia. Aside from general anesthesia, what type of anesthesia would provide the best
akinesia for his cataract surgery?
a. intracameral anesthesia
b. sub-Tenon anesthesia
c. peribulbar anesthesia
d. retrobulbar anesthesia
262 ● Study Questions
14. A patient with high axial myopia and regular against-the-rule astigmatism underwent
uncomplicated cataract surgery with a toric intraocular lens (IOL). She was very pleased
with her uncorrected vision in the operative eye on postoperative day 1, so immediately
resumed her work, which involves several hours of computer use per day. She also re-
sumed walking 2 miles per day on postoperative day 2. At postpoperative week 1, she
noticed a reduction in the clarity of her uncorrected vision and was found to have rota-
tion of her toric IOL off-axis. What most likely contributed to her postoperative toric IOL
rotation?
a. against-the-rule astigmatism
b. high axial myopia
c. excessive visual tasking
d. walking long distances
15. What layers of the lens are separated from each other during hydrodelineation?
a. capsule and cortex
b. cortex and the epinucleus
c. capsule and the nucleus
d. epinucleus and the endonucleus
16. What type of effect (ie, direct or indirect) does a peristaltic pump (or “flow pump”) of a
phacoemulsification machine have on the aspiration flow rate (AFR) and vacuum?
a. It directly controls the AFR and directly produces the vacuum level.
b. It indirectly controls AFR and directly produces the vacuum level.
c. It directly controls AFR and indirectly produces the vacuum level.
d. It indirectly controls the AFR and indirectly produces the vacuum level.
17. In manual small incision cataract surgery (MSICS), how does the width of the external
scleral incision compare to the internal corneal incision?
a. The width of the external scleral incision is greater than the width of the internal cor-
neal incision.
b. The width of the external scleral incision is equal to the width of the internal corneal
incision.
c. The width of the external scleral incision is smaller than the width of the internal cor-
neal incision.
d. The proportion of the external scleral incision to the internal corneal incision depends
on the density of the nucleus being expressed.
Study Questions ● 263
18. An ophthalmologist is performing a routine phacoemulsification when she notes that the
remaining half of the nucleus has suddenly dropped through an open posterior capsule
and appears to be suspended on the anterior vitreous face. What is the best next step?
a. immediate withdrawal of the phaco instrument and any secondary instrument from
the eye
b. injection of an ophthalmic viscosurgical device (OVD) to stabilize the anterior chamber
c. lollipopping the nucleus with the phaco tip in order to pull it into the anterior chamber
d. vigorous irrigation with a balanced salt solution to flush the remaining nucleus into the
anterior chamber
19. During phacoemulsification, the surgeon notes loss of anterior chamber depth; the eye
becomes firm, and the patient reports feeling pain. What should the surgeon do next?
a. Ask the anesthesiologist to increase the patient’s intravenous pain medication.
b. Decrease the infusion pressure and increase the aspiration rate.
c. Prepare for a pars plana vitrectomy.
d. Suture the wounds closed and examine the fundus.
20. An eye with extreme increased axial length may be susceptible to what intraoperative
issue?
a. higher risk of endothelial damage
b. higher risk of iris prolapse
c. higher risk of wound burn
d. lens–iris diaphragm retropulsion syndrome (LIDRS)
21. If vitreous prolapse occurs, what is the recommended technique for anterior vitreous re-
moval by an anterior segment surgeon?
a. coaxial anterior vitrectomy through the main corneal incision
b. complete vitrectomy and lensectomy through 2 pars plana incisions
c. manual externalization and cutting of vitreous through the main corneal incision
d. 2-port bimanual anterior vitrectomy with instruments through new limbal incisions or
cutting instrument through pars plana
22. After cataract surgery, a patient presents with a high intraocular pressure (IOP). On ex-
amination, the anterior chamber is shallow, the IOL is in the capsular bag, and there is
no space between the IOL and the posterior capsule. A laser peripheral iridotomy fails to
deepen the chamber or lower the IOP. What is the next most appropriate treatment?
a. cyclophotocoagulation
b. cycloplegia and aqueous suppression
c. surgical vitrectomy
d. topical miotics and a second laser peripheral iridotomy
264 ● Study Questions
23. A 74-year-old male patient undergoes uncomplicated cataract surgery and has a round-
edge IOL placed in the capsular bag. The anterior capsulorrhexis was noted to be smaller
than the optic of the IOL that was placed. Following cataract surgery, the patient was on a
longer-than-usual course of topical corticosteroid for prophylaxis against cystoid macu-
lar edema (CME) due to the patient’s pre-existing epiretinal membrane. In postopera-
tive month 3, he was noted to have a visually significant posterior capsule opacification
(PCO). What put him at greatest risk of developing a PCO?
a. advanced age
b. size of the capsulorrhexis
c. longer course of topical corticosteroid postoperatively
d. round-edge IOL
24. In postoperative month 1 following uncomplicated cataract surgery with a 1-piece pos-
terior chamber IOL (PCIOL), a patient notices a persistent dark crescent in the temporal
visual field of the operative eye. Confrontation visual fields are full in both eyes, and re-
sults of a dilated fundus examination in the operative eye are normal. What is the most
appropriate next step in management?
a. Observation.
b. Perform a Nd:YAG laser posterior capsulotomy.
c. Perform reverse optic capture of the existing lens.
d. Exchange the 1-piece PCIOL for a 3-piece PCIOL in the sulcus.
25. What criterion determines the type of intervention that should be performed for acute
post–cataract extraction endophthalmitis, based on the results of the Endophthalmitis
Vitrectomy Study (EVS)?
a. amount of time that has passed since surgery
b. degree of inflammation in the vitreous
c. presence of intact posterior capsule
d. visual acuity
26. A patient has decreased visual acuity secondary to CME 1 month after uncomplicated
cataract surgery. What is the next most appropriate step in management?
a. observation
b. treatment with topical corticosteroids and/or NSAIDs
c. intravitreal injection of corticosteroids
d. intravitreal injection of vascular endothelial growth factor inhibitors
Study Questions ● 265
27. During phacoemulsification, the surgeon realizes that there are 3 clock-hours (90°) of
zonular dialysis present. How should the surgeon proceed?
a. Increase the bottle height and flow rate to maintain adequate anterior chamber depth.
b. Close the corneal incision and convert to an extracapsular cataract extraction.
c. Proceed with phacoemulsification and place an anterior chamber intraocular lens
(ACIOL).
d. Place a capsular tension ring or capsular hooks, complete phacoemulsification, and
place a PCIOL.
28. What surgical step should be avoided in a patient with a posterior polar cataract?
a. hydrodissection
b. hydrodelineation
c. Little capsulorrhexis rescue technique
d. viscodissection with OVD
30. For a patient with mild glaucoma and visually significant cataract, what is the aver-
age extent of IOP lowering that may be expected from phacoemulsification and IOL
implantation?
a. <10%
b. 10%–34%
c. 35%–49%
d. >50%
Answers
1. b. Diabetes mellitus is associated with accelerated development of all age-related lens
changes. The proposed pathophysiologic reasons for these changes are multifactorial, in-
cluding increased lens hydration from sorbitol accumulation, glycation of lens proteins,
and increased oxidative stress. Multiple sclerosis, hypercholesterolemia, and ischemic car-
diovascular disease are not associated with increased risk for developing cataracts.
2. d. The oldest layers of the lens are located in the center of the lens, or the endonucleus.
This occurs because the new lens fibers are laid down in the periphery of the lens, com-
pacting the previously formed fibers in the center of the lens. The outermost fibers of the
lens are therefore the newest and make up the cortex of the lens. Of note: though there is
no morphologic difference between the cortex, nucleus, epinucleus, or endonucleus, these
structures appear different and behave differently during surgery.
3. d. After fetal development, the lens has no blood supply or organelles and depends on the
aqueous humor for removal of metabolic waste. Changes in lens size occur throughout life
as the lens epithelial cells (LECs) at the equator continue to divide.
4. d. Superoxide dismutase, catalase, and glutathione perioxidase work together to destroy
the superoxide anion (O2−), protecting the lens against oxidative and free radical damage.
There are no known repair mechanisms for free radical damage to proteins or membrane
lipids in the lens cortex. Glutathione acts as a major free radical scavenger, along with
vitamin E and ascorbic acid, in the lens. These 3 enzymes have no effect on the pH of the
lens and do not participate in the carbohydrate metabolism of the lens. Conversion from
water-soluble proteins to water-insoluble proteins appears to be a natural (age-related)
process in lens fiber maturation and does not involve an enzymatic function.
5. a. When glucose concentration increases in the lens, the sorbitol pathway is activated; al-
dose reductase, which is the key enzyme in this pathway, is increased. Glucose that is not
phosphorylated to glucose-6-phosphate (G6P) enters the sorbitol pathway, not the Krebs
cycle. The hexose monophosphate shunt is also stimulated when glucose concentration
in the lens is increased. Sorbitol accumulates in the lens both because of a low rate of me-
tabolism and poor lens permeability, promoting lens opacification.
6. b. The lens has higher concentrations of potassium (K+) and amino acids than the sur-
rounding aqueous and vitreous humors. The lens has lower concentrations of sodium
(Na+), chloride (Cl−), and calcium (Ca2+) compared with the surrounding aqueous and
vitreous. Recall that the Na+/K+-ATPase is located on the anterior lens capsule and creates
a gradient of elevated intralenticular K+ and elevated extralenticular Na+.
7. d. Microspherophakia, a developmental abnormality in which the lens is small in diameter
and spherical, is most often associated with Weill-Marchesani syndrome. It can also be as-
sociated with a variety of diseases, including Peters anomaly, Marfan syndrome, Alport
syndrome, Lowe syndrome, and congenital rubella. It is not associated with Axenfeld-
Rieger syndrome, galactosemia, or Axenfeld anomaly.
8. b. Trauma is the most common cause of acquired lens displacement. Aniridia is associated
with lens opacities, poor zonular integrity, and ectopia lentis, but it is an uncommon pan-
ocular syndrome. Marfan syndrome and homocystinuria are also associated with ectopia
lentis, but they are inherited disorders and less common than trauma as etiology for lens
displacement.
267
268 ● Answers
9. d. Glaukomflecken are composed of necrotic lens epithelial cells (LECs) and degenerated
subepithelial cortex that result from prolonged elevated intraocular pressure (IOP), as in
acute angle-closure glaucoma. Glaukomflecken are not associated with the other findings
and scenarios presented. Pseudoexfoliation syndrome is a systemic condition that is char-
acterized by the production and progressive accumulation of a fibrillar material in tissues
throughout the anterior segment and in the connective tissue of various visceral organs.
10. a. Atopic dermatitis is associated with cataract formation in up to 38% of affected patients.
The cataracts are most often bilateral, anterior or posterior subcapsular opacities that re-
semble shieldlike plaques and usually develop in the second or third decade of life. Dry
eye syndrome, cystoid macular edema, and lens subluxation are not associated with atopic
dermatitis.
11. d. Excessive alcohol consumption (more than 14 standard drinks per week for men and
7 standard drinks per week for women) increases the risk for nuclear cataract. Computer
use, veganism, and vitamin C supplementation are not associated with cataract develop-
ment. Other lifestyle habits that increase the risk of cataract are tobacco use (both smoked
and smokeless forms), long-term exposure to sunlight (due to damage from ultraviolet
radiation), and high-dose vitamin B supplementation (more than 10 times the recom-
mended daily allowance).
12. d. Ideally, the eye should be quiet without the use of topical corticosteroids after an epi-
sode of uveitis for at least 3 months before cataract surgery in order to minimize the risk of
complications from postoperative inflammation, such as iris adhesion to the lens implant
and macular edema.
13. d. Retrobulbar anesthesia provides excellent akinesia and anesthesia. Though peribulbar
and sub-Tenon anesthesia can also provide akinesia, these methods may not provide aki-
nesis as complete as retrobulbar injection does. Intracameral anesthetic does not provide
akinesia during surgery.
14. b. High axial myopia predisposed this patient to postoperative toric IOL rotation, due to
both the larger capsular bag size and the thinner optics of the lower-spherical-power IOL
needed for myopes. Alignment of the toric IOL to correct with-the rule astigmatism, not
against-the-rule astigmatism, also increases the risk of postoperative rotation. Excessive
visual tasking is not associated with postoperative toric IOL rotation. Though vigorous
physical activity can increase the risk of postoperative toric IOL rotation, walking would
not.
15. d. Hydrodelineation is used to separate the harder endonucleus from the softer epinu-
cleus, allowing the epinucleus to cushion the posterior capsule during phacoemulsifica-
tion. Hydrodissection, on the other hand, is used to separate the cortex from the lens
capsule, not only loosening the lens–cortex complex, but also facilitating nucleus rotation
during phacoemulsification.
16. c. A peristaltic pump directly creates flow with a set of rollers that move along flexible
tubing, pushing fluid through the tubing. The pressure differential between the lower-
pressure aspiration tubing and the higher-pressure anterior chamber creates a relative
vacuum. Although a vacuum limit is set on the machine, the peristaltic pump does not
directly produce the level of vacuum. Rather, it controls the aspiration flow rate (AFR),
which indirectly produces vacuum. In contrast, a Venturi (or vacuum) pump directly cre-
ates the vacuum based on the Venturi effect. Direct control of vacuum level in the pump
Answers ● 269
cassette then indirectly produces flow (while the aspiration port is not occluded) by “pull-
ing” on the fluid in the aspiration tubing. In the absence of significant occlusion, higher
vacuum levels produce a faster AFR.
17. c. In manual small incision cataract surgery (MSICS), the scleral tunnel is shaped like
a trapezoid, such that the external scleral incision is narrower than the internal corneal
incision, allowing for the delivery of the nucleus while maintaining a self-sealing external
incision. The density of the nucleus being expressed does not affect the relative widths of
the external scleral incision and the internal corneal incision.
18. b. If capsule rupture occurs during phacoemulsification, lens fragments may enter the
posterior segment. At the time of posterior capsule rupture, the surgeon should stabilize
the anterior chamber by reducing the high fluid flow and vacuum levels and by compart-
mentalizing the vitreous with an ophthalmic viscosurgical device (OVD) before removing
the phaco instrument. The surgeon should avoid immediate withdrawal of the phaco in-
strument from the eye because that would result in an outward pressure gradient, bringing
more vitreous forward into the anterior chamber and outward toward the incisions. OVD
can also be introduced posterior to a suspended nuclear fragment in an effort to float it
anteriorly. Insertion of a second instrument or lens glide behind the nuclear remnant
may help prevent the remnant from being dislocated into the vitreous, whereas attempts
to impale the nuclear fragment with the phaco tip may force the fragment posteriorly by
either the tip or the irrigation, as might also occur with an attempt to flush the fragment
with vigorous balanced salt solution irrigation. Lollipopping the nuclear fragment with
the phaco tip may force it back and may transect the fragment, exposing the aspiration
port and increasing the risk of aspirating vitreous.
19. d. With a suprachoroidal hemorrhage, the eye typically becomes very firm, and the pa-
tient becomes agitated and reports having pain. The surgeon should immediately close the
incisions and confirm the diagnosis by examining the fundus with an indirect ophthalmo-
scope or fundus lens. If the hemorrhage or effusion is significant, the operation should be
postponed.
20. d. Eyes with very high axial lengths are at risk for lens–iris diaphragm retropulsion syn-
drome (LIDRS), in which a reverse pupillary block occurs, and the anterior chamber be-
comes very deep, causing pain. Eyes with lax zonular fibers are more prone to this issue.
Lifting the iris off the capsule with a second instrument and lowering the infusion bottle
might help correct this problem. Endothelial damage and iris prolapse are more com-
monly associated with small eyes. Wound burn is more commonly associated with dense
nuclei.
21. d. The surgeon should avoid manually externalizing and cutting vitreous through the
incision. A 2-port bimanual anterior vitrectomy can be performed with separate infusion
and aspirating/cutting instruments inserted through new, properly sized limbal incisions.
Alternatively, the aspiration/cutting instrument may be placed through a pars plana inci-
sion while irrigation is continued through the limbus. This directs flow posteriorly and
reduces the amount of vitreous that migrates into the anterior segment, thereby decreas-
ing vitreoretinal traction. Coaxial anterior vitrectomy is no longer recommended.
22. b. Malignant glaucoma (also known as ciliary block glaucoma, aqueous misdirection,
or vitreous block) results from ciliolenticular block induced by anterior movement
of the lens–iris diaphragm, poor vitreous fluid conductivity, and choroidal expansion.
These factors result in a shallow anterior chamber and secondary elevation of IOP as a
270 ● Answers
consequence of angle obstruction. IOP remains elevated despite the presence of a patent
iridectomy or iridotomy. Medical treatment consists of cycloplegia and aqueous suppres-
sion. Use of miotics is not recommended because they can worsen malignant glaucoma by
exacerbating anterior displacement of the lens–iris interface. Surgical intervention con-
sists of Nd:YAG laser irido-zonulo-hyaloidotomy or vitrectomy to disrupt the anterior
vitreous face and vitreous–ciliary body interface.
23. d. The round-edge design of the IOL optic put the patient at greatest risk of developing a
posterior capsule opacification (PCO). The truncated square-edge optic design of an IOL
is associated with lower rates of PCO formation; however, this type of IOL can increase the
chance of developing positive dysphotopsias and other optical reflections. Other factors
that can increase the rate of PCO include: younger patient age, history of intraocular in-
flammation, pseudoexfoliation syndrome, a larger capsulorrhexis (such that the anterior
capsule does not cover 360° of the IOL edge), incomplete cortical cleanup, more time
elapsed since surgery, and the presence of silicone oil. There is no difference in PCO rates
with prolonged use of topical corticosteroids or nonsteroidal anti-inflammatory drugs
(NSAIDs).
24. a. This patient is describing a negative dysphotopsia, which is common in the early post-
operative period, affecting approximately 15% of all patients. Negative dysphotopsias
usually improve over time, due to anterior capsular opacification and neuroadaptation;
therefore, observation is advised initially. Only 3% of patients report symptoms 1 year
after surgery. For patients with persistent symptoms, surgery can be offered, either by
reverse optic capture of the existing posterior chamber IOL (PCIOL) or placement of an
appropriate PCIOL in the sulcus. Nd:YAG posterior capsulotomy is not indicated for the
treatment of negative dysphotopsia.
25. d. As a result of the Endophthalmitis Vitrectomy Study (EVS) findings, immediate pars
plana vitrectomy and antibiotic injections are recommended when the patient’s visual
acuity is light perception. When the visual acuity is hand motions or better, a less-invasive
anterior chamber and vitreous biopsy for cultures, with subsequent intravitreal injection
of antibiotics, is sufficient. Time passed since surgery, degree of vitreous inflammation,
and presence of intact posterior capsule were not used to determine management in the
EVS study.
26. b. Topical anti-inflammatory drugs are the typical first-line treatment for CME and may
take up to 6 months to resolve chronic cystoid macular edema (CME). Spontaneous reso-
lution of CME occurs in approximately 95% of cases but may take 3 to 12 months. Though
intravitreal injections of corticosteroids or vascular endothelial growth factor inhibitors
can be successful in treating chronic CME, these treatments are usually reserved for those
cases not responding to topical treatment.
27. d. For a patient with 90° of zonular dialysis, management consists of continued phaco-
emulsification and capsular tension ring (CTR) placement. The timing of the CTR place-
ment depends on the extent of the zonular compromise and the surgeon’s preference.
Placement of the CTR before completion of phacoemulsification stabilizes the capsule
for further lens manipulation and extraction but makes cortical aspiration more difficult.
Thorough cortical aspiration is easier if CTR placement is delayed until just before IOL
insertion. However, there is a risk of extending the zonular dialysis if phacoemulsification
is performed without adequate capsule support. The flow rate and bottle height should
not be increased, because doing so could cause vitreous to prolapse through the dialysis
Answers ● 271
and into the anterior chamber. OVD may be used as a tamponade against the forward
movement of vitreous in the area of the dialysis. Unless the situation were to deterio-
rate further, neither conversion to extracapsular cataract extraction nor use of an anterior
chamber IOL (ACIOL) would be necessary.
28. a. Hydrodissection should be avoided in a posterior polar cataract due to the possibility
of tearing the posterior capsule directly under the region of the posterior polar opacity,
because the posterior capsule may be weak or absent in this area. Gentle hydrodelineation,
however, can be performed in order to leave an epinuclear bowl to protect the capsule
during nucleus removal. After nucleus removal, OVD can be used for viscodissection of
the epinucleus from the capsular bag. The Little capsulorrhexis rescue technique is not
contraindicated in this situation.
29. d. Nanophthalmos is a rare condition in which the eye is extremely short (axial length
< 20 mm) and the ratio of lens volume to eye volume is higher than normal. These eyes
have shallow anterior chambers, narrow angles, and thickened sclerae, with little room for
the surgeon to maneuver. Small-incision bimanual surgery is an alternative technique to
consider. Intraoperative or postoperative uveal effusion is a unique hazard in nanophthal-
mic eyes. Maintaining positive pressure in the anterior chamber and limiting the length
of the procedure help prevent intraoperative uveal effusion. Scleral windows should be
considered as a prophylactic measure to treat uveal effusion. A sutured wound prevents
hypotony from contributing to this complication postoperatively. Endophthalmitis, ir-
regular astigmatism, and rhegmatogenous retinal detachments are not common compli-
cations in these patients.
30. b. Uncomplicated phacoemulsification surgery lowers IOP between 10% and 34%, though
this decrease diminishes over time. It can therefore be considered as a stand-alone proce-
dure in eyes with mild to moderate glaucoma. Microinvasive glaucoma surgery (MIGS)
can also be performed at the time of cataract surgery to benefit patients with mild to mod-
erate glaucoma. For more advanced cases of glaucoma and visually significant cataract,
adjunctive filtration procedures may need to be considered at the time of cataract surgery.
Staged procedures can also be considered (either the cataract or glaucoma procedure first,
then the second procedure at later date).
Study Questions
Please note that these questions are not part of your CME reporting process. They are provided
here for your own educational use and for identification of any professional practice gaps. The
required CME posttest is available online (see “Requesting Continuing Medical Education
Credit”). Following the questions are answers with discussions. Although a concerted effort has
been made to avoid ambiguity and redundancy in these questions, the authors recognize that
differences of opinion may occur regarding the “best” answer. The discussions are provided to
demonstrate the rationale used to derive the answer. They may also be helpful in confirming that
your approach to the problem was correct or, if necessary, in fixing the principle in your memory.
The Section 12 faculty thanks the Resident Self-Assessment Committee for developing these self-
assessment questions and the discussions that follow.
2. Within the retina, the density of rods is greatest at what degree of fixation?
a. 90
b. 45
c. 12
d. 0
3. What is the most common systemic condition associated with angioid streaks?
a. oculocutaneous albinism
b. Crohn disease
c. pseudoxanthoma elasticum
d. ulcerative colitis
4. What symptom following intravitreal injection would be unexpected and warrant imme-
diate evaluation?
a. progressive floaters with progressive blurring of vision
b. sharp pain following the injection
c. foreign body sensation
d. small floaters that are noticed immediately after injection
463
464 ● Study Questions
5. A patient with long-standing poorly controlled diabetes presents with vision loss (20/200)
in 1 eye. Clinical examination shows mild inferior vitreous hemorrhage, minimal macular
edema outside the central macula, and a well-perfused optic nerve. Fluorescein angiog-
raphy demonstrates an area of neovascularization elsewhere and a foveal avascular zone
diameter of 1500 μm. What is the most likely cause of the poor vision?
a. ischemic maculopathy
b. capillary leakage
c. traction (tractional) detachment
d. bleeding from neovascularization
6. What traumatic extraocular condition is associated with a branch retinal artery occlusion?
a. pelvic fracture
b. femur fracture
c. subarachnoid hemorrhage
d. pancreatitis
7. What is the most significant risk factor for central retinal vein occlusion (CRVO)?
a. hypertension
b. older age
c. hypercoagulable state
d. glaucoma
8. What treatment has been proven to be effective for branch retinal vein occlusion–associated
macular edema?
a. panretinal photocoagulation
b. verteporfin
c. fluocinolone acetonide intravitreal implant
d. ranibizumab
9. What is the most common cause of decreased visual acuity in patients who present with
acute CRVO?
a. macular ischemia
b. macular edema
c. vitreous hemorrhage
d. neovascular glaucoma
10. A 5-year-old boy is found to have unilateral telangiectatic retinal vessels with an exudative
retinal detachment. What histologic findings are characteristic of the likely diagnosis?
a. “foamy” histiocytes and cholesterol crystals
b. loss of capillary pericytes and thickening of the retinal capillary basement membrane
c. dilated, thin-walled vascular channels between the retinal pigment epithelium (RPE)
and outer aspect of Bruch membrane
d. focal sclerosis of retinal vessels with an overlying pocket of liquefied vitreous
Study Questions ● 465
14. A 35-year-old otherwise healthy man presents with recent onset of unilateral slightly
blurred vision. On evaluation, the right eye has a visual acuity of 20/25, there is serous
subretinal fluid in the macula, and the choroid is thickened. The patient is minimally
symptomatic. What is the most appropriate initial management?
a. observation
b. topical steroids
c. photodynamic therapy
d. laser photocoagulation therapy
15. A 24-year-old woman presents with acute-onset, paracentral “flashing lights” in 1 eye; nasal
visual field loss by confrontation testing in both eyes; and normal visual acuity in both eyes.
Vitreous cells are present posteriorly in both eyes. The fundus is otherwise unremarkable.
Visual field, multifocal electroretinogram (mfERG), and optical coherence tomography
(OCT) testing reveal scotomata corresponding to focally decreased mfERG responses and
outer retinal layer disruption in both eyes. What is the most likely diagnosis?
a. multiple evanescent white dot syndrome (MEWDS)
b. acute zonal occult outer retinopathy (AZOOR)
c. acute macular neuroretinopathy (AMN)
d. intermediate uveitis
466 ● Study Questions
16. A 42-year-old man reports decreased vision in his right eye over the past 3 days. He does
not report significant pain or discomfort, but on review of systems, the ophthalmologist
discovers that he also has ulcers on the inside of his mouth. For which HLA haplotype
would the patient most likely be positive?
a. HLA-A29
b. HLA-B27
c. HLA-B51
d. HLA-DR4
17. Once the diagnosis of sympathetic ophthalmia is established, what is the preferred initial
management?
a. intraocular injection of foscarnet to the sympathizing eye
b. panretinal photocoagulation to the sympathizing eye
c. enucleation of the exciting eye
d. administration of oral corticosteroids
18. What bilateral white dot syndrome is characterized by acute vision loss that is typically
followed by full or near-full vision recovery?
a. acute posterior multifocal placoid pigment epitheliopathy (APMPPE)
b. birdshot chorioretinopathy (vitiliginous chorioretinitis)
c. MEWDS
d. serpiginous choroiditis
19. What is the best oral induction treatment option for a patient with varicella-zoster virus–
associated acute retinal necrosis?
a. acyclovir 800 mg 5 times daily
b. valacyclovir 1 g 3 times daily
c. valacyclovir 2 g 3 times daily
d. valganciclovir 900 mg 2 times daily
20. A 30-year-old woman presents with blurred vision and photopsias that started in her right
eye and then affected her left eye as well. She had a fever, malaise, and headache 1 week
ago. Her fundus examination reveals multiple yellow-white placoid lesions 1–2 disc areas
in size. What is the most likely diagnosis?
a. AZOOR
b. APMPPE
c. MEWDS
d. serpiginous choroiditis
Study Questions ● 467
21. What is the most common pattern of color confusion associated with acquired color vi-
sion defects?
a. blue-yellow
b. red-yellow
c. blue-green
d. red-green
23. What ophthalmologic diagnostic test is characteristically expected to give a normal result
in patients with Best disease?
a. fluorescein angiography (FA)
b. electroretinogram (ERG)
c. electro-oculogram (EOG)
d. OCT
26. Dermal erythema and bullae are a presenting feature of what retinal condition?
a. X-linked retinitis pigmentosa
b. severe form of enhanced S-cone syndrome (Goldmann-Favre syndrome)
c. gyrate atrophy
d. incontinentia pigmenti
468 ● Study Questions
28. Mutations in CYP4V2 may lead to characteristic crystalline deposits in the retina with
initially normal visual function but with enlarging regions of geographic-like paracentral
atrophy. What disorder does this mutation cause?
a. cystinosis
b. oxalosis
c. Bietti crystalline dystrophy
d. macular telangiectasia type 2
30. A male patient presents with photophobia, iris transillumination, hypopigmented fundi,
and nystagmus. His skin and hair have normal pigmentation. What is the most likely in-
heritance pattern of this patient’s condition?
a. X-linked recessive
b. autosomal dominant
c. autosomal recessive
d. X-linked dominant
31. A 60-year-old woman with unexplained peripheral neuropathy but no additional oph-
thalmic or medical history reports bilateral visually significant floaters. What is the most
likely diagnosis?
a. asteroid hyalosis
b. cholesterolosis
c. amyloidosis
d. chronic vitreous hemorrhage
32. A 1-year-old boy presents with peripheral traction retinal detachment with temporal dis-
placement of the maculae bilaterally. The child’s father is similarly affected. There is no
history of premature birth. What is the most likely diagnosis?
a. familial exudative vitreoretinopathy
b. congenital retinal telangiectasia
c. juvenile retinoschisis
d. incontinentia pigmenti
Study Questions ● 469
34. Atrophic retinal holes are commonly associated with what condition?
a. vitreous hemorrhage
b. retinal detachment
c. posterior vitreous detachment
d. lattice degeneration
35. In an eye with retinoschisis, what examination findings are associated with an increased
risk of progression to retinal detachment?
a. demarcation line
b. hyperopia
c. inner and outer layer holes
d. typical peripheral cystoid degeneration
40. What is the most common ophthalmic manifestation of abusive head trauma?
a. cranial nerve paresis
b. bilateral intraocular hemorrhages
c. papilledema
d. exudative retinal detachment
41. A 68-year-old man reports a “missing spot” in his vision 1 week after cataract extraction
by a first-year resident. Corrected distance visual acuity (also called best-corrected visual
acuity) is 20/100. In addition to normal postoperative findings, the examination is sig-
nificant only for subtle foveal hypopigmentation. Macular OCT demonstrates an outer
retinal lucency. What is the most likely diagnosis?
a. nonexudative age-related macular degeneration
b. photic retinopathy
c. macular hole
d. cystoid macular edema
42. A 19-year-old man is struck in the left eye with a paintball. Visual acuity is 20/200 OS.
Examination shows no afferent pupillary defect. Intraocular pressure is 18 mm Hg. There
is no subconjunctival hemorrhage. The anterior chamber is formed. Fundus examination
shows thickened, sheenlike retinal whitening in the macula. What is the most appropriate
management for this patient?
a. laser photocoagulation
b. anterior chamber paracentesis
c. high-dose intravenous corticosteroids
d. observation
43. What imaging modality best helps determine the optimal time to surgically drain a supra-
choroidal hemorrhage?
a. B-scan ultrasonography
b. FA
c. OCT
d. computed tomography
Study Questions ● 471
44. What surgical procedure is most indicated for a total retinal detachment with proliferative
vitreoretinopathy and multiple posterior retinal breaks?
a. vitrectomy
b. indirect laser retinopexy
c. pneumatic retinopexy with cryotherapy
d. scleral buckle
45. Nine weeks after routine cataract surgery, a patient develops a steroid-responsive granulo-
matous uveitis/endophthalmitis, with a white plaque on her intraocular lens. What would
be the most appropriate management of this condition?
a. injection of intravitreal antibiotics
b. observation
c. pars plana vitrectomy and partial capsulectomy with injection of intravitreal antibiotic
d. topical antibiotics and steroids
46. What agent used for retinal tamponade persists in the eye for approximately 2 months
after pars plana vitrectomy?
a. perfluoropropane gas
b. silicone oil
c. perfluorocarbon liquid
d. sulfur hexafluoride gas
Answers
1. c. Hyaluronic acid (hyaluronan) is the main glycosaminoglycan found in vitreous. Other
major components include water and collagen. Although dermatan, chondroitin, and
keratan are glycosaminoglycans, they are not the principal ones found in the vitreous.
2. c. The rod photoreceptors have their greatest density (approximately 160,000 rods/mm2)
within the retina at around 12° of fixation. Rod density decreases toward the peripheral
retina. Cone density is greatest (exceeding 140,000 cones/mm2) in the fovea. Within the
central fovea, only cones are observed. The fovea is defined as a concave central retinal
depression seen on slit-lamp examination; it is approximately 1.5 mm in diameter.
3. c. Angioid streaks have been associated with a number of systemic disorders and with
aging. The most common systemic association is with pseudoxanthoma elasticum (PXE),
a predominantly autosomal recessive disorder caused by a mutation in ABCC6. PXE re-
sults in calcification and other mineralization of elastin, most visibly as yellow papules in
the flexor regions of the skin (“plucked chicken” appearance). Angioid streaks are typically
more prominent adjacent to or can extend from the optic nerve head, in contrast to Bruch
membrane ruptures from trauma or high myopia (“lacquer cracks”). Other systemic dis-
eases associated with angioid streaks include Paget disease of bone, β-thalassemia, sickle
cell disease, and Ehlers-Danlos syndrome. Angioid streaks are not a feature of oculocu-
taneous albinism (OCA); patients with OCA may have depigmented skin spots but not
papules. Crohn disease and ulcerative colitis are associated with uveitis but not angioid
streaks.
4. a. Vitritis, or inflammation of the vitreous, is the first presenting feature of injection-
related endophthalmitis. It is perceived by patients as progressive floaters and blurring of
vision, often culminating in essentially no formed vision (eg, light perception only). Im-
mediate evaluation is necessary to determine whether endophthalmitis is present. Sharp
pain following the injection or foreign body sensation is likely caused by surface irritation
or damage from the povidone-iodine solution, the manipulation of the conjunctiva, or
the needle stick. Occasionally, patients may have severe pain that begins within 24 hours
of the injection, which may be caused by corneal abrasions. Small floaters that are noticed
immediately after injection are usually caused by small air bubbles in the injected liquid,
a common occurrence. Less commonly, these floaters are due to silicone oil bubbles from
the syringe lubricant. Air bubbles usually disappear after a few hours, whereas silicone oil
bubbles persist but usually float out of view.
5. a. In patients with diabetic retinopathy, vision loss can be associated with capillary leak-
age, ischemic maculopathy (capillary occlusion), and sequelae from ischemia-induced
neovascularization. Vitreous hemorrhage, macular edema, traction detachment, and
macular ischemia can all cause vision loss. In this patient, macular ischemia (normal fo-
veal avascular zone is approximately 500 μm in diameter) is the most likely cause because
the vitreous hemorrhage is outside the visual axis (inferior) and the macular edema is
noncentral.
6. b. A branch retinal artery occlusion (BRAO) is the result of embolization or thrombosis of
the affected vessel. Common types of emboli include cholesterol emboli arising in the ca-
rotid arteries, platelet-fibrin emboli associated with large-vessel arteriosclerosis, and cal-
cific emboli arising from diseased cardiac valves. Rare causes of a BRAO include emboli
473
474 ● Answers
resulting from cardiac myxoma, long-bone fractures (fat emboli), infective endocarditis
(septic emboli), and intravenous drug use (talc emboli). Subarachnoid hemorrhage may
be associated with intraocular hemorrhage or Terson syndrome. Traumatic pancreatitis
is associated with Purtscher retinopathy, which is characterized by focal areas of retinal
whitening that may superficially resemble a BRAO but are not a BRAO.
7. b. The most important risk factor for central retinal vein occlusion (CRVO) is age; 90%
of affected patients are older than 50 years at the time of diagnosis. Other significant risk
factors for development of CRVO include systemic hypertension, open-angle glaucoma,
hypercoagulable states, diabetes mellitus, and hyperlipidemia.
8. d. The 2008 study Ranibizumab for Macular Edema Following Branch Retinal Vein Occlu-
sion (BRAVO) found ranibizumab to be an effective treatment for macular edema associated
with branch retinal vein occlusion (BRVO); monthly injection of either 0.5 mg or 0.3 mg of
ranibizumab was superior to sham injection for improving visual acuity. Verteporfin is used
in photodynamic therapy and is not an approved treatment for macular edema associated
with BRVO. Fluocinolone acetonide intravitreal implant and panretinal photocoagulation
are not used for treating macular edema associated with BRVO.
9. b. Patients with acute CRVO commonly present with reduced visual acuity secondary to
macular edema. Neovascularization of the iris usually occurs 3 to 5 months after symptom
onset. Peripheral ischemia with capillary dropout is seen in some severe cases of CRVO
and can promote neovascularization, resulting in vitreous hemorrhage. Macular ischemia
can occur as well but is a less likely cause of decreased acuity on presentation.
10. a. Coats disease is clinically evident within the first decade of life and is more common in
boys. It is characterized by a unilateral retinal telangiectasia with mild to severe leakage and
possible exudative retinal detachment. “Foamy” histiocytes and cholesterol crystals in the
subretinal space are common histologic findings. Loss of capillary pericytes and thickening
of the retinal capillary basement membrane may be seen in diabetic retinopathy. Dilated,
thin-walled vascular channels between the retinal pigment epithelium (RPE) and the outer
aspect of Bruch membrane are found in polypoidal choroidal vasculopathy. Focal sclerosis of
retinal vessels with an overlying pocket of liquefied vitreous is found in lattice degeneration.
11. b. Von Hippel–Lindau syndrome is caused by a tumor suppressor gene mutation on the
short arm of chromosome 3 (3p26–p25), the inheritance of which is autosomal dominant
with incomplete penetrance and variable expression.
12. a. All of the conditions listed as answer choices (persistent fetal vasculature, Coats disease,
retinoblastoma, and infection with Toxocara species) can cause unilateral leukocoria. How-
ever, only persistent fetal vasculature is associated with microphthalmos. It is also associated
with elongated ciliary processes, cataract, retinal detachment, and angle-closure glaucoma.
13. b. The term threshold retinopathy of prematurity (ROP) was coined in the mid-1980s by
investigators in the Cryotherapy for ROP (CRYO-ROP) study to define disease with equal
chances of spontaneous regression or progression to an unfavorable outcome. Threshold
disease is characterized by at least 5 contiguous clock-hours of extraretinal neovascular-
ization or 8 cumulative clock-hours of extraretinal neovascularization with plus disease as
well as retinal vessels ending in zone I or II.
14. a. The patient almost certainly has central serous chorioretinopathy (CSC). In most cases,
initial observation, with the expectation of spontaneous resolution of the subretinal fluid,
is the most appropriate management. Photodynamic therapy may be considered for CSC
Answers ● 475
when the patient is symptomatic or when the subretinal fluid does not resolve after a
period of observation. Verteporfin photodynamic therapy (PDT) has been shown to de-
crease or eliminate subretinal fluid; it also decreases choroidal thickness and reduces cho-
roidal vascular hyperpermeability. Laser photocoagulation therapy is no longer preferred
for CSC. Moreover, unlike PDT, photocoagulation has no effect on choroidal thickness.
Steroids are contraindicated in patients with CSC.
15. b. The findings are most consistent with those in acute zonal occult outer retinopathy
(AZOOR), an idiopathic condition that typically affects young women with myopia. Acute
onset of posterior photopsias may occur in many posterior inflammatory conditions, includ-
ing multiple evanescent white dot syndrome (MEWDS) and multifocal choroiditis; however,
the disruption of the outer retina observed on optical coherence tomography (OCT) dif-
ferentiates AZOOR from both MEWDS and multifocal choroiditis. MEWDS is character-
ized by multiple small gray, white, or yellow-white dots at the level of the outer retina in and
around the posterior pole and typically presents unilaterally. Acute macular neuroretinopa-
thy is characterized clinically by reddish-brown teardrop or wedge-shaped lesions around the
fovea, the tips of which point centrally; the lesions correspond in size and location to subjec-
tive paracentral scotomata. Intermediate uveitis is characterized by vitreous cells.
16. c. The clinical presentation is consistent with that of Behçet disease. Panuveitis with oc-
clusive retinal vasculitis is the most common presentation. HLA-B51 is commonly associ-
ated with Behçet disease with ocular manifestations. However, the diagnosis of Behçet is
clinical, and laboratory tests are of little value in confirming the diagnosis. HLA-A29 is
more common in patients with birdshot chorioretinopathy, HLA-B27 is associated with
acute anterior uveitis, and HLA-DR4 is weakly associated with Vogt-Koyanagi-Harada
disease (or syndrome).
17. d. In the rare instances when sympathetic ophthalmia does follow either ocular injury
or surgery, standard treatments such as corticosteroids almost always control the in-
flammation. Therefore, enucleation or evisceration of an injured eye to minimize risk of
sympathetic ophthalmia is rarely practiced. Panretinal photocoagulation has no place in
managing this form of uveitis. Foscarnet is an antiviral antibiotic that is used to treat cyto-
megalovirus retinitis or other herpes viral retinitis, such as that occurring in acute retinal
necrosis or as a complication of iatrogenic or infectious immunosuppression.
18. a. Acute posterior multifocal placoid pigment epitheliopathy (APMPPE) is an uncom-
mon, bilateral inflammatory disease characterized by acute-onset vision loss, usually fol-
lowed by substantial or near-complete improvement weeks to months later. In the acute
stage, fluorescein angiography (FA) of active lesions shows early blockage followed by
progressive late leakage and staining. There is no definitive evidence that treatment with
corticosteroids is beneficial in altering the outcome for APMPPE. Birdshot chorioretinop-
athy is chronic, progressive, and prone to recurrent episodes of inflammation. MEWDS is
typically unilateral. Symptoms include decreased vision, scotomata, and sometimes pho-
topsias. In most patients, the symptoms and fundus findings begin to improve in 2–6
weeks without treatment. Serpiginous choroiditis is an often vision-threatening, recurring
inflammatory disease involving the outer retina, RPE, and inner choroid (the choriocapil-
laris). Persistent scotomata and decreased central vision are common symptoms.
19. c. While intravenous acyclovir 10 mg/kg every 8 hours for 10–14 days may be used for induc-
tion treatment in acute retinal necrosis, oral valacyclovir 2 g 3 times daily is considered to be
an equally effective alternative. Oral acyclovir 800 mg 5 times daily or valacyclovir 1 g 3 times
476 ● Answers
daily is the appropriate maintenance dosage after induction has been completed. Oral val-
ganciclovir is reserved for cytomegalovirus-associated intraocular infections.
20. b. Acute APMPPE is a bilateral inflammatory disease that typically occurs in otherwise
healthy young adults, with men and women affected equally. Patients may have a viral
prodrome. In rare cases, cerebrovasculitis occurs. The characteristic examination find-
ing is multiple yellow-white placoid lesions that vary in size and involve the outer retina
(RPE) and inner choroid (choriocapillaris). MEWDS can also have a viral prodrome but
is usually unilateral. In some patients, a transient foveal granularity also develops and is
highly suggestive of the condition. On initial presentation, eyes with AZOOR may have a
normal-appearing fundus or show evidence of mild vitritis. Permanent visual field loss is
often associated with late development of fundus changes. Depigmentation of large zones
of RPE usually corresponds to scotomata; narrowed retinal vessels may be visible within
these areas. In serpiginous choroiditis, lesions first appear at or near the optic nerve head
and extend centrifugally in a serpentine pattern. With numerous recurrences, a serpigi-
nous (pseudopodial) or geographic (maplike) pattern of chorioretinal scarring develops.
21. a. Acquired color vision defects are most frequently blue-yellow, or tritan, abnormalities;
they affect males and females equally. Blue-yellow color defects often accompany an optic
neuropathy but also can occur in a maculopathy. Hereditary color vision defects are most
frequently red-green abnormalities. They are most often X-linked recessive and affect
5%–8% of males and 0.5% of females.
22. d. Choroideremia is an X-linked recessive degeneration of the choriocapillaris of the cho-
roid, and of the overlying RPE and retina. Patients with choroideremia have nyctalopia
and show progressive peripheral visual field loss over 3–5 decades. Most patients maintain
good visual acuity until a central island of foveal vision is lost.
23. b. In Best disease, the electroretinogram (ERG) response is characteristically normal, and
the electro-oculogram result is almost always abnormal, showing a severe loss of light
response. OCT and FA results are abnormal in Best disease and reflect the changes in the
macular lesion from vitelliform stage to atrophic appearance.
24. b. Best disease is an autosomal dominant maculopathy caused by mutations in the BEST1
(VMD2) gene. The encoded protein bestrophin localizes to the basolateral plasma mem-
brane of the RPE and functions as a transmembrane ion channel.
25. c. Dominant (familial) drusen typically extend beyond the vascular arcades and nasal to
the optic nerve head. A single missense mutation (Arg345Trp) on exon 10 in the EFEMP1
gene of some patients with dominant drusen results in a fibrillin-like extracellular matrix
protein expressed in the RPE and retina. Typically, the clinical findings first manifest bi-
laterally in the third to fourth decade of life. Although usually asymptomatic, choroidal
neovascularization may develop in this patient population and lead to loss of vision. In
contrast to drusen of age-related macular degeneration (AMD), the lesions of dominant
drusen are typically small and round. Clumping and hyperplasia of the RPE and drusen
clustering in the macula are features of AMD, not familial drusen.
26. d. Incontinentia pigmenti (Bloch-Sulzberger syndrome) is an X-linked dominant con-
dition characterized by streaky skin lesions and abnormalities of the teeth and central
nervous system. Ocular involvement includes pigmentary abnormalities as well as pe-
ripheral retinal nonperfusion and neovascularization that may cause traction (also called
tractional) and cicatricial retinal detachment. In incontinentia pigmenti, erythema and
bullae develop in the first few days of life.
Answers ● 477
X-linked retinitis pigmentosa is a retinal degeneration associated with vision loss and
pigmentary retinopathy, but no dermatologic manifestations. The severe form of enhanced
S-cone syndrome (also called Goldmann-Favre syndrome) is also a pigmentary retinopathy
and is associated with pathognomonic ERG abnormalities and nyctalopia but no skin find-
ings. Gyrate atrophy is an autosomal recessive congenital deficiency in ornithine amino-
transferase that leads to large, geographic, peripheral paving-stone–like areas of atrophy of
the RPE and choriocapillaris. It has no association with dermatologic changes.
27. d. Sorsby macular dystrophy (SMD) results from mutations in TIMP3, which plays an
important role in the regulation of extracellular matrix turnover. Early signs of SMD are
yellowish-gray drusenlike deposits or a confluent plaque of yellow material at the level of
Bruch membrane within the macula and along the temporal arcades. The deposits pro-
gress over time to include the central macula and take on the appearance of geographic
atrophy. Vision loss results from expansion of macular atrophy or from development of
subfoveal choroidal neovascularization.
Malattia Leventinese (also called Doyne honeycomb dystrophy) is caused by a mutation
in EFEMP1. The drusen in this condition often show a distinctive pattern of radial exten-
sions of small and intermediate-sized deposits emanating from the fovea. The clinical
syndrome of cuticular (also called basal laminar) drusen features small clustered drusen
and occurs in young adults. Although this syndrome is associated with mutation in com-
plement factor H, it is not a monogenic disorder. Mutations in NYX are associated with
congenital stationary night blindness.
28. c. The clinical description is most consistent with that of Bietti crystalline dystrophy,
which is associated with mutations in CYP4Y2. Oxalosis is a rare inherited disorder of
primary hyperoxaluria, which has been associated with mutations in 3 genes (AGXT,
GRHPR, DHDPSL). Cystinosis is an abnormality of cystine accumulation that may result
from genetic causes (mutation in CTNS) or exposure to some anesthetics. Typically, pa-
tients with oxalosis and cystinosis have renal insufficiency or failure resulting from amino
acid crystallization in the kidneys. Characteristic findings in macular telangiectasia type 2
include a reduced foveolar reflex, loss of retinal transparency (retinal graying), superfi-
cial retinal crystalline deposits, and foveal atrophy, but the disease is not associated with
CYP4V2; it is associated with serine metabolism abnormalities.
29. d. Batten disease, an example of a neuronal ceroid lipofuscinosis, may have retinal find-
ings, including an atrophic bull’s-eye maculopathy, optic nerve pallor, and attenuation of
retinal vessels. The ophthalmic symptoms and signs may be the first, earliest manifesta-
tions of this disorder. The cherry-red spots seen in Tay-Sachs disease result from ganglio-
side accumulation within ganglion cells. Pigmentary abnormalities and fine retinal crys-
tals are present in cystinosis (nephropathic type). Tortuous and dilated retinal vessels, as
well as cornea verticillata and tortuous conjunctival vessels, are present in Fabry disease.
30. a. The clinical presentation described is consistent with ocular albinism, which is typically
transmitted as an X-linked (recessive) trait. Albinism includes a group of different genetic
abnormalities in which the synthesis of melanin is reduced or absent. A number of genes
are associated with both oculocutaneous and ocular albinism. Oculocutaneous albinism
is typically transmitted as an autosomal recessive trait.
31. c. Vitreous opacities may be one of the initial signs of the dominantly inherited form of he-
reditary familial amyloidosis. In addition to the vitreous, amyloid may be deposited in the
retinal vasculature, the choroid, and the trabecular meshwork. Nonocular manifestations of
478 ● Answers
amyloidosis include upper- and lower-extremity polyneuropathy and central nervous system
abnormalities. Congo red staining of vitreous samples can confirm the diagnosis. Asteroid
hyalosis is typically unilateral and is rarely visually significant. Cholesterolosis, or synchysis
scintillans, occurs in eyes with a history of intravitreal hemorrhage usually related to prior ac-
cidental or surgical ocular trauma. Although prior ophthalmic pathology leading to dehemo-
globinized vitreous hemorrhage should always be considered, it is less likely in this scenario.
32. a. Unlike patients with ROP, individuals with familial exudative vitreoretinopathy (FEVR)
are born full term. FEVR is characterized by failure of the temporal retina to vascularize.
Retinal folds and peripheral fibrovascular proliferation, as well as traction and exuda-
tive retinal detachments, are often associated with FEVR. FEVR is usually inherited as
an autosomal dominant trait, but X-linked transmission also occurs. Congenital retinal
telangiectasia and juvenile retinoschisis are not typically associated with a traction retinal
detachment. Incontinentia pigmenti is typically lethal in utero to males.
33. d. The vitreous is most tightly adherent at its base, where the vitreous remains firmly ad-
herent to the anterior retina and pars plana epithelium. This attachment is so strong that it
can lead to vitreous base avulsion after severe trauma. The vitreous is also firmly attached
to the posterior lens capsule, retinal vessels, optic nerve, and fovea. With age, the vitreous
contracts, which can result in a posterior vitreous detachment. With this contraction, the
posterior cortical gel detaches toward the firmly attached vitreous base.
34. d. Atrophic retinal holes are commonly associated with lattice degeneration. They are
infrequently the cause of rhegmatogenous retinal detachment. They represent atrophic
rather than tractional retinal breaks, so they are not caused by posterior vitreous detach-
ment, and they are not associated with vitreous hemorrhage.
35. c. The presence of inner and outer layer holes in retinoschisis is associated with an increased
risk of retinal detachment. However, the presence of outer layer holes can lead to a localized
retinal detachment that usually does not progress and seldom requires treatment. The pres-
ence of hyperopia and typical peripheral cystoid degeneration does not predispose to retinal
detachment. Finally, the presence of a demarcation line in an eye with retinoschisis suggests
that a full-thickness detachment is present or was formerly present and has spontaneously
regressed; this is not likely to lead to a progressive retinal detachment.
36. b. Degenerative retinoschisis is commonly associated with hyperopia. Lattice degenera-
tion is more common in myopic eyes. Collagen vascular disease and prior retinal detach-
ment do not increase the risk of degenerative retinoschisis.
37. c. Stage 3 holes are full-thickness holes at least 400 μm in diameter. Stage 0 holes (pre–
macular hole) represent vitreomacular adhesion (VMA). OCT examination shows that a
stage 1A hole is a foveal “pseudocyst,” or horizontal splitting (schisis), associated with vitre-
ous traction on the foveal center. A stage 1B hole indicates a break in the outer fovea. Stage
2 holes are early full-thickness holes less than 400 μm in diameter. Stage 4 holes are full-
thickness holes at least 400 μm in diameter in association with a complete posterior vitreous
detachment.
38. d. The chalcosis and siderosis bulbi that result from retaining intraocular copper and iron,
respectively, have distinct manifestations. Pure copper can cause an intense inflammatory
response, whereas a copper content less than 85% can result in chronic chalcosis, with find-
ings that include sunflower cataract and a Kayser-Fleischer–like ring of deposits in Des-
cemet membrane. Intraocular iron deposits in neuroepithelial tissues produce findings that
include retinal pigmentation. ERG can be used to assess retinal toxicity in siderosis bulbi.
Answers ● 479
39. b. Severe blunt trauma can cause scleral rupture. The most common sites of scleral rupture
are at the corneal limbus and through areas of physiologic scleral thinning parallel to and
under the rectus muscle insertions. Scleral rupture is associated with markedly decreased
ocular ductions; boggy conjunctival chemosis with hemorrhage; deepened anterior cham-
ber; severe vitreous hemorrhage; and usually hypotony, although intraocular pressure may
also be normal or elevated. In this case, suspicion of occult scleral rupture is high, warrant-
ing further exploration, with disinsertion of 1 of more rectus muscles in order to identify
the rupture site. Scleral lacerations or ruptures are typically closed with 7-0, 8-0, or 9-0 non-
absorbable suture. Some specialists recommend prophylactic vitrectomy and/or placement
of a scleral buckle at the time of primary repair of some perforating ocular injuries, but the
primary goal of most open-globe repairs is to achieve corneoscleral wound closure.
40. b. Retinal hemorrhages are the cardinal manifestation of abusive head trauma (AHT, for-
merly called shaken baby syndrome). The presenting sign of child abuse involves the eye
in approximately 5% of cases. Ocular signs include retinal hemorrhages and cotton-wool
spots, retinal folds, hemorrhagic schisis cavities, and pigmentary maculopathy. The ret-
inal hemorrhages associated with AHT often have a hemispheric contour. Papilledema
occurs in less than 10% of cases of AHT. Cranial nerve paresis and exudative retinal de-
tachment are not commonly associated with AHT.
41. b. Nonexudative AMD, macular holes, and cystoid macular edema can decrease central
visual acuity; however, in the setting of recent (and likely prolonged) ophthalmic sur-
gery and outer retinal lucency, photic retinopathy secondary to exposure to the operating
microscope light source is the most likely diagnosis. Although this retinopathy is more
common following retinal surgery, it can occur with surgery times as short as 30 minutes;
prolonged focal exposure can cause photochemical injury to the fovea, much like solar
retinopathy. Patients typically present with central or paracentral scotomata, hypopig-
mented foveal lesions, and outer retinal cavitation. Most patients recover vision somewhat
over a period of 3–6 months.
42. d. The clinical presentation described is consistent with commotio retinae, and the ap-
propriate management is observation. High-dose intravenous steroids have no role in the
management of commotio retinae. Anterior chamber paracentesis may be used in cases
of acute retinal artery occlusion but has no role in commotio retinae. Laser photocoagula-
tion may be used for a retinal break but is not used for commotio retinae.
43. a. When suprachoroidal hemorrhage occurs intraoperatively, immediate drainage is rarely
effective, because of rapid coagulation of the blood. Waiting 7 to 14 days often allows the
hemorrhage to liquefy. B-scan ultrasonography is useful to help identify clot liquefaction.
It is also useful to determine whether appositional (“kissing”) suprachoroidal hemorrhage
is present and whether there is concurrent retinal detachment. In the presence of a large
suprachoroidal hemorrhage, often no view is possible for OCT or FA. Computed tomog-
raphy is not indicated for monitoring suprachoroidal hemorrhage.
44. a. For a total retinal detachment with proliferative vitreoretinopathy (PVR) and multiple pos-
terior retinal breaks, pars plana vitrectomy with or without scleral buckle is indicated, along
with membrane peeling to relieve traction, retinectomy (in case of incarceration or retinal
shortening), flattening retina by using air or perfluorocarbon liquid, endolaser photocoagula-
tion or cryopexy, and gas or silicone oil tamponade. Pneumatic retinopexy is contraindicated
in the presence of PVR. Laser demarcation is not effective for a total retinal detachment. Scleral
buckle alone is typically not effective in the presence of multiple posterior retinal breaks.
480 ● Answers
2. A patient with previous laser in situ keratomileusis (LASIK) and a refraction of −0.75 D
undergoes surface ablation. What is the reason to use mitomycin C (MMC) in this case?
a. In a patient with previous LASIK, MMC can prevent haze after surface ablation.
b. MMC is approved by the US Food and Drug Administration (FDA) for the prevention
of post–photorefractive keratectomy (PRK) haze.
c. This small amount of treatment does not present a high risk of scarring, and MMC
should be avoided because of potential toxicity.
d. Irrigation of the cornea with MMC is performed to reduce pain and haze.
209
210 ● Study Questions
4. A 45-year-old patient with myopia desires monovision correction with LASIK. The non-
dominant right eye is chosen for near vision and has a refraction of −5.00 sphere. As-
suming no nomogram adjustment is required, how many diopters of refractive treatment
would be appropriate to program into the laser?
a. −3.50 D
b. −6.50 D
c. −0.75 D
d. −1.50 D
5. In refractive surgery, MMC is commonly employed in what concentration and for what
duration?
a. 0.002% for 2–4 minutes
b. 0.02% for 12–120 seconds
c. 0.2% for 12–120 seconds
d. 2.0% for 10–30 seconds
6. Which organism is the most common pathogen in cases of infectious keratitis after
LASIK?
a. Pseudomonas spp
b. Staphylococcus spp
c. Mycobacterium spp
d. Nocardia spp
8. Haze following PRK typically peaks approximately how long after surgery?
a. 1–2 days
b. 1–2 weeks
c. 1–2 months
d. 6–8 months
10. What is a risk factor for delayed epithelial healing after PRK?
a. decentered ablation
b. keratoconjunctivitis sicca
c. hyperopia
d. high myopia
11. Which wavelength of energy, when applied to the cornea, is characterized by high energy
and precision, low penetration of tissue, and limited thermal spread?
a. 193 nm
b. 365 nm
c. 1053 nm
d. 2.13 µm
12. What is the most common risk factor for epithelial ingrowth following LASIK?
a. epithelial defect near flap edge
b. macrostriae
c. Fuchs dystrophy
d. high refractive error
13. Which refractive surgery places a patient at highest risk for a ruptured globe from blunt
trauma?
a. LASIK
b. radial keratotomy (RK)
c. refractive lens exchange with sclerocorneal incision
d. refractive lens exchange with clear corneal incision
14. What is an appropriate initial option to improve myopic astigmatism caused by post-
LASIK corneal ectasia?
a. wavefront-guided enhancement
b. refractive lens exchange
c. radial and astigmatic keratotomy
d. rigid gas-permeable contact lens
15. Why might increasing the optical zone in myopic LASIK or PRK limit the ability to treat
patients with thinner corneas?
a. It increases the risk of postoperative glare and halos.
b. It increases the chance of regression.
c. It decreases predictability of the refractive outcome.
d. It increases the ablation depth.
212 ● Study Questions
16. A 26-year-old patient undergoes LASIK for the treatment of −9.00 D myopia. The oph-
thalmologist notes slightly decreased vision and glare symptoms 1 week postoperatively.
The examination findings are normal with the exception of flap striae. What is the most
likely cause of the striae?
a. diffuse lamellar keratitis
b. high myopic treatment
c. ectasia
d. epithelial ingrowth
17. A patient presents with nuclear cataracts in both eyes. He has a history of a bilateral
16-incision RK procedure with a 3-mm optical zone. His corrected distance vision is
limited to 20/60 in both eyes related to the cataracts, and he undergoes cataract surgery
and intraocular lens (IOL) implantation. At his 2-week postoperative visit, his vision is
corrected to 20/25+ with a measured refractive error of +1.50 –0.50 × 120. He is very un-
happy with his uncorrected vision, as he had hoped for either an emmetropic or a slight
myopic outcome. What would be the best option at this point?
a. Perform an IOL exchange, leaving him hyperopic.
b. Plan a surface ablation to correct his hyperopic outcome.
c. Inform him that this is a typical outcome in RK eyes and that glasses are the best option.
d. Assure him that the hyperopia might lessen with resolution of corneal edema.
19. One month after a patient undergoes PRK, subepithelial haze develops. What is the pre-
ferred initial management?
a. topical MMC
b. oral prednisone
c. observation
d. phototherapeutic keratectomy
21. A 48-year-old woman has a persistent central corneal epithelial defect 2 weeks after PRK.
She has a bandage soft contact lens in place and is using preservative-free artificial tears
and topical nonsteroidal anti-inflammatory drugs (NSAIDs) every 2 hours. Her Schirmer
test scores are normal. What is the best initial management option?
a. Place a lower eyelid punctal plug.
b. Discontinue topical NSAIDs.
c. Add topical cyclosporine 0.05% eyedrops 2 times daily.
d. Add topical prednisolone acetate 1% eyedrops 4 times daily.
23. How are the incisions made for limbal relaxing incisions different from those made for
arcuate keratotomy?
a. more beveled
b. more peripheral
c. shorter
d. shallower
24. What medical history finding in a 25-year-old man would be a relative contraindication
for LASIK?
a. asthma
b. seropositivity for HIV
c. depression
d. multiple sclerosis
25. In LASIK, an RSB thickness of less than 250 µm is associated with which postoperative
complication?
a. haze
b. ectasia
c. epithelial ingrowth
d. diffuse lamellar keratitis
26. A patient who presents 8 months after penetrating keratoplasty (PKP) with the concern
of poor vision due to astigmatism is interested in laser refractive surgery. What is the ap-
propriate recommendation for this patient?
a. PRK is the best option because creation of a LASIK flap risks dehiscence of the graft.
b. The patient should wait at least 1 year after PKP, and the refraction should be stable.
c. LASIK is the best option to avoid postoperative haze induced by PRK.
d. Because of the risk of graft dehiscence and the unpredictable nature of the results, laser
refractive surgery should be avoided in patients who have undergone PKP.
214 ● Study Questions
27. How does laser refractive surgery affect the risk of retinal detachment in patients with
myopia?
a. The rapid increase and decrease in intraocular pressure during the procedure can
stretch the vitreous base and lead to a posterior vitreous detachment (PVD), thereby
increasing the risk of retinal detachment after refractive surgery.
b. Acoustic shock waves from the laser could lead to the development of a PVD, thereby
increasing the risk of retinal detachment after refractive surgery.
c. Current data show that laser refractive surgery does not appear to increase or decrease
the risk of retinal detachment.
d. By eliminating myopia, refractive surgery reduces the risk of retinal detachment.
28. A patient experiences pain and light sensitivity 2 months after femtosecond laser–assisted
LASIK. The eye appears white and quiet. What is the best initial treatment?
a. flap lift with culture for suspected microbial keratitis
b. intensive treatment with topical corticosteroids
c. intensive treatment with topical antibiotics
d. flap lift and irrigation of the lamellar space with sterile saline
29. Immediately after cataract surgery with a diffractive multifocal IOL, a patient reports vi-
sually disabling glare and halos that persist over the next several months. What is the best
next step in management?
a. IOL exchange with a refractive multifocal IOL
b. IOL exchange with a monofocal IOL
c. Nd:YAG capsulotomy; and, if that fails, an IOL exchange
d. piggyback IOL
30. A 42-year-old man with adult-onset diabetes mellitus reports worsening vision at distance
over the past 6 months. He has not worn eyeglasses or contact lenses but asks about the
possibility of LASIK to correct his vision. He says that his blood glucose levels have ranged
between 175 and 350 mg/dL (9.642–19.284 mmol/L) during the past 2 years, and his most
recent HbA1c was 8.5. Corrected distance visual acuity is 20/15 in each eye (right eye, 2.50
sphere; left eye, 2.00 sphere), and findings from the ophthalmologic evaluation are other-
wise normal. What is the most appropriate initial management?
a. contact lens fitting
b. eyeglass correction
c. improving glucose control prior to reevaluation
d. LASIK surgery
Study Questions ● 215
31. Which condition might prevent a 25-year-old patient from being a good candidate for
PRK?
a. pregnancy
b. posterior polymorphous corneal dystrophy
c. high myopia
d. asthma
32. How does a Placido disk–based corneal topography device determine elevation changes?
a. scanning slit beam of light swept across the cornea
b. laser reflected off the retina and captured by a lenslet array
c. image of a series of concentric rings reflected off the cornea
d. ultrasonic image of the corneal surface
33. In the creation of a lamellar flap for LASIK, what advantage does the femtosecond laser
offer compared with a microkeratome?
a. shorter procedure time
b. lower total treatment cost
c. more predictable flap thickness
d. increased iris registration success
34. What characteristic would indicate that a patient is a poor candidate for LASIK?
a. predicted postoperative keratometry of 52.00 D
b. inability to make wavefront measure
c. history of keloid formation
d. RSB thickness of 300 µm
35. Why are accurate IOL calculations difficult in a patient who has had laser refractive surgery?
a. Most traditional keratometers measure only the central corneal curvature.
b. The axial length cannot be accurately measured.
c. Tear film abnormalities make it difficult to obtain keratometry readings.
d. The relationship between the anterior and posterior cornea has changed.
38. Which process is responsible for securing a LASIK flap immediately after surgery?
a. endothelial pump activity
b. corneal scarring
c. corneal epithelialization and spreading
d. flap position and shape
39. Binocular diplopia can occur in patients after LASIK as a result of which problem?
a. decompensated phorias
b. optic nerve ischemia
c. corneal edema
d. flap striae
40. What is the mechanism through which the femtosecond laser interacts with tissues?
a. photocoagulation
b. photoablation
c. photodisruption
d. photothermal effects
Answers
1. d. Wavefront aberrometry is used to assess the total refractive error of the eye, includ-
ing both lower-order and higher-order aberrations. Of the 4 methods that are available
clinically, Hartmann-Shack is the most common. In this technique, light from a low-
power laser reflected off the retina is used as a point source of light. This light travels back
through the optical system of the eye, then passes through an array of lenslets and focuses
onto a detector. The divergence of the image from each lenslet from the expected focal
point enables the reconstruction of the total wavefront. Thin-beam single-ray tracing,
Tscherning, and optical path difference are other methods of aberrometry. Scheimpflug
imaging is used in certain corneal topographers to determine the shape of the cornea.
Spectral-domain optical coherence tomography is used in imaging but does not help to
measure the refractive power of the eye.
2. a. Mitomycin C (MMC) is used off label to decrease the chance of corneal haze develop-
ment after surface ablation in moderate to high treatments. It is also used even in small
treatments in patients who previously had photorefractive keratectomy (PRK), laser in situ
keratomileusis (LASIK), penetrating keratoplasty (PKP), or radial keratotomy (RK). MMC
is not approved by the US Food and Drug Administration (FDA) for use in refractive sur-
gery. The cornea is copiously irrigated with balanced salt solution (BSS) to remove MMC,
and some surgeons apply chilled BSS after surface ablation in the belief that this reduces
pain and haze formation.
3. d. The therapeutic range of small incision lenticule extraction (SMILE) is more limited than
that of excimer treatments. The SMILE procedure (at the time of this publication) is ap-
proved for the treatment of myopia with or without astigmatism from −1.00 D to −10.00 D
sphere, and from −0.75 D to −3.00 D cylinder with a manifest refraction spherical equiva-
lent greater than −10.00 D. It is not approved for hyperopic treatments. However, SMILE is
thought to be more stable biomechanically, less disruptive of anterior corneal innervation,
and less affected by local environmental factors.
4. a. Monovision to modify presbyopia in phakic individuals can be an effective strategy
for reducing dependence on spectacles. The process involves intentionally undercorrect-
ing a myopic patient, overcorrecting a hyperopic patient, or inducing mild myopia in an
emmetropic patient. Many refractive surgeons target mild myopia after treatment (−0.50
to −1.50 D) to achieve good uncorrected near- to intermediate-distance vision, making
a treatment of −3.50 D (leaving a residual of −1.50 D) the correct answer choice. Mini-
monovision would entail correction to achieve the lower end of myopia (−0.50 to −0.75 D)
for the near-vision eye, resulting in only a mild decrease in distance vision, retention of
good stereopsis, and a significant increase in intermediate-zone function. Blended vision
would entail correction to achieve myopia in the range of −1.00 to −1.50 D.
5. b. MMC is commonly used off label to reduce the formation of haze after surface abla-
tion procedures such as PRK or phototherapeutic keratectomy (PTK). Typical concentra-
tions used are 0.01%–0.02%, applied for 10–120 seconds. Complications caused by dosage
errors include corneal melting; other complications include infectious keratitis, delayed
corneal healing, and corneal scarring.
6. b. Post-LASIK infectious keratitis typically begins 2–3 days after surgery and features a
focal inflammatory reaction not necessarily confined to the flap interface. Often, there
217
218 ● Answers
is an inflammatory deep corneal and/or anterior chamber response. The pathogens that
most commonly cause keratitis after laser ablation are the gram-positive bacteria Staphy-
lococcus and Streptococcus spp. The next most common causative organisms are atypi-
cal Mycobacterium spp, Nocardia asteroides, and fungi. If infection is suspected, the flap
should be lifted and the interface cultured and irrigated with antibiotics.
7. a. Ectasia is far more common after LASIK than after PRK. This is related to the residual
stromal bed (RSB) thickness, which is far less after LASIK flap creation than after surface
ablation. Intrastromal corneal ring segments (ICRS) can be used to reduce the irregular
astigmatic error from ectasia but cannot prevent ectasia progression. Epikeratophakia, a
largely obsolete procedure, featured the suturing of a donor lenticule on top of the Bow-
man layer of the host cornea.
8. c. Subepithelial corneal haze develops as a result of epithelial–stromal wound healing and
appears several weeks after surface ablation, peaks in intensity at 1–2 months, and gradu-
ally diminishes or disappears over the following 6–12 months.
9. a. Arcuate, or astigmatic, keratotomy can be quite useful for the surgical management of
astigmatism. The closer the incisions are to the center of the cornea, the more effect they
have—and the greater their potential for inducing visually significant irregular astigmatism.
An optical zone of 7–9 mm is typical, while a 4-mm optical zone is too narrow and would
have a high incidence of irregular astigmatism. The incisions will flatten the cornea in the
meridian of the incision and steepen the cornea 90° away when performed in an arcuate
fashion, which is the most common pattern used for astigmatism. In this specific patient, in-
cisions centered at 135° and 315° would increase the astigmatism and are in the wrong axis. It
is critical to ensure that the incisions are performed in the correct axis for arcuate keratotomy.
10. b. Patients with ocular surface disease may have problems with epithelial healing. This
is especially true in patients with large epithelial defects that occur after surface ablation
procedures, such as PRK.
11. a. A variety of laser–tissue interactions are used in keratorefractive surgery. Argon-fluoride
excimer laser generates a wavelength of 193 nm with high energy and precision but low
tissue penetrance and little thermal spread, making this laser highly suitable for use on
the ocular surface. Femtosecond laser generates a wavelength of 1053 nm that causes pho-
todisruption, which transforms tissue into plasma with high pressure and temperature
and leads to rapid tissue expansion. This forms microscopic cavities that are useful in
the creation of stromal flaps and channels. Holmium:YAG laser generates a wavelength
of 2.13 µm that has a photothermal effect. When applied to the anterior stroma, water in
the cornea absorbs the energy and creates heat that shrinks surrounding collagen. This
is useful in causing corneal steepening at the anterior surface, and this technique (laser
thermokeratoplasty) is approved by the FDA for the treatment of low hyperopia. Collagen
crosslinking devices produce a wavelength of 365 nm (ultraviolet A) that, while not used
in refractive surgery, is used in the treatment of corneal ectasia.
12. a. Epithelial ingrowth is reported to occur in less than 3% of eyes undergoing LASIK.
Epithelial defects along the flap edge are a major risk factor, as stromal edema can lead to
poor flap adherence early after the surgery. This can allow epithelium to heal under the
flap rather than over the top of the flap. Care should be taken to prevent epithelium from
being caught under the flap at the time of LASIK. An additional risk factor is secondary
flap lifts, such as those performed for an enhancement or surgical repositioning of flap
striae. Although there can be a greater tendency for the flap to adhere poorly in the eyes
Answers ● 219
of patients with Fuchs dystrophy, this complication is unlikely to occur unless stromal
edema is present. There does not appear to be any correlation with high refractive errors
or flap striae.
13. b. Radial keratotomy (RK) incisions weaken the structure of the eye for years after the
procedure. Traumatic injury can lead to rupture of RK incisions and has been reported
up to 13 years after RK. Although refractive lens exchange with a sclerocorneal or clear
corneal incision also creates a weak point, these incisions are not as susceptible to rupture
as RK incisions. Full-thickness wounds are not created in LASIK, so there is less risk for a
ruptured globe after LASIK than after RK.
14. d. Rigid gas-permeable (RGP) contact lenses are the gold standard for the correction of
reduced vision due to ectasia. Surgical procedures that thin or destabilize the cornea (eg,
LASIK, PRK, incisional procedures) might complicate the situation further. Ectasia can be
progressive, so refractive lens exchange is also contraindicated. Because the contact lens
fit and power can be modified as the ectasia progresses, RGP contact lenses are the most
appropriate treatment. Corneal crosslinking is the first-line treatment to prevent further
ectasia, but it does not improve vision as much as RGP contact lenses.
15. d. All photoablative procedures result in the removal of corneal tissue. The depth of myo-
pic laser ablation by a broad-beam laser is estimated by the Munnerlyn formula:
Degree of Myopia (D) × Optical Zone Diameter [OZ (mm)]2
Ablation Depth (µm) ≈
3
As the ablation diameter increases, the depth required for a given refractive correction in-
creases by the optical zone squared. Thus, the increase in ablation depth is not linear and
can rapidly exceed the structural capacity of the cornea to achieve the ablation depth and
still maintain appropriate RSB thickness. However, complications of glare, halos, and re-
gression increase when the optical zone decreases, and increasing the optical zone would
actually improve these issues. For this reason, the optical zone should be 6 mm or larger.
16. b. When present, 95% of flap striae are noted by 1 week postoperatively. Risk factors in-
clude excessive irrigation under the flap at the time of surgery, thin flaps, or large ablations
with subsequent mismatch of the flap to the underlying stroma. In this clinical situa-
tion, the striae are most likely due to the high myopic treatment. Diffuse lamellar keratitis
(DLK) presents with haze near the edge of the flap to central diffuse haze. It can occur
after a primary procedure or after flap lift or trauma. It is an inflammatory process that
results in the accumulation of white blood cells in the interface. DLK does not present as
flap striae. Ectasia does not occur at 1 week postoperatively and does not present as striae.
Ingrowth is more likely to occur in patients with an epithelial defect at the time of surgery
or after re-treatments.
17. d. After cataract surgery on eyes that had prior RK, short-term flattening of the cornea
and hyperopic shift are frequently noted as a result of the corneal edema after surgery.
Therefore, in the event of a refractive surprise, neither an intraocular lens (IOL) exchange
nor PRK should be considered in post-RK eyes until the cornea and refraction stabilize,
which can take several weeks to months.
18. a. The best way to prevent flap folds, or macrostriae, is meticulous flap realignment with
minimal irrigation. Risk factors for the development of macrostriae include aggressive ir-
rigation under the flap during LASIK flap repositioning (such as to remove debris), thin
flaps, deep ablations with flap–bed mismatch, and postoperative flap slippage. In addition,
220 ● Answers
the use of hypotonic saline or sterile distilled water as the interface irrigating solution
may cause flap swelling and reduce flap diameter, delay flap adhesion, and worsen striae
after the flap dehydrates. When present, macrostriae almost always become apparent in
the early postoperative period. It is important to recognize and address this complication
early, as the success of interventions decreases greatly over time, and this problem can
cause visually significant irregular astigmatism. The surgeon must examine the surgical
eye(s) carefully to check for the presence of gross full-thickness folds (intraoperatively) or
flap slippage (postoperatively).
19. c. Although subepithelial haze can occur early or late after PRK, the incidence peaks at
1 to 2 months and gradually diminishes over 1 year. Increased numbers and activity of
stromal keratocytes may be the source of extracellular deposits. Haze is known to resolve
spontaneously with normal wound remodeling; thus, observation is indicated soon after
surgery. If clinically significant haze persists, superficial keratectomy or phototherapeutic
keratectomy (PTK) may be performed but should be delayed for 6 to 12 months. Topical
MMC may be used to prevent recurrence after superficial keratectomy or PTK. Steroids,
typically applied topically, can be increased in patients with haze and in patients who are
undercorrected to improve both haze and residual refractive error. Oral prednisone would
not be indicated.
20. b. The Kamra inlay (CorneaGen) uses a 1.6-mm central aperture to produce a pinhole
effect and increase depth of field in the nondominant eye, thus providing near vision. The
Flexivue Microlens (Presbia; not FDA approved) is a hydrophilic acrylic clear inlay with
a different index of refraction than the cornea and a central aperture for distance vision.
A hydrogel inlay that steepened the central cornea to induce myopia (Raindrop Near Vi-
sion Inlay; Revision Optics) was formerly available but has been withdrawn. Conductive
keratoplasty uses radiofrequency energy to shrink collagen and steepen the central cornea
to achieve myopia.
21. b. Topical nonsteroidal anti-inflammatory drugs (NSAIDs) can be applied after surface
ablation to reduce postoperative pain, but they can slow epithelialization and result in
corneal melting, stromal scarring, and irregular astigmatism. Corneal epithelial defects
after surface ablation typically heal in 3–4 days postoperatively. Topical NSAID drops
should be discontinued for any patient who is not healing normally after refractive sur-
gery. A frequent cause of delayed epithelialization is keratoconjunctivitis sicca, which can
be treated with increased lubrication, topical cyclosporine, and/or temporary punctal oc-
clusion; however, this patient has normal Schirmer test scores and is already receiving
frequent preservative-free artificial tears, so these measures are not indicated as first-line
treatments. Topical steroidal anti-inflammatory drops such as prednisolone acetate will
not likely improve epithelialization in this case.
22. c. Optical aberrations measured by wavefront analyzers, such as the Hartmann-Shack
wavefront sensor, can be described by an infinite number of systems. Zernike polynomi-
als, one set of wavefront descriptors, are the mathematical formulas used to describe sur-
faces. The most important Zernike coefficients affecting visual quality are coma, trefoil,
and spherical aberration. Coma and trefoil are third-order aberrations. With coma, light
rays at one edge of the pupil come into focus before rays at the opposite edge; the effective
image resembles a comet, having vertical and horizontal components. Coma is common
in patients with decentered corneal grafts, keratoconus, and decentered laser ablations.
Lower-order (second-order) aberrations include myopia, hyperopia, and regular astigma-
Answers ● 221
tism. Myopia produces positive defocus, hyperopia produces negative defocus, and regu-
lar (cylindrical) astigmatism produces a wavefront aberration that has orthogonal and
oblique components. Spherical aberration, a fourth-order aberration, is the most clini-
cally significant higher-order aberration. It increases depth of field, but decreases contrast
sensitivity, causes night myopia, results in halos around point images, and is commonly
increased after myopic LASIK and surface ablation.
23. b. The closer an incision is to the center of the visual axis, the greater the effect on the
refractive error of the eye. Arcuate keratotomy (AK) incisions have been made at various
optical zones. Limbal relaxing incisions (LRIs) are placed closer to the limbus than AK
incisions. In general, to achieve a similar effect, LRIs should be longer than AK incisions,
although this is not specific to the definition. In addition, the cornea is thicker in the pe-
riphery; therefore, LRIs should generally be deeper than AK incisions for the equivalent
refractive result. The degree of beveling is not specifically different for most surgeons’
technique for astigmatic incisions.
24. b. Immunocompromised status, including HIV infection or AIDS, represents a relative
contraindication to refractive surgery because of the potential for increased risk of in-
fectious complications. The FDA recommends that patients with an immunodeficiency
disease not undergo LASIK because the risk outweighs the benefits. Although the other
responses do not represent contraindicated states per se, it is important to assess for con-
ditions associated with each of them: atopy in the case of asthma, optic neuritis in the
case of multiple sclerosis, and dry eye disease due to antidepressant use in the case of
depression.
25. b. While exact mechanisms of postrefractive corneal ectasia are not fully known, an RSB
thickness of less than 250 µm is a recognized risk factor for this postoperative compli-
cation. Flap creation thicker than anticipated, preoperative forme fruste keratoconus,
higher myopic correction, thin corneas, and multiple laser ablations are also risk factors.
However, no single risk factor is an absolute predictor of ectasia after LASIK. In order to
reduce the risk of this complication, surgeons should review topography and perform
pachymetry prior to surgery. In addition, the RSB thickness should be calculated. It can be
estimated preoperatively by subtracting the anticipated flap thickness plus ablation depth
from the central corneal thickness; however, intraoperative pachymetry is the most ac-
curate way to measure flap thickness and calculate the RSB thickness to confirm whether
it is safe to proceed with ablation. Although the other 3 answer choices are complications
of LASIK, they are not related to the RSB and can occur at any RSB thickness.
26. b. Laser refractive surgery is an option for improving ametropia after PKP. Most surgeons
advocate waiting at least 1 year after transplant and an additional 4 months after sutures
are removed. Moreover, refraction and corneal topography should be stable, as measured
on 2 consecutive visits at least 1 month apart. Both LASIK and PRK are options for pa-
tients who have had PKP. To avoid postoperative haze associated with PRK, MMC should
be used. LASIK after PKP is subject to the same patient evaluation constraints as con-
ventional LASIK. There is a small but significant risk of wound dehiscence during both
LASIK and PRK; therefore, the graft–host interface must be carefully inspected for areas
of variable coaptation. Although the results of laser refractive surgery in post-PKP eyes
are less accurate and predictable than in eyes with naturally occurring refractive errors,
the surgery can still significantly reduce refractive error and improve the visual outcome
when combined with glasses or contact lenses.
222 ● Answers
27. c. Patients with high myopia are at a higher risk of retinal detachment, but LASIK and
PRK do not appear to increase the risk for these patients compared with patients in simi-
lar populations who have not had laser refractive surgery. While it has been proposed that
intraocular pressure changes or acoustic shock waves from the laser can induce a poste-
rior vitreous detachment, studies do not show an increased retinal detachment risk in
patients after laser refractive surgery. Although laser refractive surgery eliminates the re-
fractive component of myopia, the natural history of the highly myopic eye is unchanged.
Therefore, the elevated retinal detachment risk of a myopic eye remains.
28. b. Postoperative photophobia with good acuity is a rare entity that can occur weeks to
months after femtosecond laser–assisted LASIK. Patients develop acute onset of pain and
photophobia in a white and quiet eye in which both the cornea and the flap interface ap-
pear normal. The treatment consists of frequent application of topical corticosteroids and
cyclosporine. The flap does not have to be touched.
29. b. All multifocal lenses have the potential to cause halos. Many patients experience a de-
crease in halos over several months, possibly from neuroadaptation. However, if the halos
are persistent and disabling, an IOL exchange with a monofocal IOL may be the only
option. A piggyback IOL would not address the halos. Although patients with multifocal
IOLs may be more sensitive to the visual effects of posterior capsule opacification (PCO),
symptoms from PCO are typically not present initially but gradually develop over the
first few weeks to months after surgery. In contrast, intrinsic IOL symptoms appear early,
if not immediately, in the postoperative course and stay consistent over time. Moreover,
performing an Nd:YAG capsulotomy would make future lens exchanges more difficult.
30. c. Refractive error can fluctuate with changes in the blood glucose level, so the blood
sugar of a patient with diabetes must be well controlled at the time of evaluation to ensure
an accurate refraction. For this reason, diabetic patients with labile blood glucose should
be carefully counseled if eyeglasses or contact lenses are to be prescribed. Elective ocular
surgery should not be performed in a diabetic patient with poor or erratic blood glucose
control.
31. a. Pregnancy and breastfeeding can cause a temporary change in refraction, which makes
refractive surgery potentially less accurate. Many surgeons recommend waiting at least 3
months after delivery and cessation of breastfeeding before performing the refractive sur-
gery evaluation and procedure. High myopia, posterior polymorphous corneal dystrophy,
and asthma are not contraindications for PRK.
32. c. A Placido disk–based corneal topography instrument captures and analyzes the image
of a series of concentric rings reflected off the corneal surface. The computer measures
the distance from the edge of each ring to the next ring along multiple semi-meridians
and generates a map of the corneal surface that would be required to produce the captured
image. These data are frequently expressed in pseudocolor maps. The system can present
the pseudocolor maps in several ways, including axial curvature, tangential curvature, and
best-fit sphere; it can also show an image of the rings themselves as seen by the computer.
33. c. Several studies have compared the benefits of the mechanical microkeratome with
those of femtosecond lasers in creating flaps. Minimal differences between the techniques
have been found for most patients. However, the flap thickness achieved with the femto-
second laser is generally more predictable, with less variability than flaps created with a
mechanical microkeratome. See Chapter 4, Table 4-1, which summarizes the advantages
and disadvantages of the femtosecond laser.
Answers ● 223
40. c. The femtosecond laser is used to create flaps for LASIK, channels for stromal inlays,
and keratoplasty incisions, as well as in femtosecond laser–assisted cataract surgery. It
works by a mechanism called photodisruption. This process occurs when tissue is changed
into plasma that leads to the formation of stromal microscopic cavities. Contiguous pho-
todisruption allows for creation of the corneal flap, channel, or incision. Photothermal
laser effects cause tissue modification as illustrated by use of a holmium:YAG laser, which
generates heat that results in collagen shrinkage. An example of photoablation is the ex-
cimer laser used in LASIK and PRK. This process breaks chemical bonds within tissue.
Photocoagulation is primarily used in retinal lasers and results in a thermal burn to tissue.